diff --git "a/AUS/test.tgt.txt" "b/AUS/test.tgt.txt" new file mode 100644--- /dev/null +++ "b/AUS/test.tgt.txt" @@ -0,0 +1,102 @@ +HIGH COURT OF AUSTRALIA 13 March 2019 NORTHERN TERRITORY OF AUSTRALIA v MR A. GRIFFITHS (DECEASED) AND LORRAINE JONES ON BEHALF OF THE NGALIWURRU AND NUNGALI PEOPLES & ANOR; COMMONWEALTH OF AUSTRALIA v MR A. GRIFFITHS (DECEASED) AND LORRAINE JONES ON BEHALF OF THE NGALIWURRU AND NUNGALI PEOPLES & ANOR; MR A. GRIFFITHS (DECEASED) AND LORRAINE JONES ON BEHALF OF THE NGALIWURRU AND NUNGALI PEOPLES v NORTHERN TERRITORY OF AUSTRALIA & ANOR [2019] HCA 7 Today, the High Court unanimously allowed two appeals in part and dismissed one appeal from a judgment of the Full Court of the Federal Court of Australia, in relation to the compensation payable, pursuant to s 51 of the Native Title Act 1993 (Cth), to the Ngaliwurru and Nungali Peoples ("the Claim Group") for the extinguishment of their non-exclusive native title rights and interests. The Court awarded compensation for economic loss equating to 50 per cent of the freehold value of the affected land with simple interest, and compensation for cultural loss in the amount of $1.3 million. The Northern Territory was responsible for 53 acts held to have impaired or extinguished the Claim Group's native title rights and interests, which gave rise to the Claim Group's entitlement to compensation on just terms under s 51 of the Native Title Act ("the compensable acts"). At issue was the amount of compensation payable to the Claim Group. The trial judge held the Claim Group was entitled to an award for economic loss equating to 80 per cent of the freehold value of the affected land, simple interest on that award, and compensation for cultural loss in the amount of $1.3 million. On appeal, the Full Court varied the trial judge's assessment of economic loss from 80 per cent to 65 per cent of the freehold value of the land, but otherwise, relevantly, affirmed the trial judge's decision. By grants of special leave, the Claim Group, the Northern Territory and the Commonwealth appealed to the High Court. The Claim Group contended among other things that its economic loss equated to the freehold value of the affected land without reduction. The Northern Territory and the Commonwealth contended among other things that the value of the Claim Group's economic loss did not exceed 50 per cent of the freehold value of the affected land, and the award for cultural loss was manifestly excessive. The High Court unanimously allowed the Northern Territory and Commonwealth appeals in part, and dismissed the Claim Group's appeal. The Court held, by majority, that the economic value of the Claim Group's rights and interests involved determining the percentage reduction from full exclusive native title represented by the Claim Group's non-exclusive native title rights and interests relative to full exclusive native title, and then applying that percentage reduction to full freehold value as a proxy for the economic value of full exclusive native title. The Court held that the value of the Claim Group's non-exclusive native title rights and interests, expressed as a percentage of freehold value, was no more than 50 per cent. The Court rejected that the Claim Group was entitled to compound interest on that sum and awarded simple interest at the Pre-Judgment Interest Rate fixed by the Federal Court of Australia Practice Note CM16. That interest was not part of the total compensation payable for the extinguishment of native title within the meaning of s 51A of the Native Title Act. The Court upheld the award for cultural loss of $1.3 million. The Court held that assessment of cultural loss required determining the spiritual relationship which the Claim Group have with their country and then translating the spiritual hurt caused by the compensable acts into compensation and that the assessment will vary according to the compensable act, the identity of the native title holders, the native title holders' connection with the land or waters by their laws and customs and the effect of the compensable acts on that connection. The Court held that the award to the Claim Group was not manifestly excessive and was not inconsistent with acceptable community standards. +HIGH COURT OF AUSTRALIA 9 December 2015 [2015] HCA 47 Today the High Court delivered judgment in an appeal from the Full Court of the Supreme Court of South Australia, unanimously allowing the appeal in part and dismissing it in part. The Court held that the respondent, who suffered major injuries as a result of a motor vehicle accident, was not contributorily negligent under s 47 of the Civil Liability Act 1936 (SA) ("the Act") for travelling in a car driven by an intoxicated driver, but that she was contributorily negligent under s 49 of the Act for failing to wear a seatbelt. At approximately 2 am on 12 March 2007, the appellant, the respondent and the appellant's friend went for a drive around Port Victoria, South Australia. The respondent was driving, and the appellant and his friend had been drinking alcohol. After about 15 minutes, the respondent stopped the car on the side of the road and went to urinate behind some bushes. When she returned to the car, the appellant was in the driver's seat and insisted that she get in the car, which she did. The car was outside the township and approximately 500 metres from the Port Victoria Hotel, where the group was staying. It was dark, and the respondent gave evidence that she was disoriented and did not know how near she was to the hotel. The appellant drove erratically for several minutes, during which time the respondent did not fasten her seatbelt. The respondent was flung from the car when it collided with a tree, and sustained spinal injuries that rendered her paraplegic. The appellant's blood alcohol level was estimated to have been 0.229 per cent at the time of the accident. At trial, it was held that the appellant's negligence caused the respondent's injuries. The trial judge held that the respondent was not contributorily negligent under s 47 of the Act for relying on the care and skill of an intoxicated person because the exception in s 47(2)(b) of the Act applied such that the respondent could not reasonably be expected to have avoided the risk of travelling with the appellant in the circumstances. Failure to wear a seatbelt constitutes contributory negligence under s 49 of the Act and the trial judge rejected the respondent's contention that the appellant's erratic driving had prevented her from fastening her seatbelt. On appeal, a majority of the Full Court dismissed the appellant's appeal on the s 47(2)(b) issue. The Full Court unanimously allowed the respondent's cross-appeal on the s 49 issue, holding that her failure to fasten her seatbelt was a result of her direct and natural response to the appellant's erratic driving. By grant of special leave, the appellant appealed to the High Court on both issues. The High Court unanimously dismissed the appeal on the s 47(2)(b) issue and held that, given the facts of the case, the respondent could not reasonably be expected to have avoided the risk of travelling with the appellant. The Court allowed the appeal in respect of the s 49 issue, on the basis that there was no reason to interfere with the trial judge's finding of fact that the appellant's driving did not prevent the respondent from fastening her seatbelt. +HIGH COURT OF AUSTRALIA 16 May 2018 EMP144 v THE REPUBLIC OF NAURU [2018] HCA 21 Today the High Court unanimously dismissed an appeal from a decision of the Supreme Court of Nauru that the Refugee Status Review Tribunal ("the Tribunal") had not erred in dismissing the appellant's claim for complementary protection under the Refugees Convention Act 2012 (Nr) ("the Refugees Act"). The appellant was from Pakhu in Nepal where he lived with his wife and child. He was a member and became the vice-president of the local branch of a pro-Royalist political group. He arrived in the Republic of Nauru ("Nauru") in 2013 and applied to the Secretary of the Department of Justice and Border Control of Nauru ("the Secretary") to be recognised as a refugee under the Refugees Act or, alternatively, as a person to whom Nauru owed complementary protection under the Act. He claimed that had a well-founded fear of being persecuted in Nepal by reason of his political views. He alleged that local Maoist groups had previously attacked him and had burned down his home because he did not support the Maoist ideology. The Secretary rejected the application. The Tribunal accepted that the appellant had suffered serious harm amounting to persecution at the hands of particular local Maoist groups because of his political opinion, and that such harm might re-occur in the future if he were to return to the area where he had suffered the harm. The Tribunal affirmed the Secretary's decision, however, on the basis that, because it was localised harm, the appellant could reasonably relocate to elsewhere in Nepal. The Supreme Court dismissed an appeal against the Tribunal's decision. The appellant appealed as of right to the High Court, alleging that the Tribunal had erred in determining his claim for complementary protection by reference to his ability reasonably to relocate within Nepal. He further alleged that the Tribunal had erred in failing to draw his attention to the determinative issue of whether it would be reasonable for him to relocate within Nepal, and in failing to consider articulated reasons as to why he could not reasonably relocate. It was also claimed that the Tribunal misunderstood country information about Nepali citizenship law. The High Court held that, having regard to international jurisprudence, unless the feared persecution emanates from or is condoned or tolerated by State actors (which was not an issue in this case), an applicant's ability reasonably to relocate within his or her country of origin, including the ability safely and legally to travel to the place of relocation, is relevant to whether the applicant is in need of complementary protection. The Court further held that, having regard to the evidence and submissions before the Tribunal, the Tribunal had considered the appellant's objections to internal relocation and the appellant and his legal representatives were aware from the outset of the significance of the issue of whether it would be reasonable for him to relocate within Nepal. It was also held that the Tribunal had not failed to take into account the factors relevant to the appellant's claim for complementary protection, and that the Tribunal had not misunderstood the country information about Nepali citizenship. The Court therefore dismissed the appeal. +HIGH COURT OF AUSTRALIA Manager, Public Information 11 March 2009 MINISTER FOR IMMIGRATION AND CITIZENSHIP v AMIT KUMAR & ANOR The High Court decided today that the Migration Review Tribunal was not required to disclose to Mr Kumar the identity of the person who provided information it had received concerning his application for a spouse visa. Mr Kumar was born in Fiji on 14 September 1982. On 8 May 2004 he married Ms Rachel Sunita Krishna at Lidcombe in New South Wales. Ms Krishna is an Australian citizen. On 10 June 2004 Mr Kumar applied for permanent residence in Australia on the basis that he was the spouse of an Australian citizen. The delegate of the Minister for Immigration and Citizenship was not satisfied that Mr Kumar and Ms Krishna were in a genuine and continuing marriage relationship, or that they had a mutual commitment to a shared life as husband and wife. On 29 September 2004 Mr Kumar’s application for a spouse visa was refused. Mr Kumar applied to the Migration Review Tribunal for a review of the delegate’s decision. At a hearing before the Tribunal on 31 October 2005 Mr Kumar was given a letter which invited him to comment on information received by the Tribunal in confidence stating that his marriage to Ms Krishna was contrived for the sole purpose of migrating to Australia. Mr Kumar was given 28 days within which to provide any comments about the allegation. At the hearing Mr Kumar denied the allegation but did not provide any further evidence or comments within that 28 day period. In its reasons dated 3 February 2006 the Tribunal affirmed the delegate’s decision. Having regard to financial aspects of the relationship, the nature of the household, social aspects of the relationship and the nature of the commitment between Mr Kumar and Ms Krishna, the Tribunal was not satisfied their relationship was genuine. The Tribunal was also persuaded by the information supplied to it in confidence, which it described as “credible and significant adverse information” that Mr Kumar and Ms Krishna were not in a genuine and continuing spousal relationship. Mr Kumar’s application for review of the Tribunal’s decision was dismissed by a Federal Magistrate. However three judges of the Full Court of the Federal Court unanimously upheld his appeal from the Magistrate’s decision. They did so on the basis that the Tribunal had failed to disclose to him the identity of the informant and the full nature of the information provided by the informant. They said that this was required by section 359A of the Migration Act. The Minister appealed against the Full Court’s decision to the High Court. Section 359A requires the Tribunal to give to an applicant “particulars of any information that the Tribunal considers would be the reason, or a part of the reason, for affirming the decision … under review”. Sub-section (4) of section 359A states that the requirement to give particulars does not apply to “non-disclosable information”. Non-disclosable information is defined to include information whose disclosure would found an action for breach of confidence. In a unanimous decision the High Court held the Tribunal had complied with the requirement of section 359A when it alerted Mr Kumar to the advice it had received that his marriage had been contrived for the sole purpose of migrating to Australia. The High Court held that the identity of the person who provided the information fell within the definition of “non-disclosable information”. +HIGH COURT OF AUSTRALIA 9 May 2018 [2018] HCA 18 Today the High Court unanimously allowed an appeal from the Court of Appeal of the Supreme Court of Queensland. The appellant was convicted of a number of sexual offences, including rape, against the same complainant. It was the defence case that the complainant had become disinhibited by alcohol and that she had engaged in consensual sexual conduct, including sexual intercourse with the appellant. The complainant's mother ("Ms M") gave evidence of the terms of her daughter's complaint made in a telephone call after the incident. At the trial, in 2014, in evidence in chief, Ms M stated "[the complainant] phoned me to tell me that she had been raped". In cross-examination, Ms M agreed that at the committal hearing, which took place in 2007, Ms M said that the complainant told her "I think I've been raped" and "I had some wine and I felt funny and I don't remember every – anything after a certain time". Ms M accepted that her evidence in 2007 was the best recollection she was able to give to the Court of what the complainant had said. She also accepted that her memory in 2007 was better than her memory at the trial in 2014. The jury was instructed that the use that it could make of Ms M's 2007 account was in assessing the credibility and reliability of Ms M's evidence at the trial, but that the 2007 account was not evidence that the complainant said those things to Ms M. The appellant appealed against his convictions to the Court of Appeal of the Supreme Court of Queensland. The Court of Appeal found that the trial judge had misdirected the jury: Ms M had adopted the 2007 account and it was open to the jury to assess the credibility and reliability of the complainant's evidence against the 2007 account. Despite this misdirection, the Court of Appeal dismissed the appeal under the proviso to the common form criminal appeal provision, because it determined that no substantial miscarriage of justice had actually occurred. Their Honours took this course notwithstanding that the prosecutor had conceded on the hearing of the appeal that "if the appellant's argument was accepted, it could not be submitted that there had been no substantial miscarriage of justice". Also, the Court of Appeal had not put the appellant on notice that notwithstanding the prosecutor's concession, it was disposed to dismiss the appeal under the proviso. By grant of special leave, the appellant appealed to the High Court on the ground that the Court of Appeal erred in dismissing his appeal under the proviso without notice. The respondent filed a notice of contention, submitting that the Court of Appeal erred in finding that the jury had been misdirected. The High Court unanimously rejected the respondent's contention. The Court found that the Court of Appeal was in error in failing to give the appellant an opportunity to be heard on the question of dismissal under the proviso. The High Court considered for itself whether notwithstanding the misdirection no substantial miscarriage of justice actually occurred. The Court held that Ms M's 2007 account had the capacity, if accepted, to affect the jury's assessment of the credibility and reliability of the complainant's account. In the circumstances, the High Court held it was not open to find that no substantial miscarriage of justice actually occurred. The appellant's convictions and sentences were quashed, and a new trial ordered. +HIGH COURT OF AUSTRALIA 21 March 2018 IN THE MATTER OF QUESTIONS REFERRED TO THE COURT OF DISPUTED RETURNS PURSUANT TO SECTION 376 OF THE COMMONWEALTH ELECTORAL ACT 1918 (CTH) [2018] HCA 10 Today the High Court sitting as the Court of Disputed Returns published reasons for the answers it gave on 13 February 2018 to questions reserved for its consideration by Nettle J pursuant to s 18 of the Judiciary Act 1903 (Cth). Those answers were to the effect that: the vacancy in the representation of South Australia in the Senate for the place for which Ms Skye Kakoschke-Moore was returned on 4 August 2016 should be filled by a special count; the fact that Ms Kakoschke-Moore renounced her British citizenship with effect from 6 December 2017 did not render her capable of being chosen to fill that vacancy; and Mr Timothy Storer should not be excluded from the special count. Nettle J had previously held Ms Kakoschke-Moore incapable of being chosen or of sitting, after the Senate referred to this Court the question whether, by reason of s 44(i) of the Constitution, there was a vacancy in the representation of South Australia in the Senate for the place for which Ms Kakoschke-Moore was returned. Ms Kakoschke-Moore was a British citizen when she nominated, as a nominee of the Nick Xenophon Team ("NXT"), as a candidate to be elected as a senator for South Australia at the general election held on 2 July 2016. In the group nomination form lodged on behalf of NXT, Ms Kakoschke-Moore was listed as the third of four in the order of candidates, before Mr Storer. Under the Commonwealth Electoral Act 1918 (Cth) ("the Act"), the order in which the candidates are listed determines the order in which preferences for "above the line" votes are distributed. Ms Kakoschke-Moore was returned as elected. In November 2017, Ms Kakoschke-Moore resigned her position as senator after receiving confirmation that she was a British citizen. With effect from 6 December 2017, Ms Kakoschke-Moore renounced her British citizenship. Ms Kakoschke-Moore contended that the vacancy in the representation of South Australia in the Senate for the place for which she had been returned should be filled by this Court declaring her to be elected. Alternatively, Ms Kakoschke-Moore submitted that if a special count were to be conducted: (i) she should not be excluded, as she was no longer a British citizen; and (ii) Mr Storer should be excluded, as he had ceased to be a member of NXT, and the voters' true legal intent was that Ms Kakoschke-Moore be replaced by someone of the same political party. The High Court unanimously held that because Ms Kakoschke-Moore was incapable of being chosen at the election held on 2 July 2016 she was incapable of being chosen by the special count, the purpose of which is to complete that electoral process. It was held that a special count is part of the electoral process; it is not a separate, new electoral process by which a new choice is to be made. The Court further held that the voters who cast their votes above the line for NXT at the 2 July 2016 election must be taken to have intended that their votes should, if sufficient, elect Mr Storer. Nothing in the Constitution or the Act requires that a person qualified to be elected and duly elected must remain affiliated with the party that endorsed him or her before the completion of the election. It was therefore necessary for Mr Storer to be included in the special count in order to ensure that it achieved the true legal intent of the voters. +HIGH COURT OF AUSTRALIA 20 June 2018 PAUL IAN LANE v THE QUEEN [2018] HCA 28 Today the High Court unanimously allowed an appeal from the Court of Criminal Appeal of the Supreme Court of New South Wales. The appellant was charged with murder. The offence was alleged to have occurred during an altercation between the appellant and the deceased. CCTV footage of the incident showed the deceased falling to the ground on two occasions. On the first occasion, the deceased retreated towards a roadway, with the appellant in pursuit, before falling backwards and striking his head. After rising to his feet, the deceased could be seen to fall to the road a second time. After the second fall, the deceased lost consciousness, and died in hospital nine days later. The jury acquitted the appellant of murder, but found him guilty of manslaughter. The appellant appealed against his conviction to the Court of Criminal Appeal of the Supreme Court of New South Wales. The Court of Criminal Appeal held that the trial judge had erred in failing to direct the jury that it must be unanimous in its deliberations as to the factual basis on which it might convict the appellant of manslaughter; that being the first fall, the second fall, or both falls. However, the Court of Criminal Appeal, by majority, dismissed the appeal under the proviso to s 6(1) of the Criminal Appeal Act 1912 (NSW), on the basis that no substantial miscarriage of justice had actually occurred. The majority held that the evidence was not capable of supporting a finding beyond reasonable doubt that a deliberate act of the appellant caused the first fall. The absence of a specific unanimity direction did not prevent the jury from considering the appellant's guilt on the basis that his deliberate act caused the second fall and, having done so, it was not open to the jury to have had a reasonable doubt as to his being guilty of manslaughter by that unlawful and dangerous act. By grant of special leave, the appellant appealed to the High Court on the ground that the majority of the Court of Criminal Appeal erred in applying the proviso. The High Court accepted this contention. The Court noted that the case was left to the jury on the basis that it was open to it to convict the appellant by pooling individual jurors' conclusions on issues in respect of which the jury was required to be unanimous. Irrespective of whether an appellate court might conclude that the evidence in respect of the first fall was incapable of supporting a conviction, it was distinctly possible that some jurors may have convicted on the basis of the first fall alone. To dismiss the appeal, as the majority of the Court of Criminal Appeal did, was to disregard the requirement of a unanimous verdict on the part of the jury and to substitute trial by appeal court for trial by jury. The Court held that such an error is apt to deny the application of the proviso because it means that it cannot be said that no substantial miscarriage of justice has actually occurred. Accordingly, the Court ordered that the appellant's appeal be allowed, his conviction be quashed, and a new trial be had. +HIGH COURT OF AUSTRALIA Public Information Officer 1 October, 2003 AUSTRALIAN COMMUNICATION EXCHANGE LIMITED v DEPUTY COMMISSIONER OF TAXATION The High Court of Australia today held that the superannuation guarantee charge paid by an employer of casual workers under a specific award was required to make superannuation deductions from wages payable for work performed during ordinary working hours. Australian Communication Exchange operated a national telephone relay service for people with hearing or speech impairments. Relay officers acted as a medium, converting typed messages to voice and vice versa. The company provided the service 24 hours a day under an agreement with the federal government. It operated call centres in Melbourne and Brisbane, but the appeal related to workers at the Brisbane call centre only. The workers, all casual, were paid according to Queensland’s Clerical Employees Award. The company paid the employer superannuation guarantee charge (then three per cent of pay for the period in question between June 1, 1995, to June 30, 1998) based on rates for ordinary hours, while the Australian Taxation Office claimed the contribution was also payable on hours worked in excess of the 38-hour standard week. Under the relevant legislation, the Superannuation Guarantee Charge Act 1992 and the Superannuation Guarantee (Administration) Act 1992, employers’ obligations with respect to deductions depended upon the terms used in and the language of the award. The outcome of the appeal essentially depended therefore upon the construction to be placed on the award. In the Federal Court, Justice John Dowsett found in favour of the company. The ATO appealed to the Full Court of the Federal Court, which adopted a compromise between the parties’ positions by unanimously holding that the contribution was owed on the ordinary hourly rate paid for overtime but not on the overtime loading. The company appealed to the High Court and the ATO cross-appealed. The High Court, by a 4-3 majority, allowed Australian Communication Exchange’s appeal, reinstating Justice Dowsett’s decision, and dismissed the cross-appeal. The majority held that the contribution should be calculated on ordinary time earnings only and not on any component of overtime pay. +HIGH COURT OF AUSTRALIA 8 March 2005 RAYMOND AKHTAR ALI v THE QUEEN Mr Ali’s defence counsel’s conduct at his trial was not such as to result in a miscarriage of justice, the High Court of Australia held today. Mr Ali, 51, was convicted of murdering his daughter just after she was born to 20-year-old Amanda Leanne Blackwell who lived with the Ali family at Logan Village between Brisbane and the Gold Coast. Mr Ali was also convicted of improperly interfering with a corpse, Ms Blackwell was convicted of manslaughter and both were convicted of concealing the birth of a child. Ms Blackwell gave birth beside a tank stand in September 1998 and the dismembered corpse of the baby, with her reproductive organs removed, was found on an adjoining property. DNA evidence showed the child was Mr Ali’s. Ms Blackwell gave conflicting accounts to police but at the trial she said Mr Ali was the father, he was present at the birth, he took the baby away and she never saw her again, and Mr Ali told her he had disposed of the body. Mr Ali was a butcher who slaughtered goats and removed their sexual organs and sold halal meat to other Muslims. He allegedly prostituted Ms Blackwell and demanded she become engaged to a relative. He denied all involvement and did not give evidence at his trial. Queensland Supreme Court Justice Margaret Wilson sentenced Mr Ali to life imprisonment. Mr Ali twice unsuccessfully appealed to the Queensland Court of Appeal, first on the grounds of alleged unreasonableness of the verdicts and errors in Justice Wilson’s summing up, and then later on a claim of a miscarriage of justice because of the incompetence of his counsel. Special leave to appeal to the High Court was granted in relation to the second appeal. Mr Ali listed 27 instances where he claimed his counsel failed to object to evidence or otherwise acted incompetently. The Court held that these claims were either not made out, or any harm has been cured by Justice Wilson’s directions. Other evidence either had strong probative value so was unlikely to have been rejected by Justice Wilson or was innocuous. There may also have been tactical reasons for not objecting to evidence that might be technically inadmissible. Mr Ali complained about some of his counsel’s cross-examination of Ms Blackwell and other witnesses, but again the Court found any errors were minor. The Court held that the case against Mr Ali was strong and any suggested defects in counsel’s performance were not so significant that he lost the chance of acquittal. No miscarriage of justice was shown. The Court unanimously dismissed his appeal. +HIGH COURT OF AUSTRALIA 3 August 2004 The High Court of Australia has cleared the way for a bus driver injured at work to take action against his employer in the District Court of Western Australia. Mr Kuligowski, now 49, worked for Perth bus company Metrobus. In March 1994 he twisted his left ankle at work and suffered soft tissue injuries. In December 1994 another work accident exacerbated the condition of his ankle. Mr Kuligowski claimed that in April 1995, while shopping, his left ankle gave way and he twisted his left knee. A doctor certified he was unfit for work and Metrobus began paying workers’ compensation, but a year later Metrobus lodged with the Conciliation and Review Directorate an application under the Workers’ Compensation and Rehabilitation Act disputing Mr Kuligowski’s entitlement. A review officer found Metrobus had established a basis to dispute liability to pay compensation but declined to suspend payments pending a further hearing. Another review officer who conducted the later hearing was not satisfied that Mr Kuligowski was incapacitated for work and ordered the weekly payments cease. An appeal to the Compensation Magistrates Court was dismissed. In June 1998 a District Court deputy registrar granted Mr Kuligowski leave to institute proceedings for damages at common law and dismissed Metrobus’s application to have the proceedings struck out on the ground of issue estoppel. Issue estoppel arises when a tribunal makes a final decision so that matters fundamental to the tribunal’s conclusion cannot be redetermined. In this case, the second review officer’s finding that Mr Kuligowski’s injury had recovered left no scope for finding that any later disability which he suffered was caused by the initial accident. He instituted proceedings and alleged the accident was caused by Metrobus’s negligence, but in October 2000 Commissioner John Ley of the District Court allowed appeals by Metrobus against the deputy registrar’s orders. Mr Kuligowski appealed to the Full Court of the Supreme Court, which held, by majority, that the doctrine of issue estoppel could apply to a quasi-judicial determination of a review officer when the District Court was due to determine a common law action. He then appealed to the High Court. The Court held that a finding that the original injury had recovered was a different issue from whether in April 1995 Mr Kuligowski had ongoing instability in his left ankle. It would not be inconsistent with the second review officer’s findings for the District Court to hold there was ongoing instability in his ankle which caused him to twist and injure his left knee in April 1995. The findings of the review officer would not operate as an issue estoppel. The issues which the officer examined, namely whether Mr Kuligowski’s injuries could be characterised as a disability and whether they incapacitated him, were distinct from issues raised in the District Court, including ongoing susceptibility to injury. The High Court held that the doctrine of issue estoppel does not prevent Mr Kuligowski from pursuing District Court proceedings. It unanimously allowed the appeal and remitted the matter to the District Court for hearing. +HIGH COURT OF AUSTRALIA 14 March 2013 ASSISTANT COMMISSIONER MICHAEL JAMES CONDON v POMPANO PTY LTD & ANOR [2013] HCA 7 Today the High Court unanimously upheld the validity of provisions of the Criminal Organisation Act 2009 (Q) ("the Act") which relate to "criminal intelligence" relied upon in support of applications to the Supreme Court of Queensland to have an organisation declared a "criminal organisation". The High Court held that the provisions were not inconsistent with the institutional integrity of the Supreme Court. On 1 June 2012, the Assistant Commissioner of the Queensland Police Service filed an application in the Supreme Court seeking a declaration that the Finks Motor Cycle Club, Gold Coast Chapter, and Pompano Pty Ltd, said to be a part of that Chapter (together "the respondents"), constituted a criminal organisation under s 10 of the Act ("the substantive application"). In support of the substantive application, which is still pending, information which the Supreme Court had previously declared to be criminal intelligence is relied upon. Information which is criminal intelligence relates to actual or suspected criminal activity, the disclosure of which could reasonably be expected to "prejudice a criminal investigation", "enable the discovery of the existence or identity of a confidential source of information relevant to law enforcement", or "endanger a person's life or physical safety". As required by ss 66 and 70 of the Act, the Supreme Court heard the application to declare that particular information was criminal intelligence without notice to the respondents and in a "special closed hearing". Because the Supreme Court made the declaration sought, it will be required by s 78(1) of the Act to order any part of the hearing of the substantive application in which the criminal intelligence is to be considered a closed hearing. The effect is that the criminal intelligence will be kept secret from the respondents. In the High Court, the respondents contended that those provisions of the Act (among others) were invalid because they denied procedural fairness to a respondent to an application to have an organisation declared a criminal organisation. The parties agreed to submit a special case to the High Court asking whether such provisions were invalid for impairing the institutional integrity of the Supreme Court, thereby infringing Ch III of the Constitution. The High Court upheld the validity of the provisions. The Court held that while the provisions may depart from the usual incidents of procedure and judicial process, the Supreme Court nonetheless retains its capacity to act fairly and impartially. The Court held that the provisions do not impair the essential characteristics of the Supreme Court, or its continued institutional integrity. +HIGH COURT OF AUSTRALIA 14 December 2005 ALAN DAVID DOYLE v AUSTRALIAN SECURITIES AND INVESTMENTS COMMISSION A director on the board of a Western Australian mining company had improperly used his position to gain an advantage for his company and its clients, the High Court of Australia held today. Mr Doyle was a director of Doyle Capital Partners (DCP), engaged as a consultant to Chile Minera NL with respect to potential mining interests in Chile. Chile Minera had three directors: chairman Bernard Roland Mountford, who was in Chile during the relevant period; Mr Satterthwaite; and John David Hopkins, who resigned on 15 November 1996. In September 1996, after losses for 1995-96 of more than $12 million, the board resolved to concentrate its activities in Chile, in particular acquiring 75 per cent of Carrizal Alto Prospect. A resolution approving this strategy (and a name change from InterChrome NL) was passed by a show of hands at an extraordinary general meeting (EGM) on 18 October 1996, but with some strong dissent. Two days earlier, Chile Minera allotted eight million new shares, half to DCP and half to two DCP clients, raising $400,000. It paid DCP $24,000 to bring about the share placement. However, on 17 October the Australian Stock Exchange told Chile Minera that the share placement breached the ASX rule prohibiting a listed company from issuing more than 10 per cent of its capital in any 12-month period without shareholder approval. The ASX was also concerned that the allottees of the eight million shares had voted at the EGM. Trading was suspended on 13 November. ASIC’s precursor, the Australian Securities Commission, won Federal Court orders setting aside the EGM resolution approving the acquisition of Carrizal Alto Prospect. The company was restrained from proceeding with the acquisition until it had shareholder approval in the manner required by the Corporations Law and the ASX rules. On 21 November, Mr Doyle sought the return of the $400,000. He, as alternative for Mr Mountford, and Mr Satterthwaite voted to return the money to the DCP trust account and to have DCP return the $24,000 placement fee. The next day Mr Doyle agreed to be a director. He and Mr Satterthwaite and Mr Mountford (by phone) voted to cancel the share placement. Another EGM in February 1997 replaced the directors. The company took separate proceedings in 1997 in the WA Supreme Court against DCP seeking repayment of the placement money and against the three directors. Both matters were settled with DCP paying the company $250,000, minus the $24,000 placement fee. In the WA Supreme Court in 2002, after a trial of the action brought by ASIC, Justice Roberts-Smith made a declaration that Mr Doyle had made improper use of his position to gain directly an advantage for other persons by putting the allottees’ interests ahead of the company’s, contravening section 232(6) of the Corporations Law. Justice Roberts-Smith ordered Mr Doyle to pay to the Commonwealth $30,000 and prohibited him from managing a corporation for two years. On appeal, the Full Court reduced the prohibition period to six months. Mr Doyle appealed to the High Court, seeking to have the finding of breach of section 232(6) set aside. The Court unanimously dismissed the appeal, upholding the Court of Appeal’s finding that all five elements of section 232(6) were present: Mr Doyle was at the relevant time an officer of a corporation; he used that position; his use of his position was improper; the improper use was for the purpose of gaining an advantage; and the advantage was for himself or someone else. In contravention of section 232(1), he was present when the board considered a matter in which he had a material personal interest and had voted on it. Knowledge of the conflict of interest by all the directors did not in all the circumstances alter the case. +HIGH COURT OF AUSTRALIA 9 November 2016 VAUGHAN RUDD BLANK v COMMISSIONER OF TAXATION [2016] HCA 42 Today the High Court unanimously dismissed an appeal from a decision of the Full Court of the Federal Court of Australia. The High Court held that a lump sum paid to a taxpayer in instalments pursuant to an incentive profit participation agreement after termination of his employment was income according to ordinary concepts, and therefore part of the taxpayer's assessable income under s 6-5 of the Income Tax Assessment Act 1997 (Cth). The taxpayer ("Mr Blank") was employed by various companies within the Glencore Group. Glencore International AG ("GI"), a company within the Glencore Group, operated employee profit participation plans for the benefit of certain employees of GI and its subsidiaries. Mr Blank was selected to participate in such a plan. His participation was governed by various agreements at various stages. Relevantly, under a profit participation agreement executed in 1999 ("PPA 1999"), he was granted claims to a share of GI's profits in the form of a contractual claim and Genussscheine ("GS"), which were profit sharing certificates provided for under Swiss law. Under a related agreement, Mr Blank was issued shares in Glencore Holding AG ("GH"), the ultimate holding company of the Glencore Group. The PPA 1999 was relevantly replaced in 2005 by another agreement, the IPPA 2005. All GS or equivalents issued under previous agreements became "Profit Participation Units" ("PPU") under the IPPA 2005. Under the IPPA 2005, Mr Blank was granted "deferred compensation" such that if he ceased employment with the Glencore Group, and executed a declaration under which he relinquished his claims under his PPU and assigned his shares in GH, he would be entitled to a lump sum paid in instalments. Mr Blank's employment with the Glencore Group was terminated on 31 December 2006. He relinquished his claims under his PPU and assigned his shares in GH in consideration of the payment of USD 160,033,328.25 in 20 quarterly instalments ("the Amount"). Edmonds J of the Federal Court of Australia held that the Amount was income according to ordinary concepts because it was deferred compensation for services rendered by Mr Blank. That decision was upheld by a majority of the Full Court of the Federal Court. By grant of special leave, Mr Blank appealed to the High Court. The High Court unanimously held that the Amount was paid as deferred compensation for the services Mr Blank had performed for the Glencore Group and was therefore income according to ordinary concepts. The Amount was not assessable as a capital gain because it did not represent, as Mr Blank contended, the proceeds of the exploitation of interconnected rights that conferred on him a right to receive, in the future, a proportion of the profit of GI. +HIGH COURT OF AUSTRALIA 9 December 2020 [2020] HCA 45 Today the High Court dismissed an appeal from a judgment of the Court of Appeal of the Supreme Court of the Northern Territory. The appeal concerned the permission, implied by the common law, to enter upon premises and approach a dwelling to engage in lawful purposes ("implied licence") and arose in the context of police activities to address domestic violence. On 1 June 2017, a Domestic Violence Order ("DVO") was issued against the appellant to protect her partner in circumstances where the appellant had consumed alcohol. On 6 April 2018, three police officers visited the unit where the appellant and her partner resided. At the time, police in the Northern Territory were engaged in a wider operation which targeted domestic violence. One of the officers knocked on the flyscreen door of the unit. The officer called the appellant to the door for the purpose of a DVO check. The appellant answered the door in a state of intoxication and was requested to take a breath test. The test was positive for alcohol, suggesting a violation of a condition of the DVO. Regulation 6 of the Domestic and Family Violence Regulations (NT) required a defendant to comply with a reasonable direction by an officer to submit to a breath test but did not authorise entry onto premises for that purpose. Section 126(2A) of the Police Administration Act (NT) authorised entry onto premises, but only if the officer believed on reasonable grounds that a contravention of a DVO had occurred, was occurring or was about to occur. In proceedings brought for breach of the DVO, evidence from the breath test was excluded on the ground that it was obtained unlawfully. The trial judge held that the police did not have the power to attend at the unit to check the appellant's compliance with the DVO under reg 6, nor under s 126(2A) because the officer did not hold the requisite belief. On appeal, the Supreme Court of the Northern Territory agreed with the trial judge that neither provision applied and went on to hold that the police did not have an implied licence for the purpose of investigating whether a breach of the law had occurred. On further appeal, the Court of Appeal found that the police had a dual purpose in entering the premises: to determine whether the DVO was being honoured and to check on the wellbeing of the appellant's partner. The police officers had an implied licence because the latter purpose involved lawful communication with the occupier of the unit and did not involve an interference with the occupier's possession, or injury to the person or property of either occupier. Dismissing the appeal, a majority of the High Court found that the police officers had an implied licence to enter onto the premises. All Justices of the Court accepted that the common law will not imply a licence for police where entry is for the sole purpose of exercising coercive powers. However, the majority found it could not be inferred from the unchallenged evidence that the officer who administered the breath test intended to do so when he entered onto the premises regardless of the appellant's condition when he first observed or spoke to her. The Court of Appeal was correct to hold that the police had a purpose of enquiring about the welfare of the appellant's partner. The purpose of checking on the welfare of the appellant's partner was a sufficient foundation for an implied licence. Once the officer observed the appellant's state of intoxication, he had the requisite belief for the purposes of s 126(2A) and was authorised to remain on the premises and to require a sample of the appellant's breath. +HIGH COURT OF AUSTRALIA 15 December 2010 ROWE & ANOR v ELECTORAL COMMISSIONER & ANOR [2010] HCA 46 On 6 August 2010, the High Court declared, by majority, that certain provisions that amended the Commonwealth Electoral Act 1918 (Cth) were invalid. Today, the Court published the reasons for its decision. The provisions that were declared invalid related to the cut-off dates for consideration of claims for enrolment and for transfer of enrolment as an elector. The provisions prevented the Electoral Commissioner from considering claims for enrolment lodged after 8 pm on the date of the issue of writs for an election for the House of Representatives or the Senate until after the close of polling. They also prevented consideration of claims for transfer of enrolment from one divisional roll to another from 8 pm on the third working day after the date of the issue of the writs until after the close of polling. In separate judgments, Chief Justice French, Justices Gummow and Bell, and Justice Crennan held that these provisions contravened the requirement, contained in ss 7 and 24 of the Constitution, that members of both Houses of the Commonwealth Parliament be "directly chosen by the people". The Chief Justice considered that the adverse legal and practical effect of the challenged provisions upon the exercise of the entitlement to vote was disproportionate to their advancement of the requirement of direct choice by the people. Justices Gummow and Bell, with whom Justice Crennan broadly agreed, held that the provisions operated to achieve a disqualification from the entitlement to vote and that the disqualification was not reasonably appropriate and adapted to serve an end compatible with the maintenance of the system of government prescribed by the Constitution. Justice Crennan held that the democratic right to vote is supported and protected by the Constitution. In separate dissenting judgments, Justices Hayne, Heydon and Kiefel each held that the provisions did not contravene any limitation imposed by the Constitution on the legislative power of the Commonwealth to fix the date and time after which claims for enrolment or transfer of enrolment may not be considered before an election. Their Honours considered that the requirement of direct choice by the people was not infringed by the provisions challenged. +HIGH COURT OF AUSTRALIA 3 February 2017 IN THE MATTER OF QUESTIONS REFERRED TO THE COURT OF DISPUTED RETURNS PURSUANT TO SECTION 376 OF THE COMMONWEALTH ELECTORAL ACT 1918 (CTH) [2017] HCA 4 Today the High Court unanimously held that Rodney Norman Culleton was a person who was convicted and subject to be sentenced for an offence punishable by imprisonment for one year or longer at the time of the 2016 federal election, and therefore was incapable of being chosen as a Senator under s 44(ii) of the Constitution. On 2 March 2016, Mr Culleton was convicted, in his absence, in the Local Court of New South Wales at Armidale of the offence of larceny. As the offence of which Mr Culleton was convicted concerned property of a value less than $5,000, he was liable to imprisonment for a maximum term of two years. Under s 25(1)(a) of the Crimes (Sentencing Procedure) Act 1999 (NSW) ("the CSP Act"), a sentence of imprisonment may not be imposed upon an "absent offender". The Local Court issued a warrant for the arrest of Mr Culleton in order to have him brought to the Court for sentencing. On 16 May 2016, a writ was issued for the election of 12 Senators for the State of Western Australia to serve in the Senate of the Parliament of the Commonwealth. Mr Culleton was nominated as a candidate in a group nomination for Pauline Hanson's One Nation party. Polling day for the election was 2 July 2016. On the return of the writ, Mr Culleton was noted as elected and he has since sat as a Senator. On 8 August 2016, the warrant for the arrest of Mr Culleton was executed, and the Local Court granted an annulment of Mr Culleton's conviction pursuant to s 8 of the Crimes (Appeal and Review) Act 2001 (NSW). On 25 October 2016, the Local Court found Mr Culleton guilty of the offence of larceny on his own plea but, pursuant to s 10(1)(a) of the CSP Act, dismissed the charge without proceeding to conviction. On 8 November 2016, the President of the Senate referred to the High Court, in its capacity as the Court of Disputed Returns, the question whether Mr Culleton's conviction had the effect of disqualifying him from being chosen as a Senator by reason of s 44(ii) of the Constitution. The Senate's reference also asked how the vacancy should be filled if Mr Culleton were found to have been incapable of being chosen as a Senator. The Court held that Mr Culleton was convicted and subject to be sentenced for an offence punishable by imprisonment for one year or longer at the date of the 2016 election, both as a matter of fact and as a matter of law. The subsequent annulment of the conviction had no effect on that state of affairs. The Court held that the resulting vacancy should be filled by a special count of the ballot papers. +HIGH COURT OF AUSTRALIA 10 June 2015 [2015] HCA 20 Today the High Court unanimously allowed an appeal from a decision of the Court of Appeal of the Supreme Court of Victoria. The High Court held that the decision-making process used by the respondent's delegate in exercising power under s 84P(e) of the Domestic Animals Act 1994 (Vic) ("the Act") was contrary to natural justice because a fair-minded observer might reasonably apprehend that a person who took part in the decision-making might not have brought an impartial mind to the decision. The appellant had been convicted in the Ringwood Magistrates' Court of an offence under s 29(4) of the Act, on a charge that her Staffordshire terrier had attacked a person and caused serious injury. Following a hearing before a panel, a delegate of the respondent who had been a member of that panel made a decision under s 84P(e) of the Act that the appellant's dog be destroyed. Another member of the panel, who had participated fully in the panel's decision-making process following the hearing and drafted the reasons for the decision, was an employee of the respondent whose duties involved the regulation of domestic animals under the Act. She had been substantially involved in the prosecution of the charge in the Magistrates' Court. The appellant, unsuccessfully, sought judicial review of the respondent's decision in the Supreme Court of Victoria. The appellant's appeal to the Court of Appeal of the Supreme Court of Victoria was limited to the ground of apprehended bias. The Court of Appeal found that the ground was not made out and dismissed the appeal. By grant of special leave, the appellant appealed to the High Court. The High Court unanimously allowed the appeal. The Court found that a fair-minded observer might reasonably apprehend that the respondent's employee might not have brought an impartial mind to the decision to destroy the appellant's dog, because her role in the Magistrates' Court the proceedings gave her an decision-making process of the panel. The Court found that, although another member of the panel was responsible for making the decision to order the destruction of the dog, there was still an apprehension that the involvement of the respondent's employee in the Magistrates' Court prosecution might affect not only her own decision-making, but also that of the other members of the panel. The Court found that natural justice required that she not participate in making the decision, and that the decision of the respondent's delegate must therefore be quashed. incompatible with her involvement that was interest +HIGH COURT OF AUSTRALIA Manager, Public Information 2 April 2009 R & R FAZZOLARI PTY LTD v PARRAMATTA CITY COUNCIL MAC’S PTY LTD v PARRAMATTA CITY COUNCIL & ANOR Today the High Court decided that Parramatta City Council may not compulsorily acquire land owned by R & R Fazzolari Pty Ltd and Mac’s Pty Ltd without their approval. Parramatta City Council proposed to redevelop a block within the city centre bounded by Smith, Darcy, Church and Macquarie Streets. The redevelopment was to be called “Civic Place” and was to be carried out under a Public Private Partnership between Parramatta City Council and two companies in the Grocon group. The agreement between the Council and Grocon provided, amongst other things, that the Council would acquire certain land within the redevelopment block and would transfer some of the acquired land to the Grocon companies, and in return, the Council would receive substantial financial payments and other benefits from Grocon. Some of the land which the Council had to compulsorily acquire in order for the agreement between it and Grocon to come to fruition and for the redevelopment to proceed belonged to R& R Fazzolari Pty Ltd. Some belonged to Mac’s Pty Ltd. Some parts of the land – in Darcy St and Church St - were actually parts of the public roads owned by the Council under the Roads Act. Although those two streets were vested in the Council, it had to compulsorily acquire them from itself in order for them to be utilised in the redevelopment. That procedure was authorised by section 7B of the Land Acquisition (Just Terms Compensation) Act. In NSW section 188(1) of the Local Government Act provides that if land is being acquired for the purpose of re-sale, then it may not be compulsorily acquired without the approval of the owner. Section 188(2) qualifies this constraint on a Council’s power to compulsorily acquire land – it provides that the owner’s approval is not required if the land is part of, adjoins or lies in the vicinity of other land acquired at the same time under Part 1 of Chapter 8 of the Local Government Act for a purpose other than re-sale. The Council sent proposed acquisition notices to owners of land within the redevelopment block, including Fazzolari and Mac’s. Each of them challenged the proposed acquisitions on the basis that their land was being acquired in order to re-sell it to Grocon. Before the NSW Land and Environment Court they successfully argued the land could not be compulsorily acquired without their consent. However three judges of the NSW Court of Appeal agreed with the Council that the land belonging to Fazzolari and Mac’s was being acquired to implement the Council’s “Civic Place” project and not for the purpose of re-selling it to Grocon. The Court of Appeal held the Council did not need the owners’ consent to compulsorily acquire the land. Fazzolari and Mac’s appealed to the High Court. The High Court considered that the acquisition of the Fazzolari land and the Mac’s land could be characterised as steps along the way in an arrangement which was directed towards the “Civic Place” re-development, but that did not detract from the fact that, under the development agreement, the Council’s purpose in acquiring the specific parcels of land owned by Fazzolari and Mac’s was to re-sell them to Grocon. At the hearing before the High Court the Council argued that, even if the land owned by Fazzolari and Mac’s was being compulsorily acquired for the purpose of re-sale, both parcels of land adjoined other land (ie – Darcy St and part of Church St) acquired under Part 1 of Chapter 8 of the Local Government Act for a purpose other than re-sale. The Council argued that the application of section 188(2) of the Local Government Act meant it was not required to obtain the owners’ approval for the compulsory acquisition. The High Court, however, found the Council was acquiring Darcy St and part of Church St under section 7B of the Just Terms Compensation Act, not under Part 1 of Chapter 8 of the Local Government Act, and therefore section 188(2) did not apply to relieve the Council of the need to obtain the approval of Fazzolari and Mac’s before it compulsorily acquired their land. +HIGH COURT OF AUSTRALIA 10 August 2011 ABLE SEAMAN JOSEPH ANTHONY PETER HASKINS v THE COMMONWEALTH OF AUSTRALIA PAUL NICHOLAS v THE COMMONWEALTH OF AUSTRALIA & ANOR [2011] HCA 28; [2011] HCA 29 Today the High Court held that items 3, 4 and 5 of Schedule 1 to the Military Justice (Interim Measures) Act (No 2) 2009 (Cth) ("the Interim Measures No 2 Act") are valid laws of the Commonwealth Parliament. The High Court also held that the Interim Measures No 2 Act provided lawful authority justifying the detention of Able Seaman Haskins. On 26 August 2009, in Lane v Morrison (2009) 239 CLR 230, the High Court declared that the Australian Military Court ("AMC") established under the Defence Force Discipline Act 1982 (Cth) ("the Discipline Act") was not validly created. In September 2009, in response to the Court's decision in Lane v Morrison, the Commonwealth Parliament enacted the Military Justice (Interim Measures) Act (No 1) 2009 (Cth) ("the Interim Measures No 1 Act") and the Interim Measures No 2 Act. The Interim Measures No 1 Act amended the Discipline Act to restore the system of military disciplinary tribunals in existence before the AMC's creation. The Interim Measures No 2 Act imposed disciplinary sanctions on persons corresponding to punishments which had been invalidly imposed by the AMC in the period between the AMC's establishment and the declaration of invalidity by the High Court. The Interim Measures No 2 Act did not purport to validate any convictions or punishments imposed by the AMC or convict any person of any offence. Able Seaman Haskins enlisted in the Royal Australian Navy on 5 April 2004. On 11 December 2008, he was found guilty by the AMC of 11 counts of misusing a Defence Travel Card. On one count he was sentenced to a severe reprimand and on the others he was sentenced to detention for various periods. He served his sentences at the Defence Force Corrective Establishment between December 2008 and January 2009. From 1 January 2004 until 25 August 2008, Mr Paul Nicholas was a commissioned officer in the Australian Army, holding the rank of Captain. In August 2008 he was convicted by the AMC of two charges of obtaining a financial advantage contrary to s 135.2(1) of the Criminal Code (Cth) and two charges relating to perverting the course of justice. He was sentenced to be reduced in rank to Lieutenant, to dismissal from the defence force and to a severe reprimand. Orders were made that he pay reparation to the Commonwealth. After Lane v Morrison was decided and the Interim Measures Acts enacted, Able Seaman Haskins and Mr Nicholas separately brought proceedings in the High Court's original jurisdiction. Able Seaman Haskins claimed, among other things: (a) a declaration that he was falsely imprisoned by the Commonwealth, its officers, representatives, servants and/or agents, (b) damages, and (c) a declaration that his claims in (a) and (b) were unaffected by the Interim Measures No 2 Act. Mr Nicholas sought a declaration that item 5 of Schedule 1 to the Interim Measures No 2 Act was invalid, together with declarations that the convictions recorded by the AMC against him, the punishments imposed on him and his dismissal from the defence force were invalid and of no effect. In both proceedings, the High Court, by majority, held that the impugned provisions of the Interim Measures No 2 Act did not contravene Ch III of the Constitution and were valid. In the proceedings brought by Able Seaman Haskins, the High Court held that, on its proper construction, the Interim Measures No 2 Act provided lawful authority justifying his detention. The impugned provisions of the Interim Measures No 2 Act were also held not to contravene s 51(xxxi) of the Constitution on the basis that no action for false imprisonment lay against the Commonwealth and that therefore no property of Able Seaman Haskins was acquired. +HIGH COURT OF AUSTRALIA Public Information Officer 17 May 2006 SARAH DAVISON v STATE OF QUEENSLAND VANESSA FAYNE JEAN GIBSON v STATE OF QUEENSLAND STEPHEN GIRARD v STATE OF QUEENSLAND JASON THOMAS ORR v STATE OF QUEENSLAND NATASHA YARRIE v STATE OF QUEENSLAND ALEXANDER JANE ORR v STATE OF QUEENSLAND Six former wards of the State of Queensland will be able to argue for an extension of time to bring action against the State, the High Court of Australia held today. The six wished to commence actions in tort relating to abuse allegedly suffered as children in foster care under the care and supervision of the State. The three-year limitation period had long since expired. Each sought an order extending the limitation period, pursuant to section 31(2) of the Limitation of Actions Act, which provides that where a material fact of a decisive character relating to the right of action was not within the means of knowledge of an applicant until a date during the last year of the limitation period, the court may extend the limitation period so that it expires one year after that date. Each appellant argues that a material fact of a decisive character became known to them on 18 June 2003 when Brisbane’s Courier-Mail newspaper ran a story detailing widespread abuse of foster children. However, due to the time elapsed, each failed to comply with rules for commencing proceedings set out in the Personal Injuries Proceedings Act. Section 43 of this Act provides for actions to be brought, despite non-compliance, if a court is satisfied of an urgent need to start a proceeding. An order giving leave to do so may be made on conditions the court considers necessary or appropriate. Each appellant applied to the Queensland Supreme Court for leave under section 43 during the one-year period referred to in section 31(2). Justice James Douglas refused the applications, holding there was no urgent need to start proceedings. Appeals were dismissed. The appellants filed further applications, which Justice Catherine Holmes granted. Proceedings were commenced and applications to extend time under section 31(2) filed, but these applications have not been heard because an appeal from the State was allowed. The Court of Appeal held that an applicant seeking a section 43 order after the expiry of the ordinary limitation period must demonstrate a reasonably arguable case to grant an extension which it said the appellants had failed to do. The appellants then appealed to the High Court. The Court unanimously allowed the appeals. It held that the function of section 43 of the Personal Injuries Act is to avoid harsh results flowing from failure to comply with the preconditions contained in that Act for commencing proceedings in cases where the limitation period, or the one-year period in section 31(2) of the Limitations Act, was about to expire. The Court held that to require of those seeking leave to initiate proceedings under section 43 to show a reasonably arguable case to grant an extension, was to read words into section 43 in an impermissible manner. The risk of injustice for defendants from leave granted too freely under section 43 is reduced by a court’s capacity to impose appropriate conditions. The Court held there was some prospect of establishing at a hearing for a section 31(2) application that a material fact of a decisive character relating to the right of action was not within the appellants’ means of knowledge until 18 June 2003. The appellants claimed to be unaware of widespread abuse or of the State’s possible knowledge of abuse or of criminal investigations relating to such abuse, and each believed they were the only victim of abuse in their foster family. They did not realise before the Courier-Mail story that the State might be at fault. The Court held that Justice Holmes was right to say there was a strong basis for an argument that a material fact of a decisive character became known to the former foster children on 18 June 2003. +HIGH COURT OF AUSTRALIA Public Information Officer 7 August 2008 MARJORIE HEATHER OSLAND v SECRETARY TO THE DEPARTMENT OF JUSTICE The Victorian Court of Appeal, in considering whether public interest overrode legal professional privilege attaching to advice that Mrs Osland should not be pardoned for murder, should have inspected the documents in question, the High Court of Australia held today. In 1996, Mrs Osland was convicted of murdering her husband, Frank Osland, in 1991, allegedly after years of violence. She was sentenced to 14-and-a-half years' imprisonment, with a non-parole period of nine-and-a-half years. The High Court dismissed her appeal against conviction and sentence in 1998. Mrs Osland then submitted a petition for mercy to the Victorian Attorney- General, seeking a pardon from the Governor. On 6 September 2001, Attorney-General Rob Hulls announced that the Governor had refused the petition. In a press release Mr Hulls noted that legal advice had been received from three senior counsel (including Susan Crennan QC, now a Justice of the High Court, who did not hear this appeal). Mrs Osland sought access under the Victorian Freedom of Information Act (FOI Act) to various pieces of advice related to her request for a pardon. The Department of Justice refused access to the documents, both initially and upon internal review. It said the documents were exempt from disclosure by reason of section 30 (relating to internal working documents) and section 32 (relating to legal professional privilege). That decision was overturned by the President of the Victorian Civil and Administrative Tribunal, Justice Stuart Morris. He found that the documents fell within section 32, but that the “public interest override” provided by section 50(4) of the FOI Act nevertheless required access be given to all the documents in dispute. The Secretary successfully appealed to the Court of Appeal. In that appeal, Mrs Osland maintained her action only in relation to the advice from the three senior counsel, known as Document 9. The Court of Appeal held that Justice Morris correctly decided that legal professional privilege had not been waived in respect of Document 9 but erred in dealing with the public interest override. He had inspected the documents but the Court of Appeal did not. Mrs Osland appealed to the High Court. She argued that Mr Hulls had waived the legal professional privilege of Document 9 because his press release disclosed the substance and gist of the advice and the conclusions reached in it. Mrs Osland argued that the Court of Appeal erred in concluding that there was no basis for applying the public interest override under section 50(4) without having inspected the documents for itself. The Court, by a 5-1 majority, allowed the appeal. It held that legal professional privilege had not been waived in relation to Document 9 by Mr Hulls's press release, but that the Court of Appeal should have examined the documents in question before deciding that, in the circumstances of the case, there was no basis for the application of section 50(4). It remitted the matter to the Court of Appeal for further hearing to enable it to inspect the documents to consider whether public interest overrode legal professional privilege. +HIGH COURT OF AUSTRALIA 14 February 2018 AUSTRALIAN BUILDING AND CONSTRUCTION COMMISSIONER v CONSTRUCTION, FORESTRY, MINING AND ENERGY UNION & ANOR [2018] HCA 3 Today, the High Court, by majority, allowed an appeal from a decision of the Full Court of the Federal Court of Australia, regarding whether a judge has power to order either that a union not that union official indemnify a union official against a pecuniary penalty ("non-indemnification order") or that a union official not seek or accept indemnity or contribution from the union in respect of a pecuniary penalty imposed on the union official ("personal payment order"). imposed on The second respondent ("Myles") was an officer of the first respondent ("the CFMEU"). In 2013, Myles and some 20 associates carried out a blockade that disrupted a concrete pour at a construction site with the intention of coercing compliance with Myles' requests that there be a CFMEU delegate on the site. At the end of the blockade, Myles called out to the General Superintendent of the site words to the effect of: "I'll be back tomorrow to stop the concrete pour" and: "You won't pour again until you put a delegate on". The following day, Myles returned to the site and told the General Superintendent: "[I]f you don't want to put a delegate on then we will have [a war]." Proceedings were instituted against the respondents in the Federal Court. The respondents admitted to contravening s 348 of the Fair Work Act 2009 (Cth) ("the FWA"), and the hearing before the primary judge proceeded in relation to penalty only. The primary judge ordered that, under s 546 of the FWA, Myles pay pecuniary penalties of $18,000 and the CFMEU pay pecuniary penalties of $60,000. The primary judge also made a non-indemnification order, pursuant to s 545(1) of the FWA. The respondents appealed to the Full Court, which held that the primary judge had no power under s 545(1) to make the non-indemnification order since such an order would "add to the penal outcome" authorised by s 546 and there was no clear and express power to do so. By grant of special leave, the appellant appealed to the High Court on the question whether s 545(1) of the FWA or s 23 of the Federal Court of Australia Act 1976 (Cth) empowers a judge to make a non-indemnification order. Following the hearing, the appellant was granted leave to amend the notice of appeal to raise the question whether s 546 of the FWA empowers a judge to make a non-indemnification or personal payment order. The Court unanimously held that neither s 545(1) nor s 23 empowers a judge to make a non-indemnification order: such an order is penal and is beyond the scope of those sections. The Court also held unanimously that s 546 could not support a non-indemnification order, with the plurality holding that an order directed at someone other than the party subject to the pecuniary penalty is not authorised by that section. By majority, however, the Court held that a personal payment order could be made under s 546; the power to make such an order was within the implied power to make such orders as are reasonably required for, or legally ancillary to, the accomplishment of the effect that the pecuniary penalty is calculated to achieve. The appeal was allowed and the matter remitted to the Full Court for the imposition of penalties. +HIGH COURT OF AUSTRALIA Public Information Officer 26 March 2008 AK v THE STATE OF WESTERN AUSTRALIA The High Court of Australia today ordered a retrial for a boy charged with indecently dealing with a child after the trial judge sitting without a jury gave insufficient reasons for his decision to convict. In February 2002, AK, then aged 13, allegedly dealt indecently with his 15-year-old female cousin as they slept on a double mattress with his brother, R, aged 14, and her sister, aged 16. AK’s family had visited the cousin’s family in Geraldton, Western Australia, when the two families decided to travel to Tardun and spend the night in a caravan. During the night AK allegedly touched his cousin’s breasts and vagina and placed her hand on his penis. In March 2003, the cousin became pregnant and subsequently had an abortion. She told other family members and the police that AK was the father. In 2004, AK was charged in the WA Children’s Court with three counts of indecent dealing with a child aged between 13 and 16. These charges related to the alleged incident of February 2002. He was also charged with rape and indecent assault of the cousin in early 2003. In cross-examination of the cousin, an issue of identification was raised and the suggestion was put to her that R could have been the one who touched her in 2002. She was adamant it was AK, not R, but could not say how in the darkness she knew. After a trial in July 2005 before Judge Henry Wisbey sitting alone, AK was convicted of the counts of indecent dealing but acquitted of the other two charges. He was sentenced to a six-month intensive youth supervision order and is subject to seven-and-a-half years of reporting conditions. AK, now aged 19, appealed against his convictions for indecent dealing. Section 120(2) of the WA Criminal Procedure Act provides that the judgment of the judge in a trial by judge alone must include the principles of law they have applied and the findings of fact on which they have relied. The Court of Appeal unanimously held that Judge Wisbey’s reasons for judgment were inadequate because they did not identify and address the issue which arose at trial as the identity of the offender. However, by majority, the Court dismissed the appeal, applying the proviso that no substantial miscarriage of justice had occurred. AK appealed to the High Court. The High Court agreed that Judge Wisbey erred in law in failing to give adequate reasons for his decision to convict AK, only stating his satisfaction beyond reasonable doubt that all elements of indecent dealing had been established. The Court held that Judge Wisbey was obliged to say why and how he resolved the question of identification in favour of the prosecution. However, the Court, by a 3-2 majority, held that the Court of Appeal erred in applying the proviso. The majority held that the complete failure to meet the mandatory requirements of section 120(2) of the Act with respect to the central issue of identification was a substantial miscarriage of justice. The High Court ordered that AK’s convictions be quashed and directed that a new trial be held. +HIGH COURT OF AUSTRALIA 5 August 2015 AUSNET TRANSMISSION GROUP PTY LTD v THE COMMISSIONER OF TAXATION OF THE COMMONWEALTH OF AUSTRALIA [2015] HCA 25 Today the High Court, by majority, dismissed an appeal from a decision of a Full Court of the Federal Court of Australia, which held that certain charges paid by the appellant were outgoings of a capital nature and therefore not tax deductible. The appellant purchased the assets of a State-owned electricity transmission business, Power Net Victoria ("PNV"). The assets included an electricity transmission licence held by PNV which had been issued pursuant to Pt 12 of the Electricity Industry Act 1993 (Vic) ("the transmission licence"). The Victorian Governor in Council, by Order made pursuant to s 163AA(1) of the Act, had imposed specified charges on the holder of the transmission licence. When the appellant became the holder of the transmission licence, it became liable to pay the charges by force of the Act. In addition, the relevant asset sale agreement expressly required that the appellant pay the charges to the State of Victoria and refrain from contesting their validity. Separately, it required the appellant to pay a specified "total purchase price" to PNV for its assets. Section 8-1(2)(a) of the Income Tax Assessment Act 1997 (Cth) provides that a loss or outgoing of capital, or of a capital nature, is not tax deductible under that section. Relevantly, the appellant claimed that the payments of the specified charges were deductible from its assessable income. The respondent, the Commissioner of Taxation of the Commonwealth of Australia, disallowed the deductions claimed and the appellant's subsequent objection. At first instance, the Federal Court affirmed the Commissioner's position. On appeal to a Full Court, the majority held that the specified charges were not deductible because they were of a capital nature and dismissed the appeal. By grant of special leave, the appellant appealed to the High Court. By majority, the Court held that, from a practical and business point of view, the appellant assumed the liability to make the payments in order to acquire the transmission licence and the other assets of PNV. The payments of the charges were outgoings of a capital nature and were therefore not tax deductible. +HIGH COURT OF AUSTRALIA 12 December 2012 [2012] HCA 59 Today a majority of the High Court allowed an appeal by Mr Michel Baini against the decision of the Court of Appeal of the Supreme Court of Victoria to uphold his convictions for blackmail of Mr Hassan Rifat. The High Court remitted the matter to the Court of Appeal for its further consideration. The appellant and another accused were charged with numerous counts of blackmail. All but one of the charges alleged blackmail of Mr Rifat. The other count alleged blackmail of Mr Nicholas Srour. In the course of the trial, an application was made to sever the trial of the count relating to Mr Srour but the trial judge refused that application. The jury found the appellant guilty of some of the counts, including the count relating to Mr Srour. The appellant appealed to the Court of Appeal, which held that the trial judge should have severed the trial of the count relating to Mr Srour. The Court of Appeal concluded that this error resulted in a substantial miscarriage of justice in the trial of the count relating to Mr Srour but not in the trial of the counts relating to Mr Rifat. It therefore ordered that there be a new trial of the count relating to Mr Srour but upheld the appellant's convictions for blackmail of Mr Rifat. By special leave, the appellant appealed to the High Court in relation to his convictions for blackmail of Mr Rifat. A majority of the High Court allowed the appeal. The majority held that, because all the counts were tried together, the jury heard prejudicial evidence from Mr Srour which would not have been admissible in a separate trial of the counts alleging blackmail of Mr Rifat. Unless this error could have had no effect on the jury's guilty verdicts, the error in refusing to sever the counts resulted in a substantial miscarriage of justice in the trial of the counts about Mr Rifat. The Court of Appeal not having considered whether the error could have had no effect on the outcome of the trial having regard to the full record of the trial, and the High Court not having available to it the full record, the majority allowed the appeal and remitted the matter to the Court of Appeal for its further consideration. +HIGH COURT OF AUSTRALIA 10 November 2021 JONG HAN PARK v THE QUEEN [2021] HCA 37 Today the High Court unanimously dismissed an appeal from a decision of the Court of Criminal Appeal of the Supreme Court of New South Wales. The appeal concerned the correct interpretation of s 22 of the Crimes (Sentencing Procedure) Act 1999 (NSW) ("the Sentencing Act"), which prescribes how certain courts are required to take into account an offender's guilty plea in passing sentence. Section 22(1) relevantly provided that if an offender has pleaded guilty the court may impose a lesser penalty "than it would otherwise have imposed". The appellant pleaded guilty and was sentenced in the District Court of New South Wales to an aggregate sentence of imprisonment of 11 years, with a non-parole period of eight years, for multiple offences including the offence of taking a conveyance without the consent of the owner contrary to s 154A(1)(a) of the Crimes Act 1900 (NSW) ("the offence"). The maximum penalty for the offence was five years' imprisonment but, because it was dealt with as a "related offence" in accordance with s 165 of the Criminal Procedure Act 1986 (NSW), the District Court's sentencing power was limited to the maximum penalty of two years' imprisonment that the Local Court could have imposed for the offence ("the jurisdictional limit"). As the sentencing judge imposed an aggregate sentence, his Honour was required by s 53A of the Sentencing Act to indicate the sentence that would have been imposed had separate sentences been imposed. The sentencing judge indicated a sentence of two years' imprisonment for the offence and noted that a 25% discount had been applied to the sentence that would otherwise have been imposed to reflect the utility of the appellant's early plea of guilty. Therefore, but for the appellant's guilty plea, the sentencing judge's indicative sentence would have been two years and eight months' imprisonment, being a sentence in excess of the jurisdictional limit. The appellant argued that this was not a sentence the sentencing judge "would otherwise have imposed" within the meaning of s 22(1) of the Sentencing Act because, by reason of the jurisdictional limit, it was not a sentence that the court could impose. The High Court rejected the appellant's construction of s 22(1) of the Sentencing Act and held that the sentence that the court "would otherwise have imposed" is the appropriate sentence determined in accordance with the Sentencing Act and without regard to any jurisdictional limit. Any relevant jurisdictional limit should be applied by the sentencing judge after the judge has determined the appropriate sentence for an offence in accordance with s 21A of the Sentencing Act. A jurisdictional limit relates to the sentencing court, not to the task of identifying and synthesising the relevant factors that are weighed to determine the appropriate sentence. +HIGH COURT OF AUSTRALIA 23 June 2010 Manager, Public Information SAEED v MINISTER FOR IMMIGRATION AND CITIZENSHIP [2010] HCA 23 The High Court today held that, in the case of offshore visa applicants, s 51A of the Migration Act 1958 (Cth) does not exclude the Minister's obligation, imposed by law, to afford an applicant the opportunity to comment on information before the Minister that is adverse to the applicant, prior to a decision on the application being made. The appellant is a Pakistani citizen who applied for a Skilled – Independent Visa (Subclass 175). One of the eligibility requirements for the visa was employment in a skilled occupation for at least 12 months in the 24-month period before lodging the application. The appellant had provided documents to demonstrate that she had been employed as a cook in Pakistan. Australian immigration officers in Pakistan had investigated the appellant's claims. They discovered that no employee records were kept at the restaurant and were told that no woman had ever worked in the kitchen ("the adverse information"). On the basis of the adverse information, on 16 July 2008, a delegate of the Minister for Immigration and Citizenship refused the application because she could not be satisfied that the appellant had met the eligibility criteria. The appellant was not made aware of the adverse information until it was outlined in a letter refusing her application. Subdivision AB of Part 2 of the Migration Act is entitled "Code of procedure for dealing fairly, efficiently and quickly with visa applications". It sets out procedures for dealing with visa applications after they are lodged and before a decision is made. Section 57(2), which is in subdivision AB, provides that certain "relevant information" received by the Minister must be provided to a visa applicant for comment. Section 57(3) provides, however, that this obligation does not arise unless the visa can be granted when the applicant is in the Australian migration zone; and unless the Migration Act provides, under Parts 5 or 7, for review of a decision to refuse to grant the visa. In the appellant's case, neither condition in s 57(3) was met. The particular visa could only be granted while the appellant was offshore and no avenue of review was provided for under Parts 5 or 7 of the Act. The Minister was therefore not obliged to follow the procedure in s 57 to give the appellant an opportunity to comment on the adverse information. The appellant sought judicial review of the delegate's decision in the Federal Magistrates Court of Australia. She claimed that she had been denied procedural fairness because she was not given an opportunity to comment on the adverse information prior to the delegate's decision. Because the procedural obligation under s 57 did not apply, she relied upon the natural justice hearing rule at common law. In a case such as this, that rule requires that a person be given an opportunity to comment on adverse information that is credible, relevant and significant to the decision to be made. On 2 December 2008, Federal Magistrate Emmett dismissed the application on the basis that the common law natural justice hearing rule was excluded by the operation of s 51A of the Migration Act. Section 51A provides that subdivision AB is "taken to be an exhaustive statement of the requirements of the natural justice hearing rule in relation to the matters it deals with". The Full Court of the Federal Court of Australia dismissed the appellant's appeal. On 3 November 2009, the appellant was granted special leave to appeal to the High Court. The appellant submitted that s 51A did not exclude the application of the natural justice hearing rule in this case. The Court noted that the statement in s 51A is qualified by the words "in relation to the matters it deals with". It held that the "matter" with which s 57 deals is the provision of relevant information for comment to the persons to whom the information is required to be provided. The terms of s 57(3) limit the beneficiaries of the statutory procedure to onshore visa applicants. Therefore, the provision of information to offshore visa applicants, such as the appellant, is not a "matter" dealt with by the section. The Court concluded that the application of the natural justice hearing rule was not, in the appellant's case, excluded by s 51A. The Minister was obliged to provide the appellant with an opportunity to respond to the adverse material. This conclusion made it unnecessary for the Court to consider the appellant's alternative argument that s 51A was constitutionally invalid. The Court allowed the appeal, quashed the decision of the Minister's delegate and ordered that the Minister consider and determine the appellant's application in accordance with the Court's reasons. +HIGH COURT OF AUSTRALIA 7 April 2020 [2020] HCA 12 Today, the High Court granted special leave to appeal against a decision of the Court of Appeal of the Supreme Court of Victoria and unanimously allowed the appeal. The High Court found that the jury, acting rationally on the whole of the evidence, ought to have entertained a doubt as to the applicant's guilt with respect to each of the offences for which he was convicted, and ordered that the convictions be quashed and that verdicts of acquittal be entered in their place. On 11 December 2018, following a trial by jury in the County Court of Victoria, the applicant, who was Archbishop of Melbourne at the time of the alleged offending, was convicted of one charge of sexual penetration of a child under 16 years and four charges of committing an act of indecency with or in the presence of a child under the age of 16 years. This was the second trial of these charges, the jury at the first trial having been unable to agree on its verdicts. The prosecution case, as it was left to the jury, alleged that the offending occurred on two separate occasions, the first on 15 or 22 December 1996 and the second on 23 February 1997. The incidents were alleged to have occurred in and near the priests' sacristy at St Patrick's Cathedral in East Melbourne, following the celebration of Sunday solemn Mass. The victims of the alleged offending were two Cathedral choirboys aged 13 years at the time of the events. The applicant sought leave to appeal against his convictions before the Court of Appeal. On 21 August 2019 the Court of Appeal granted leave on a single ground, which contended that the verdicts were unreasonable or could not be supported by the evidence, and dismissed the appeal. The Court of Appeal viewed video-recordings of a number of witnesses' testimony, including that of the complainant. The majority, Ferguson CJ and Maxwell P, assessed the complainant to be a compelling witness. Their Honours went on to consider the evidence of a number of "opportunity witnesses", who had described the movements of the applicant and others following the conclusion of Sunday solemn Mass in a way that was inconsistent with the complainant's account. Their Honours found that no witness could say with certainty that these routines and practices were never departed from and concluded that the jury had not been compelled to entertain a reasonable doubt as to the applicant's guilt. Weinberg JA dissented, concluding that, by reason of the unchallenged evidence of the opportunity witnesses, the jury, acting rationally on the whole of the evidence, ought to have had a reasonable doubt. On 17 September 2019, the applicant applied to the High Court for special leave to appeal from the Court of Appeal's decision on two grounds. On 13 November 2019, Gordon and Edelman JJ referred the application for special leave to a Full Court of the High Court for argument as on an appeal. The application was heard by the High Court on 11 and 12 March 2020. The High Court considered that, while the Court of Appeal majority assessed the evidence of the opportunity witnesses as leaving open the possibility that the complainant's account was correct, their Honours' analysis failed to engage with the question of whether there remained a reasonable possibility that the offending had not taken place, such that there ought to have been a reasonable doubt as to the applicant's guilt. The unchallenged evidence of the opportunity witnesses was inconsistent with the complainant's account, and described: (i) the applicant's practice of greeting congregants on or near the Cathedral steps after Sunday solemn Mass; (ii) the established and historical Catholic church practice that required that the applicant, as an archbishop, always be accompanied when robed in the Cathedral; and (iii) the continuous traffic in and out of the priests' sacristy for ten to 15 minutes after the conclusion of the procession that ended Sunday solemn Mass. The Court held that, on the assumption that the jury had assessed the complainant's evidence as thoroughly credible and reliable, the evidence of the opportunity witnesses nonetheless required the jury, acting rationally, to have entertained a reasonable doubt as to the applicant's guilt in relation to the offences involved in both alleged incidents. With respect to each of the applicant's convictions, there was, consistently with the words the Court used in Chidiac v The Queen (1991) 171 CLR 432 at 444 and M v The Queen (1994) 181 CLR 487 at 494, "a significant possibility that an innocent person has been convicted because the evidence did not establish guilt to the requisite standard of proof". +HIGH COURT OF AUSTRALIA 12 February 2004 ROBERT JOHN PUTLAND v THE QUEEN The High Court of Australia today upheld the imposition by the Northern Territory Supreme Court of an aggregate sentence for multiple federal offences. Mr Putland pleaded guilty in 2001 to two offences of defrauding the Commonwealth contrary to the Crimes Act and four offences against the Bankruptcy Act. The offences arose from the failure of his Alice Springs trucking business and involved two counts each of tax fraud, disposing of property with intent to defraud his creditors after being declared bankrupt in 1992, and carrying on a business until 1996 as an undischarged bankrupt. The Supreme Court imposed a single aggregate sentence of four years’ jail and ordered that Mr Putland be released after serving 12 months upon entering into a three-year good behaviour bond. He appealed unsuccessfully to the NT Court of Criminal Appeal on various grounds. The sole ground of appeal pursued in the High Court was that the judge did not have power to impose an aggregate term of imprisonment. Aggregate sentences may be imposed for multiple offences arising from a course of conduct. Section 52 of the NT Sentencing Act allowed courts to impose one term of imprisonment for a number of offences and section 68 of the federal Judiciary Act provided that State or Territory laws regarding arrest, custody, trial and sentencing shall apply to people charged with Commonwealth offences. Hence offenders in different parts of Australia may experience different treatment due to differences in laws between States and Territories. Mr Putland argued that section 68 of the Judiciary Act did not give the Supreme Court power under section 52 of the Sentencing Act to impose one term of imprisonment in respect of all offences. He argued that Part 1B, in particular section 4K, of the Crimes Act covered the field regarding sentencing of federal offenders in higher courts, with aggregate sentencing only allowed for summary offences, not for indictable offences. The High Court held that the Supreme Court had power, pursuant to section 68 of the Judiciary Act, to impose aggregate sentences. The Court, by a 4-1 majority, dismissed Mr Putland’s appeal. +HIGH COURT OF AUSTRALIA Public Information Officer 4 February, 2003 PLAINTIFF S157/2002 v THE COMMONWEALTH OF AUSTRALIA and RE MINISTER FOR IMMIGRATION AND MULTICULTURAL AND INDIGENOUS AFFAIRS & REFUGEE REVIEW TRIBUNAL; EX PARTE APPLICANTS S134/2002 Plaintiff S157, intending to commence legal proceedings to challenge administrative decisions concerning his protection visa application, faced two provisions included in the Migration Act in 2001: section 474, a privative clause preventing certain administrative decisions from being made the subject of judicial review; and section 486A, imposing a 35-day limit on applications for such review. He brought an action to challenge the validity of those sections and to contest their application to his case. The merits of his proposed challenge to the administrative decisions were not in issue before the High Court of Australia. Only the validity and effect of sections 474 and 486A were in question. The High Court’s jurisdiction to grant relief against Commonwealth officers under section 75(v) of the Constitution cannot be ousted by act of Parliament. The Commonwealth accepted this and did not contend that section 474 of the Migration had totally excluded judicial review, pursuant to section 75(v), of decisions which section 474 applied. The parliamentary debates, to which the Commonwealth referred the Court, reveal that parliament did not intend the section to have that effect. The Court held that sections 474 and 486A are valid but apply to what the Act defines as privative clause decisions. It held that decisions affected by certain kinds of error – jurisdictional error – are not privative clause decisions as defined. Accordingly, the Court held that neither section would apply to the proceedings which S157 would initiate. In the second case, relating to Applicants S134, the High Court, by majority, rejected challenges to decisions refusing protection visas for a woman and her five children claiming Afghan nationality. They arrived by boat in 2001, apparently unaware their husband and father had arrived in 1999 and held a three-year temporary protection visa. The family claimed the Refugee Review Tribunal had erred by not taking into account the man’s circumstances and that the Immigration Minister should have exercised his power to substitute a more favourable decision. The Court held that rejection of their applications was not marked by jurisdictional error, illegality or impropriety and the Minister was not bound to exercise his power. Note: Since October 2001, the Migration Act has provided that courts cannot name plaintiffs seeking protection visas. +HIGH COURT OF AUSTRALIA 13 December 2018 COMMISSIONER OF TAXATION FOR THE COMMONWEALTH OF AUSTRALIA v TOMARAS & ORS [2018] HCA 62 Today the High Court answered a question of law that had been stated for the opinion of the Full Court of the Family Court of Australia, and otherwise dismissed an appeal from the decision of that Court. Part VIII of the Family Law Act 1975 (Cth) ("the Act") provides for spousal maintenance and the division of the property of the parties to a marriage. In proceedings with respect to the property of the parties to a marriage, s 79 provides that a court may make an order altering the interests of the parties to the marriage in the property, but that the court shall not make such an order unless it is satisfied that in all the circumstances it is just and equitable to make the order. Part VIIIAA of the Act allows a court to make an order under s 79 that is directed to, or alters, the rights, liabilities or property interests of a third party. Section 90AE(1)(b) allows the court to make an order directed to a creditor of one party to the marriage to substitute the other party to the marriage in relation to the debt owed to that creditor. Section 90AE(3) lists a number of preconditions to the making of such an order. The first respondent ("the wife") and the second respondent ("the husband") married in 1992 and separated in 2009. During their marriage, the appellant ("the Commissioner") issued various assessments requiring the wife to pay, among other things, income tax, the Medicare levy, penalties and the general interest charge ("GIC"). The wife failed to pay the amounts assessed and did not lodge objections to the assessments. On 12 November 2009, the Commissioner obtained default judgment against the wife. On 5 November 2013, the husband was declared bankrupt. In December 2013, the wife commenced proceedings against the husband in the Federal Circuit Court of Australia. The wife sought an order pursuant to s 90AE(1)(b) that, in respect of the wife's indebtedness to the Commissioner for certain taxation related liabilities plus GIC, the husband be substituted for the wife as the debtor and the husband be solely liable to the Commissioner for the debt. The Federal Circuit Court stated a question of law for the opinion of the Full Court of the Family Court. The question asked was whether s 90AE(1)-(2) of the Act granted the court power to make the order sought by the wife. The Full Court answered the question in the affirmative, subject to the proviso that s 90AE(1) confers power only to make an order that the Commissioner be directed to substitute the husband for the wife in relation to the debt owed by the wife to the Commissioner. By grant of special leave, the Commissioner appealed to the High Court. The Court unanimously held that, in relation to a debt owed to the Commonwealth by a party to a marriage, s 90AE(1) confers power on the court to make an order that the Commissioner be directed to substitute the husband for the wife in relation to that debt. A majority of the Court further held that it was otherwise inappropriate to answer the question of law stated (i) without it being found, or agreed, that, within the meaning of s 90AE(3), the making of the order is reasonably necessary, or reasonably appropriate and adapted, to effect a division of property between the parties to the marriage, and it is not foreseeable at the time that the order is made that to make the order would result in the debt not being paid in full; and (ii) without the court being satisfied that, in all the circumstances, it is just and equitable to make the order. +HIGH COURT OF AUSTRALIA 15 December 2005 This appeal concerned the application of the “proviso”, by which a criminal appeal may be dismissed, even though a trial judge has made an error of law, where there has been no substantial miscarriage of justice. Mr Weiss was convicted in the Victorian Supreme Court of beating Helen Elizabeth Gray to death on 24 November 1994. He was charged with her murder six years later. His former de facto wife, Jean Horstead, testified that on the night of the murder Mr Weiss confessed he had killed Ms Gray, a friend of theirs. Ms Horstead originally provided him with a false alibi, but after she split up with him and moved to America she contacted police to tell the truth. After the murder, Mr Weiss began a relationship with a 14-year-old girl named Renée. In November 2000, in an interview with police, he admitted visiting Ms Grey, quarrelling with her and striking her with her son’s cricket bat. Mr Weiss admitted stripping the rubber grip from the bat handle and throwing it from the car on his way home. He told the trial these confessions were false. The Victorian Court of Appeal unanimously held that evidence of Renée’s age should not have been admitted as it was not relevant, could not be used to bolster the credit of Ms Horstead, and any probative value was outweighed by its prejudicial quality. The Court however dismissed the appeal, holding that it should apply the proviso. The Court held that a distinction should be drawn between whether, without the wrongly admitted evidence, the jury in the case would inevitably have convicted Mr Weiss, and whether, without that evidence, any reasonable jury, properly instructed, would inevitably have convicted him. On the “this jury” test, the Court concluded that conviction was inevitable. On the “any reasonable jury” test, conviction was not necessarily inevitable because another jury might have a different view of Ms Horstead’s evidence or the reliability of Mr Weiss’s confession. The Court held that the “this jury” test was the relevant test and dismissed the appeal. Mr Weiss was granted special leave to appeal to the High Court. The High Court unanimously allowed the appeal. It held that consideration of the proviso is not to be undertaken by attempting to predict what a specific jury would do. Rather, in applying the proviso, the task is to decide whether a substantial miscarriage of justice has actually occurred. The appeal court must make its own assessment of the evidence and the strength of the case for or against the accused. The Court remitted the matter to the Court of Appeal for further consideration in accordance with the reasoning of the High Court. +HIGH COURT OF AUSTRALIA 6 August 2004 AHMED ALI AL-KATEB v PHILIPPA GODWIN, DEPUTY SECRETARY, DEPARTMENT OF IMMIGRATION AND MULTICULTURAL AND INDIGENOUS AFFAIRS; JULIE HELEN KEENAN, ACTING DIRECTOR OF THE UNAUTHORISED ARRIVALS SECTION; AND MINISTER FOR IMMIGRATION AND MULTICULTURAL AND INDIGENOUS AFFAIRS MINISTER FOR IMMIGRATION AND MULTICULTURAL AND INDIGENOUS AFFAIRS v ABBAS MOHAMMAD HASAN AL KHAFAJI The High Court of Australia today held that unsuccessful asylum seekers who could not be removed to another country, despite their wish to leave Australia, could continue to be held in immigration detention indefinitely. Both cases involve asylum seekers who had asked to be returned to the Middle East. The Federal Government has been unable to reach arrangements with other countries to take them, but argued that the Migration Act required they be held in immigration detention in the mean- time, regardless of whether such an arrangement could ever be reached. Mr Al-Kateb, 28, is a stateless Palestinian who was born and lived most of his life in Kuwait. He arrived in Australia in December 2000 and applied for a protection visa. His application was dismissed by the department, the Refugee Review Tribunal, the Federal Court and the Full Court of the Federal Court. Mr Al-Kateb then told the department he wished to leave Australia and be sent to either Kuwait or Gaza. In February 2003 he initiated action in the Federal Court claiming he was being unlawfully detained. The Court held that although there was no likelihood of removal in the reasonably foreseeable future, he was not unlawfully detained. Justice John Mansfield ordered Mr Al-Kateb’s release in April 2003, pending an appeal. Mr Al Khafaji, 31, is an Iraqi national who fled with his family to Syria in 1980. He arrived in Australia in January 2000. His application for a protection visa was unsuccessful and he asked to return to Syria. The Federal Court ordered his release from detention as there was no real prospect of removal in the reasonably foreseeable future. The Minister appealed to the Full Court. In August 2003, on application from the federal Attorney-General, the High Court ordered that both cases be removed into this Court to be heard together. The High Court, by a 4-3 majority, held that Migration Act provisions requiring the continued detention of unlawful non-citizens are not invalid. Under the Act, detention must continue until an unlawful non-citizen is either removed from Australia, deported or granted a visa. The majority held that as a purpose of detention was the eventual removal of unlawful non-citizens the detention was not prohibited by the Constitution. Accordingly, the Court dismissed Mr Al- Kateb’s appeal and allowed the Minister’s appeal in Mr Al Khafaji’s case. +HIGH COURT OF AUSTRALIA 9 February 2022 CONSTRUCTION, FORESTRY, MARITIME, MINING AND ENERGY UNION & ANOR v PERSONNEL CONTRACTING PTY LTD [2022] HCA 1 Today, the High Court allowed an appeal from a judgment of the Full Court of the Federal Court of Australia. The appeal concerned whether a labourer was engaged by a labour-hire company as an employee or an independent contractor. The second appellant ("Mr McCourt") was a 22-year-old British backpacker with limited work experience who had travelled to Australia on a working holiday visa. He sought work from the respondent, a labour-hire company (trading as "Construct"). He was offered a role and signed an Administrative Services Agreement ("ASA") with Construct. The ASA described Mr McCourt as a "self-employed contractor". Construct assigned Mr McCourt to work on two construction sites run by Construct's client, Hanssen Pty Ltd ("Hanssen"). Mr McCourt performed basic labouring tasks under the supervision and direction of supervisors employed by Hanssen. The relationship between Construct and Hanssen was governed by a Labour Hire Agreement. There was no contract between Mr McCourt and Hanssen. The first appellant and Mr McCourt commenced proceedings against Construct in the Federal Court of Australia seeking compensation and penalties pursuant to the Fair Work Act 2009 (Cth) ("the Act"). The crucial issue was whether Mr McCourt was an employee of Construct for the purposes of the Act. The primary judge held that Mr McCourt was an independent contractor, and an appeal to the Full Court was dismissed. Both courts applied a "multifactorial" approach, by reference to the terms of the ASA and the work practices imposed by each of Construct and Hanssen. The High Court, by majority, held that Mr McCourt was Construct's employee. The majority held that where parties have comprehensively committed the terms of their relationship to a written contract, the efficacy of which is not challenged on the basis that it is a sham or is otherwise ineffective under general law or statute, the characterisation of that relationship as one of employment or otherwise must proceed by reference to the rights and obligations of the parties under that contract. These rights and obligations are to be ascertained in accordance with established principles of contractual interpretation. Absent a suggestion that the contract has been varied, or that there has been conduct giving rise to an estoppel or waiver, a wide-ranging review of the parties' subsequent conduct is unnecessary and inappropriate. Under the ASA, Construct had the right to determine for whom Mr McCourt would work, and Mr McCourt promised Construct that he would co-operate in all respects in the supply of his labour to Hanssen. In return, Mr McCourt was entitled to be paid by Construct for the work he performed. This right of control, and the ability to supply a compliant workforce, was the key asset of Construct's business as a labour-hire agency. These rights and obligations constituted a relationship between Construct and Mr McCourt of employer and employee. That the parties chose the label "contractor" to describe Mr McCourt did not change the character of that relationship. +HIGH COURT OF AUSTRALIA 7 February 2018 FALZON v MINISTER FOR IMMIGRATION AND BORDER PROTECTION [2018] HCA 2 Today the High Court unanimously upheld the validity of s 501(3A) of the Migration Act 1958 (Cth) ("the Act"). The High Court held that s 501(3A) does not authorise or require the detention of a non- citizen and does not confer upon the Minister for Immigration and Border Protection ("the Minister") the judicial power of the Commonwealth. The plaintiff is a national of Malta who has resided in Australia since 1956. He arrived in Australia with his family at the age of three. He did not at any time obtain Australian citizenship. Until 10 March 2016, he held an Absorbed Person Visa and a Class BF Transitional (Permanent) Visa. In 2008, the plaintiff was convicted of trafficking a large commercial quantity of cannabis. He was sentenced to 11 years' imprisonment with a non-parole period of eight years. Four days before the expiration of the plaintiff's non-parole period, a delegate of the Minister cancelled his Absorbed Person Visa pursuant to s 501(3A) of the Act, with the effect that the Minister was taken to have decided to cancel the plaintiff's other visa. The plaintiff was taken into immigration detention upon being released from criminal custody. The plaintiff sought revocation of the decision to cancel his visa. The Assistant Minister decided not to revoke the decision. Section 501(3A) relevantly provides that the Minister must cancel a visa that has been granted to a person if the Minister is satisfied that the person has a substantial criminal record and is currently serving a sentence of imprisonment on a full-time basis for an offence against a law of the Commonwealth, a State or a Territory. A cancellation decision under s 501(3A) has the immediate effect that the person's status is changed from lawful non-citizen to unlawful non-citizen. An unlawful non-citizen is liable to immigration detention. The plaintiff commenced proceedings in the original jurisdiction of the High Court, challenging the validity of s 501(3A) on the ground that it purports to confer the judicial power of the Commonwealth on the Minister and thereby infringes Ch III of the Constitution. The plaintiff argued that, in its legal operation and effect, s 501(3A) further punished him for the offences he committed and that is its purpose. The plaintiff relied on the principle that the power to adjudge guilt of, or determine punishment for, breach of the law is a power exclusive to the Ch III judiciary. The High Court unanimously dismissed the plaintiff's application. The Court held that s 501(3A) did not authorise or require the detention of the plaintiff. Section 501(3A) required that the Minister cancel the plaintiff's visa on account of his criminal history and his imprisonment. The purpose of the statutory scheme of which s 501(3A) forms a part is to regulate the presence in Australia of non- citizens, in the national interest, and to remove or deport from Australia non-citizens who, in the view of Parliament, should not be permitted to remain in Australia. The Court held that cancellation of a visa for that purpose does not involve any determination or punishment of criminal guilt and does not involve the exercise of judicial power. +HIGH COURT OF AUSTRALIA 2 December 2015 FAIR WORK OMBUDSMAN v QUEST SOUTH PERTH HOLDINGS PTY LTD & ORS [2015] HCA 45 Today the High Court unanimously allowed an appeal from the Full Court of the Federal Court of Australia. The High Court held that s 357(1) of the Fair Work Act 2009 (Cth) prohibits an employer from misrepresenting to an employee that the employee performs work as an independent contractor under a contract for services with a third party. Quest South Perth Holdings Pty Ltd ("Quest") operated a business of providing serviced apartments and employed Ms Margaret Best and Ms Carol Roden as housekeepers. Contracting Solutions Pty Ltd ("Contracting Solutions") operated a labour hire business. Quest and Contracting Solutions purported to enter into a triangular contracting arrangement under which Contracting Solutions purported to engage Ms Best and Ms Roden as independent contractors under contracts for services, and then purported to provide the services of Ms Best and Ms Roden as housekeepers to Quest under a labour hire agreement. Quest represented to Ms Best and Ms Roden that they were performing housekeeping work as independent contractors of Contracting Solutions, despite the fact that they continued to perform that work for Quest under implied contracts of employment. Section 357(1) provided that: "A person (the employer) that employs, or proposes to employ, an individual must not represent to the individual that the contract of employment under which the individual is, or would be, employed by the employer is a contract for services under which the individual performs, or would perform, work as an independent contractor." The Fair Work Ombudsman commenced a proceeding in the Federal Court claiming, amongst other relief, pecuniary penalty orders against Quest for contraventions of s 357(1). The Federal Court ordered at first instance that the proceeding be dismissed so far as it related to that claim, and an appeal from that order was dismissed by the Full Court. The Full Court held that s 357(1) would only be contravened by an employer's representation to an employee which mischaracterised the contract of employment that existed between the employee and the employer as a contract for services made between the employee and the employer, not between the employee and a third party. By grant of special leave, the Fair Work Ombudsman appealed to the High Court. The High Court unanimously allowed the appeal, holding that s 357(1) prohibited the misrepresentation of an employment contract as a contract for services with a third party. The Court declared that Quest contravened s 357(1) by representing to Ms Best and Ms Roden that the contracts of employment under which they were employed by Quest were contracts for services under which they performed work as independent contractors. +HIGH COURT OF AUSTRALIA 9 March 2005 VIRGINIO VIGOLO v WANDA MARY BOSTIN AND LEOPOLDO VIGOLO (as executors of the will of Lino Vigolo deceased), WANDA MARY BOSTIN, LEOPOLDO VIGOLO, NANCY Virginio Vigolo had no claim on the estate of his father, Lino Vigolo, as proper provision had already been made for his maintenance, support, education and advancement in life, so the Western Australian Supreme Court was correct to reject his application to have his father’s will amended in his favour, the High Court of Australia held today. Virginio Vigolo, 47, was the eldest of five children who had spent most of his working life farming with his parents near Albany in WA. When Lino Vigolo died in 1997 his estate, worth $1.9 million, was divided among the other four children with no provision made for Virginio, or for his widow Rosario. Virginio left school at 16 to work on the farm while also taking other jobs. When he had saved $10,000 he planned to buy his own farm but his father suggested they buy another farm together because when he died it would all be his son’s. Virginio Vigolo said his father made many similar promises over the years, telling him his wages were low because he would eventually inherit. They bought the second farm in 1978 with Mr Vigolo and his parents as equal partners operating both farms. They later bought a third farm and the income generated by all three enabled them to buy as investments a produce market and a service station. In 1988 the partnership became a company with Virginio Vigolo and his parents as equal shareholders. About this time, he and his wife bought a hairdressing business, which his wife ran, then a farm of their own. Virginio and his father fell out over this separate accumulation of assets. In 1993 the company was dissolved by a deed of settlement that divided up the assets based on market valuation. Rosario Vigolo gave her share of the original farm to her son and his wife and the couple then bought out the father’s share at commercial rates. After settlement was completed Lino Vigolo drew up a new will. He left Virginio out of the will because he believed he had made adequate provision for him during his lifetime and because Mr Vigolo had received his mother’s share of the farm. A year after his father’s death in 1997, Virginio and his wife sold the farm for $1.68 million. Virginio Vigolo applied to the WA Supreme Court for an order under section 6 of the Inheritance (Family and Dependants Provision) Act which provides that a court may use its discretion to modify a will to make provision out of the deceased’s estate for the proper maintenance, support, education or advancement in life of family members listed in section 7. Justice Carmel McLure rejected the application. She found that Mr Vigolo was adequately compensated for his contributions to the farming business and compared with his siblings he was given significant financial advantage by his parents. The Full Court of the Supreme Court dismissed an appeal. Mr Vigolo appealed to the High Court. The Court unanimously dismissed the appeal. It held that Mr Vigolo was not entitled to an order under section 6 of the Act to vary his father’s will in his favour because he was in a strong financial position and the 1993 deed of settlement rendered the promise to inherit no longer relevant. The Court held that he was not left without proper provision for his maintenance, support, education or advancement in life. +HIGH COURT OF AUSTRALIA 14 April 2021 DVO16 v MINISTER FOR IMMIGRATION AND BORDER PROTECTION; BNB17 v MINISTER FOR IMMIGRATION AND BORDER PROTECTION [2021] HCA 12 Today, the High Court unanimously dismissed these two appeals. DVO16 v Minister for Immigration and Border Protection was an appeal from the Full Court of the Federal Court of Australia, and BNB17 v Minister for Immigration and Border Protection was an appeal from a single judge of the Federal Court of Australia exercising appellate jurisdiction. Each appeal concerned the effect on a review by the Immigration Assessment Authority ("the Authority") under Pt 7AA of the Migration Act 1958 (Cth) of translation errors in questions asked and responses given at a protection visa interview between an applicant, assisted by an interpreter, and a delegate of the Minister. In each case, the High Court was asked to decide whether translation errors resulted in the Authority's decision being affected by jurisdictional error. The High Court observed that whether and if so in what circumstances mistranslation might result in jurisdictional error turns necessarily on whether and if so in what circumstances mistranslation might result in non-compliance with a condition expressed in or implied into the statute which authorises the decision-making process and sets the limits of the decision-making authority. In relation to a decision of the Authority under Pt 7AA, the High Court explained that there are two ways in which translation errors might result in non-compliance with express or implied statutory conditions. The first involves breach of the implied condition of reasonableness. In circumstances where the Authority has knowledge of translation errors, the Authority might breach the condition of reasonableness implied into its powers to get and consider new information if it fails to exercise those powers to interview the referred applicant and then consider the applicant's testimony as correctly translated. The Authority might equally breach the reasonableness condition implied into its duty to review the referred decision by considering the review material if it makes findings adverse to the applicant with knowledge of the translation errors, without having exercised its powers to get and consider new information which might address those errors. The second way in which translation errors might result in non-compliance with Pt 7AA is through non-compliance with the Authority's overriding duty to "review" the referred decision. Mistranslation has the potential to result in the Authority failing to understand and therefore to consider the substance of an applicant's claim. Mistranslation in that way has the potential to result in the Authority failing to discharge the core element of its overriding duty, namely to assess the claims for protection in fact made by an applicant against the criteria for the grant of the visa in determining for itself whether it is satisfied that the criteria for the grant of the visa have been met. In each of the cases under appeal, the High Court held that the translation errors did not result in jurisdictional error. In DVO16, the translation errors could not have borne on the question of reasonableness because the Authority was not aware of them. In BNB17, although the Authority was aware of three examples of errors, the High Court held that those errors were not so grave or extensive as to render the course adopted by the Authority unreasonable. As to the second way in which invalidity could occur, in each case the High Court held that the translation errors did not result in the Authority failing to understand and therefore to consider the appellant's claims. +HIGH COURT OF AUSTRALIA 11 May 2016 ROBERT BADENACH & ANOR v ROGER WAYNE CALVERT [2016] HCA 18 Today the High Court unanimously allowed an appeal from a decision of the Full Court of the Supreme Court of Tasmania. The Court held that, in the circumstances of the present case, a solicitor did not owe a duty of care to a beneficiary under a will to advise the testator of the options available to the testator to avoid exposing his estate to a claim under the Testator's Family Maintenance Act 1912 (Tas) ("the TFM Act"). The first appellant ("the solicitor") is a legal practitioner and was a partner of the second appellant, a law firm. The solicitor received instructions from Mr Jeffrey Doddridge ("the client") to prepare his will, by which the entirety of his estate was to pass to the respondent (who was the son of the client's long-term de facto partner). The client's principal assets were two properties which he owned as a tenant in common in equal shares with the respondent. The client died later that year having executed a will drawn by the solicitor in accordance with his instructions. Following the client's death, his daughter from a previous marriage (and for whom he made no provision in his will) brought proceedings under the TFM Act and was successful in obtaining a court order that provision be made for her out of the client's estate. As a result, the client's estate was substantially depleted. The respondent brought proceedings against the appellants in which the respondent claimed that the solicitor had been negligent in failing to advise the client of the possibility that his daughter might make a claim under the TFM Act and the options available to him to reduce or extinguish his estate so as to avoid such a claim. At first instance, the Supreme Court of Tasmania held that the solicitor owed the client a duty of care to enquire as to the existence of any family members and, upon discovering the existence of the client's daughter, to advise the client of the risk to his estate of a successful claim under the TFM Act. However, the primary judge was not satisfied that the solicitor's advice about a possible claim under the TFM Act would have triggered an enquiry by the client about how to protect the respondent's position. The Full Court allowed the respondent's appeal. In their Honours' view, the solicitor's duty to the client extended to advising of the possible steps the client could take so as to avoid exposing his estate to a TFM claim, even if the client did not make any enquiry about those steps. The Full Court also held that the solicitor owed a duty of care to the respondent that was co-extensive with that owed to the client. By grant of special leave, the appellants appealed to the High Court. The High Court held that the duty to the respondent recognised by the Full Court did not arise. The interests of the client were not coincident with the interests of the respondent and as such the solicitor could not owe any duty to the respondent that was co-extensive with the solicitor's duty to the client. +HIGH COURT OF AUSTRALIA 30 July 2008 Public Information Officer NORTHERN TERRITORY OF AUSTRALIA AND DIRECTOR OF FISHERIES (NT) v ARNHEM LAND ABORIGINAL LAND TRUST, NORTHERN LAND COUNCIL, NATIVE TITLE HOLDERS*, NORTHERN TERRITORY SEAFOOD COUNCIL AND COMMONWEALTH OF AUSTRALIA Traditional Aboriginal owners had the right to exclude fishermen and others from tidal waters within Blue Mud Bay in north-east Arnhem Land, the High Court of Australia held today. In 1980, pursuant to the Commonwealth Aboriginal Land Rights (Northern Territory) Act (the Land Rights Act), the Governor-General granted estates in fee simple of two areas to the Arnhem Land Aboriginal Land Trust, which held title to the land and exercised its powers as owner for the benefit of the Aboriginals concerned. The Land Trust had to do so in accordance with the direction of the Northern Land Council (NLC). The mainland grant covered 90,000 square kilometres between the mouth of the East Alligator River in Van Diemen Gulf in the west and the mouth of the Roper River in the Limmen Bight in the east, excluding Cobourg Peninsula. The islands grant covered all islands, except Groote Eylandt, adjacent to the mainland grant. Each grant extended to the low-water mark. The Yolngu people are the traditional owners of parts of Arnhem Land, including Blue Mud Bay, which lies in the mainland grant. Section 70(1) of the Land Rights Act prohibited outsiders from entering or remaining on Aboriginal land. Under the NT Aboriginal Land Act, a land council may grant permission to enter and remain on Aboriginal land. In the Federal Court of Australia in 2005, the Land Trust, the NLC and the native title holders sought declarations of their rights under the grants, along with orders restraining the NT and the Director of Fisheries from issuing fishing licences for areas subject to native title claims, including areas within the grants. Commercial fishing and fishing by non-Aboriginals in Blue Mud Bay had been observed over many years. The NLC argued that the Director could not authorise fishing in waters overlying Aboriginal land. The Director argued that tidal waters were not part of Aboriginal land and fishing licences validly authorised fishing there. Justice Bradley Selway dismissed the proceeding. He held that grants under the Land Rights Act gave the Land Trust an estate in fee simple to the low-water mark but did not confer a right to exclude fishing in tidal waters. Before orders giving effect to these conclusions could be made, Justice Selway died, and Justice John Mansfield finalised the orders. The Land Trust, the NLC and native title holders appealed to the Full Court of the Federal Court against Justice Mansfield’s orders. The Full Court made declarations that the NT Fisheries Act did not apply to the grant areas, did not confer on the Director of Fisheries a power under that Act to grant licences to authorise or permit fishermen to enter and take seafood from those areas, and was invalid and of no effect regarding the grant areas. The NT and the Director appealed to the High Court. The High Court, by a 5-2 majority, allowed the appeal in part, to the extent that the declarations were too broad and needed to be reframed. The Land Trust, the NLC and native title holders accepted that none of the three declarations should stand and that a more limited declaration should be made, to the effect that the Fisheries Act did not by itself authorise or permit entry into the grant areas. The Court held that without permission from a land council a person holding a fishing licence could not fish in tidal waters within the grant areas. Fishing in those waters was to enter or remain on Aboriginal land, contrary to section 70(1). The common law right of fishing could be abrogated or regulated. By necessary implication, the Fisheries Act abrogated any public right to fish in NT tidal waters and the comprehensive statutory regulation of fishing provided by that Act supplanted any public right to fish in tidal waters. The NT, Director of Fisheries and the Seafood Council submitted that the section 70 prohibition against entering or remaining on Aboriginal land extended only to land and did not include entering or remaining in tidal waters over that land. The Land Trust, the NLC and native title holders submitted that the grants related to a defined geographical area and that entry to that area, whether covered by tidal waters or not, was prohibited by section 70. The majority accepted that submission. It held that the Land Rights Act expressly provided for the grant of estates in fee simple over areas that included sections that would be covered by tidal waters. The High Court made the declaration that the Fisheries Act did not confer on the Director of Fisheries a power to grant a licence under that Act which would, without more, authorise or permit the holder to enter and take fish or aquatic life from areas within the boundary lines of the mainland grant and the islands grant. * The listed native title holders are Gawirrin Gumana, Djamawa Marawili, Marrirra Marawili, Nuwandjali Marawili, Daymambi Munuggurr, Manman Wirrpanda and Dhukal Wirrpanda, on behalf of the Yarrwidi Gumatj, Manggalili, Gumana Dhalwangu, Wunungmurra (Gurrumuru) Dhalwangu, Dhupuditj Dhalwangu, Munyuku, Yithuwa Madarrpa, Gupa Djapu, Dhudi Djapu, Marrakula1, Marrakula2, and Nurrurawu Dhappuyngu (Dhurili/Durila) groups. +HIGH COURT OF AUSTRALIA 9 March 2022 HOBART INTERNATIONAL AIRPORT PTY LTD v CLARENCE CITY COUNCIL & ANOR; AUSTRALIA PACIFIC AIRPORTS (LAUNCESTON) PTY LTD v NORTHERN MIDLANDS COUNCIL & ANOR [2022] HCA 5 Today the High Court, by majority, dismissed two appeals from a judgment of the Full Court of the Federal Court of Australia. The principal question was whether a dispute sought to be agitated by the Clarence City Council and the Northern Midlands Council ("the Councils") concerning the meaning of a contractual term in leases to which the Councils were not parties involves a "matter" for the purposes of Ch III of the Constitution. In 1998, as part of a project to privatise Australia's federal airports, the Commonwealth entered into leases pursuant to s 22 of the Airports (Transitional) Act 1996 (Cth) ("the Leases") with the operators of the Hobart Airport and Launceston Airport ("the Lessees") for the Hobart Airport site and the Launceston Airport site ("the Airports") respectively. The Airports are not amenable to council rates or State land tax because they are on Commonwealth land. Clause 26.2(a) of the Leases requires that, in lieu of paying rates, the Lessees pay the Councils an amount equivalent to the amount that would have been payable if the Airports were not on Commonwealth land, but relevantly only in respect of parts of the Airports on which "trading or financial operations are undertaken". The Lessees are required to use "all reasonable endeavours" to enter into agreements with the Councils to make such payments. The Councils have never been parties to the Leases. The Councils sought declaratory relief regarding the proper construction of cl 26.2(a) and the Lessees' obligations to make payments in circumstances where the Commonwealth and the Lessees were not in dispute about the operation of cl 26.2(a) or the Lessees' compliance with it. The primary judge dismissed the applications on the basis that the Councils lacked standing. The Full Court of the Federal Court allowed the Councils' appeals, holding that they had standing, and that the Councils' claims raised a "matter". A majority of the High Court reasoned that the rights, duties and liabilities in dispute are private law rights, duties and liabilities of the Commonwealth and the Lessees under the Leases (although they might have a "public" dimension or complexion). They owe their existence to, and therefore "arise under", a Commonwealth law within s 76(ii) of the Constitution. Whether there is a justiciable controversy turned on whether the Councils have standing. An outsider to a contract seeking declaratory relief in relation to private rights may, for reasons other than having legally enforceable rights – in "exceptional circumstances" – have a "sufficient" or "real" interest to seek declaratory relief as to the meaning and effect of a contract between contracting parties. On this basis, three Justices held that, in this case, the Councils have a "sufficient" or "real" interest in seeking declaratory relief about the proper construction of cl 26.2(a) and, therefore, the dispute involves a "matter" for the purposes of Ch III of the Constitution. Two Justices reached the same conclusion on the basis that the Councils' interests in the relief sought are distinctive, substantial and squarely within the scope of those third party interests that were sought in the public interest to be advanced through entering into the Leases in the exercise of statutory authority. +HIGH COURT OF AUSTRALIA 7 September 2005 BANKSTOWN CITY COUNCIL v ALAMDO HOLDINGS PTY LIMITED The Council was not liable for stormwater damage to two industrial buildings owned by Alamdo, the High Court of Australia held today. Alamdo bought its site at Chester Hill in Sydney in 1988. The land is at a low point of the local catchment area and managing director Anthony Maurici was aware of an incident of short-lived flooding of both buildings not long before he bought the site. In 1998 and 2000, parts of the land, including one building on the first occasion, were briefly flooded. A stormwater channel, connected the Council’s drainage system, runs alongside the land. Water ran into the channel from a “gross pollutant trap” or barrage where two 2.4-metre stormwater pipes met near the property’s boundary. Alamdo brought a nuisance action in the New South Wales Supreme Court. Justice Ian Gzell held that the frequency with which the land was likely to be inundated had significantly increased from once every five to 10 years in 1960 to more than once every two years by 1998. This affected plans for the site and was an unreasonable interference with the use and enjoyment of the land. Justice Gzell granted an injunction restraining the Council from causing or permitting stormwater from inundating the land and ordered the Council to carry out abatement works which the Council estimated would cost $1.5 million. A valuer called by the Council estimated the value of the site had diminished by more than $1.4 million, while Alamdo claimed the figure was higher and that the risk of flooding had cut rental values for both buildings. Justice Gzell did not award damages. The Court of Appeal dismissed both an appeal by the Council and a cross-appeal by Alamdo, but it suspended the injunction pending the outcome of the High Court appeal by the Council. The Council based the appeal on its contention that section 733 of the NSW Local Government Act provided an exemption from liability for decisions about land liable to flooding. The High Court unanimously allowed the appeal. It held that a body such as the Council is not, without negligence on its part, liable for a nuisance attributable to the exercise of, or failure to exercise, its statutory powers. The Court rejected Alamdo’s contentions that section 733 did not extend to protecting the Council from exposure to injunctive relief and that the Council had not acted in good faith so could not rely upon section 733. The Court held that section 733 applied in this case. +HIGH COURT OF AUSTRALIA 12 October 2022 BOSANAC v COMMISSION OF TAXATION & ANOR [2022] HCA 34 Today, the High Court unanimously allowed an appeal from the Full Court of the Federal Court of Australia. The appeal concerned whether the appellant ("Ms Bosanac") held half of her interest in a residential property ("the Property") on trust for her husband, the second respondent ("Mr Bosanac"), where the purchase money was paid jointly by Ms and Mr Bosanac. Ms Bosanac purchased the Property with loans held and taken out jointly with Mr Bosanac, secured against properties they each separately owned. The Property was registered in Ms Bosanac's name alone and used as the matrimonial home of the Bosanacs. Mr Bosanac never claimed an interest in the Property. Ms and Mr Bosanac had a history of holding their substantial real and other properties in their own names and using them as security for joint loans. The first respondent ("the Commissioner") is a creditor of Mr Bosanac. The Commissioner, relying upon the presumption of resulting trust, sought a declaration that Ms Bosanac holds half of her interest in the Property on trust for Mr Bosanac. The Commissioner argued that the presumption of advancement of a wife by her husband, which operates to preclude a resulting trust from arising, is no longer part of the law of Australia in relation to the matrimonial home. The primary judge dismissed the Commissioner's application, holding that the presumption of advancement arose in Ms Bosanac's favour, and that the evidence did not support an inference that Mr Bosanac intended to have an interest in the Property. The Full Court allowed an appeal, holding that the presumption was rebutted by the evidence in this case. The High Court held the presumption of resulting trust will not arise where there is evidence from which it may be inferred that the parties' objective intention is inconsistent with the person providing the purchase money obtaining an interest in a property. The "presumption" of advancement allows an inference to be drawn from the fact of certain relationships, such as husband and wife, that the presumption of resulting trust will not arise. When evidence of the way in which the spouses deal with their property is given, inferences to the contrary of the presumption may readily be drawn. Both presumptions cease to be of practical significance other than in a rare case where the totality of evidence is incapable of founding an inference as to what the purchaser intended. The question of intention as to whether a trust arises is entirely one of fact, and the facts of this case do not give rise to such an intention. The proper inference to be drawn from the objective facts was that the parties objectively intended Ms Bosanac to be the sole beneficial owner of the Property, and that Mr Bosanac was merely facilitating Ms Bosanac's acquisition of the Property. The Commissioner invited the Court to abolish the presumption of advancement as having no acceptable rationale and being anomalous, anachronistic, and discriminatory. The Court refused, observing the "presumption" of advancement is an entrenched "land-mark" of the law in Australia. +HIGH COURT OF AUSTRALIA 9 November 2016 TIMBERCORP FINANCE PTY LTD (IN LIQUIDATION) v COLLINS & ANOR; TIMBERCORP FINANCE PTY LTD (IN LIQUIDATION) v TOMES [2016] HCA 44 Today the High Court unanimously dismissed two appeals from the Court of Appeal of the Supreme Court of Victoria. The High Court held that it was not unreasonable, in the Port of Melbourne Authority v Anshun Pty Ltd (1981) 147 CLR 589; [1981] HCA 45 ("Anshun") sense, for the respondents in each appeal not to have raised issues specific to them in a group proceeding. Accordingly, the respondents were not estopped from raising these issues in subsequent proceedings, nor was it an abuse of process to do so. The appellant in both appeals was part of the Timbercorp Group. Its purpose was to provide loans to investors in managed investment schemes operated by another company in the Timbercorp Group. Each of the members of the Timbercorp Group was placed in liquidation in June 2009. A group proceeding was commenced against members of the Timbercorp Group and certain of their directors in the Supreme Court of Victoria in October 2009 under Pt 4A of the Supreme Court Act 1986 (Vic) by Mr Woodcroft-Brown ("the lead plaintiff"), on his own behalf and on behalf of persons who held an interest in certain managed investment schemes, including the respondents ("the group proceeding"). The respondents had previously applied for loans from the appellant to fund their investments in the schemes. The respondents each received and read "opt out notices", but chose not to opt out of the group proceeding. The lead plaintiff's case was essentially that Timbercorp Securities, another member of the Timbercorp Group, had failed to comply with its statutory obligations to disclose information about risks and had made certain misrepresentations, and that the appellant was involved in that conduct. The group proceeding was not successful at trial or on appeal. The appellant subsequently commenced separate proceedings against Mr and Mrs Collins and against Mr Tomes, in which it alleged that the respondents had defaulted under their respective loan agreements and sought recovery of the outstanding principal and interest. Mr and Mrs Collins, in their defence, contended that they did not acquire an interest in the project in which they sought to invest through Timbercorp Securities and that no loan was advanced to them by the appellant for that purpose. Mr Tomes' defence included allegations that no loan agreement was concluded between him and the appellant, that the monies he paid were not properly applied, and that an agent of the appellant had represented that, in the event of his default under the loan, the appellant would not seek recourse against him. The appellant claimed that the respondents were precluded, by reason of Anshun estoppel or abuse of process, from raising their defences because they should have raised them for determination in the group proceeding, or alternatively they should have opted out of the group proceeding. The primary judge held that the respondents were not precluded from raising their defences. The Court of Appeal dismissed the appeals from that decision. By grant of special leave, the appellant appealed to the High Court. The High Court unanimously dismissed both appeals. The High Court held it was not unreasonable for the respondents not to have raised their individual issues concerning the loan agreements in the group proceeding, in circumstances where: the issues in the group proceeding concerned undisclosed risks and the making of misrepresentations; there was no issue in the group proceeding as to the validity of the loan agreements; and the respondents had limited control over the group proceeding. Nor was it unreasonable in the circumstances of the case for the respondents not to have opted out of the group proceeding. Further, the respondents' failure in the group proceeding to bring their claims to the attention of the Court did not affect the case management decisions open to the Court, and thus raising the defences in these proceedings did not amount to an abuse of process. +HIGH COURT OF AUSTRALIA 14 March 2018 IN THE MATTER OF QUESTIONS REFERRED TO THE COURT OF DISPUTED RETURNS PURSUANT TO SECTION 376 OF THE COMMONWEALTH ELECTORAL ACT 1918 (CTH) CONCERNING MS JACQUI LAMBIE [2018] HCA 6 Today the High Court published reasons for orders it made on 6 February 2018. Those orders were that Mr Steven Martin is not incapable of being chosen or of sitting as a senator by reason of s 44(iv) of the Constitution. On 8 December 2017 the High Court, sitting as the Court of Disputed Returns, answered questions referred to it by the Senate under s 376 of the Commonwealth Electoral Act 1918 (Cth) ("the Act") concerning Ms Jacqui Lambie. The answers given by the Court included that, by reason of s 44(i) of the Constitution, there was a vacancy in the representation of Tasmania in the Senate for the place for which Ms Lambie was returned at the federal election held on 2 July 2016, and that the vacancy should be filled by a special count of the ballot papers. The special count that was conducted identified Mr Martin as one of the candidates who would be elected as a senator for Tasmania as a result of the special count. On 13 December 2017, Nettle J stated for the consideration of the Full Court the question of whether Mr Martin is incapable of being chosen or of sitting as a senator by reason of s 44(iv) of the Constitution. Mr Martin has at all relevant times held the elected offices of mayor and of councillor of Devonport City Council, a local government corporation established under the Local Government Act 1993 (Tas) ("the Local Government Act"). In respect of each office, Mr Martin has a statutory entitlement to be paid a substantial annual allowance by the Council. There was no dispute that the positions which Mr Martin held answered the description of "offices of profit". The issue before the Court was whether the positions were offices of profit "under the Crown" within the meaning of s 44(iv), which would render Mr Martin "incapable of being chosen or of sitting" as a senator. The Court unanimously held that the offices of mayor and of councillor held by Mr Martin are not offices of profit "under the Crown" within the meaning of s 44(iv). A majority of the Court held that the particular conflict to which s 44(iv) is addressed is that which would arise from a member of Parliament holding at the will of an executive government an office in respect of which he or she receives a financial gain. The majority held that the Local Government Act did not confer on the executive government of Tasmania effective control over the holding of or profiting from the office of mayor or of councillor. The Court unanimously held that Mr Martin is not incapable of being chosen or of sitting as a senator by reason of s 44(iv) of the Constitution. +HIGH COURT OF AUSTRALIA 12 April 2023 DAVIS v MINISTER FOR IMMIGRATION, CITIZENSHIP, MIGRANT SERVICES AND MULTICULTURAL AFFAIRS & ORS; DCM20 v SECRETARY OF DEPARTMENT OF HOME AFFAIRS & ANOR [2023] HCA 10 Today the High Court, by majority, allowed two appeals from a decision of the Full Court of the Federal Court of Australia. The dispositive issue considered by the Court was whether instructions issued to departmental officers by the Minister in 2016 in the exercise of Commonwealth executive power ("Ministerial Instructions"), and purported decisions made by departmental officers in compliance with them, exceeded the limits of the executive power of the Commonwealth because the Ministerial Instructions purported to instruct those officers to make a decision which, by legislative stipulation, could be made only by the Minister personally. Each appellant, upon a delegate of the Minister refusing their visa application and an administrative tribunal affirming that refusal, requested that the Minister exercise the power under s 351(1) of the Migration Act 1958 (Cth) ("the Act") to substitute a more favourable decision for the tribunal's adverse decision "[i]f the Minister thinks that it is in the public interest to do so". Pursuant to s 351(3), that power may only be exercised by the Minister personally. The Ministerial Instructions relevantly instructed departmental officers not to refer such requests to the Minister for consideration unless satisfied that the case had "unique or exceptional circumstances". The officers were not so satisfied and, in accordance with the Ministerial Instructions, the appellants' requests were finalised by the Department without referral. At first instance and on appeal before the Full Court, the appellants unsuccessfully argued that the departmental officers' decisions were legally unreasonable. The Full Court refused leave to raise a new ground which alleged the Ministerial Instructions were unlawful. The High Court granted the appellants special leave to appeal on both the unlawfulness and unreasonableness grounds of appeal. The power conferred personally on the Minister by s 351(1) comprises two distinct decisions, each involving a non-delegable exercise of the statutory power: (1) a procedural decision to consider or not to consider whether to make a substantive decision; and (2) a substantive decision to substitute or not to substitute, in the public interest, a more favourable decision. The Minister could not exercise executive power, which is constrained by the statutory scheme, to delegate either of these decisions to departmental officers. The Court found that the broad criterion of "unique or exceptional circumstances" in the Ministerial Instructions required, in effect, a departmental officer to evaluate the public interest and make a decision entrusted exclusively to the Minister. For that reason, the Ministerial Instructions exceeded the limitation imposed by s 351(3) on the executive power of the Commonwealth. The decisions made in purported compliance with the Ministerial Instructions were therefore unlawful. As the departmental officers' decisions were not decisions made under the Act, the appeals were not excluded from the jurisdiction of the Federal Court under s 476A(1) of the Act, and accordingly were not excluded from the High Court's appellate jurisdiction under s 73 of the Constitution. +HIGH COURT OF AUSTRALIA 7 February 2008 Public Information Officer GYPSY JOKERS MOTORCYCLE CLUB INCORPORATED v COMMISSIONER OF POLICE The High Court of Australia today upheld the validity of Western Australian legislation concerning judicial review of fortification removal notices. The Gypsy Jokers owns a clubhouse in an industrial area of Maddington in Perth. The clubhouse had a concrete front wall, surveillance cameras, steel doors and modified timber doors. In 2004, the Western Australian Police Commissioner applied to the Corruption and Crime Commission (CCC) for a fortification warning notice in respect of the clubhouse. The notice, issued on 31 March 2004, said the CCC was satisfied that there were reasonable grounds for believing that the clubhouse was “heavily fortified” and was “habitually used as a place of resort by members of a class of people a significant number of whom may reasonably be suspected to be involved in organised crime”. The Gypsy Jokers’ solicitors wrote back explaining that the fortifications were necessary because the area had high rates of burglary and car theft, 10 valuable customised Harley Davidson Big Twin motorcycles were stored there, and the fortifications had received necessary approvals. The letter did not address the CCC’s belief that people involved in organised crime used the clubhouse. On 5 May 2004, the Commissioner issued a fortification removal notice listing for removal the concrete wall, the gates attached to the wall, surveillance cameras and monitors and an internal door. The Gypsy Jokers applied pursuant to section 76(1) of the Corruption and Crime Commission Act to the WA Supreme Court for review of the fortification removal notice. Section 76(2) provided that the Police Commissioner may identify information as confidential if its disclosure might prejudice police operations and such information would then be for the Court’s use only and not disclosed in any way. The Gypsy Jokers received only an edited version of an affidavit provided by the Commissioner to the Court. The affidavit listed 59 members, all but one of whom the Commissioner alleged had criminal records, and 130 charges with which the Commissioner alleged members or associates had been charged. The Gypsy Jokers challenged the validity of section 76(2) and Justice Peter Blaxell referred the issue to the Court of Appeal. The review of the fortification removal notice has yet to be adjudicated. The Court of Appeal, by majority, upheld the validity of section 76(2). The Gypsy Jokers appealed to the High Court, arguing that section 76(2) purported to exercise an impermissible form of control over the exercise by the Supreme Court of its jurisdiction and constituted a denial of procedural fairness. The High Court, by a 6-1 majority, dismissed the appeal. The Court held that it was for the Supreme Court, not for the Police Commissioner, to determine whether disclosure of information provided by the Commissioner might prejudice police operations. Section 76(2) of the CCC Act did not render unexaminable by the Supreme Court the decision of the Commissioner. It did not direct the Supreme Court how to exercise its jurisdiction so as to impair the character of the Court as independent and impartial. The High Court held that without section 76(2), a public interest immunity could apply, which meant that the Supreme Court could not consider information subject to the immunity and consequently could not exercise its review function. +HIGH COURT OF AUSTRALIA 14 August 2019 GLENCORE INTERNATIONAL AG & ORS v COMMISSIONER OF TAXATION OF THE COMMONWEALTH OF AUSTRALIA & ORS [2019] HCA 26 Today the High Court unanimously dismissed proceedings by which the plaintiffs, being companies within the global Glencore plc group, sought an injunction restraining the defendants – the Commissioner, the Second Commissioner and the Deputy Commissioner of Taxation – and any other officer of the Australian Taxation Office from making any use of documents described as the "Glencore documents", on the basis that those documents are subject to legal professional privilege. The High Court held that legal professional privilege is not an actionable legal right capable of sounding in injunctive relief. The plaintiffs identified the Glencore documents as having been created for the sole or dominant purpose of the provision by Appleby (Bermuda) Limited ("Appleby"), an incorporated law practice in Bermuda, of legal advice to the plaintiffs. They were amongst documents known as the "Paradise Papers", which were stolen from Appleby's electronic file management systems, provided to the International Consortium of Investigative Journalists, and further disseminated. After becoming aware that the Glencore documents were in the defendants' possession, the plaintiffs asserted that those documents are subject to legal professional privilege, and requested that the defendants return them and provide an undertaking that they would not be referred to or relied upon. The defendants did not accede to those requests. The plaintiffs brought proceedings in the original jurisdiction of the High Court, seeking an injunction in equity's auxiliary jurisdiction restraining the defendants' use of the Glencore documents and orders requiring the delivery up of the documents. The plaintiffs claimed that legal professional privilege was a sufficient basis for the grant of the injunction sought, and did not seek to rely on the equitable doctrine of breach of confidence, or to expand any other area of law. The defendants demurred on the principal ground that no cause of action was disclosed by which the plaintiffs were entitled to that relief. The Court upheld the demurrer on the principal ground, holding that legal professional privilege is not a legal right which may found a cause of action. The privilege is only an immunity from the exercise of powers that would otherwise compel the disclosure of privileged communications. The Court held that policy considerations cannot justify the creation of a new, actionable right respecting privileged documents in circumstances where that development is not available having regard to the state of settled principles. The Court observed that on the present state of the law, once privileged communications have been disclosed, resort must be had to the equitable doctrine of breach of confidence for protection respecting the use of that material. +HIGH COURT OF AUSTRALIA 4 May 2016 [2016] HCA 17 Today the High Court unanimously dismissed an appeal from a decision of the Court of Criminal Appeal of the Supreme Court of New South Wales. The High Court held that the Court of Criminal Appeal was correct to quash the sentences imposed by the sentencing judge and re-sentence the appellant as was done. The appellant shot and wounded the deceased, who was a police officer, while the deceased was lawfully executing a search warrant in company with other police officers on premises in close proximity to the appellant's home. The shot struck the deceased in the arm, thereby causing him a serious but non-fatal gunshot wound. In the course of the fire-fight which ensued, one of the other police officers fired a shot which was intended for the appellant, but which unfortunately instead hit the deceased in the neck, thereby inflicting a wound from which he later died. About two weeks before these events, the appellant was the victim of an attempted robbery. The Crown accepted that it could not exclude as a reasonable possibility that the appellant honestly believed that the deceased was someone posing as a police officer who was intent on robbing the appellant. The appellant pleaded guilty to one count of manslaughter and one count of wounding with intent to cause grievous bodily harm. The appellant was taken to have accepted responsibility for killing the deceased by excessive self-defence on the basis that, by firing at the deceased, he substantially contributed to the ensuing exchange of gunfire where it was reasonably foreseeable that someone in the vicinity of the exchange might be fatally, even if inadvertently, shot. The appellant was sentenced by a judge of the Supreme Court of New South Wales to a total effective sentence of nine years and six months' imprisonment. On a Crown appeal, the Court of Criminal Appeal held, with reference to R v De Simoni (1981) 147 CLR 383 ("De Simoni"), that the sentencing judge erred in her assessment of the objective gravity of the offence of manslaughter by contrasting it with what the sentencing judge supposed would have been the gravity of the offence if the appellant had known that the deceased was a police officer. The Court of Criminal Appeal also considered that the sentencing judge erred by not cumulating part of the sentence imposed for the offence of wounding on the sentence imposed for the offence of manslaughter. The Court of Criminal Appeal was satisfied that the sentences were manifestly inadequate. The Court of Criminal Appeal quashed the sentences and re-sentenced the appellant to a total effective sentence of 17 years and two months' imprisonment. By grant of special leave, the appellant appealed to the High Court. The High Court unanimously dismissed the appeal, holding that although the Court of Criminal Appeal's reference to De Simoni was misplaced, it was correct to hold that the sentencing judge erred in her assessment of the objective gravity of the offence of manslaughter and correct to quash the sentences imposed by the sentencing judge and re-sentence the appellant as was done. +HIGH COURT OF AUSTRALIA Manager, Public Information 2 September 2009 RALPH DESMOND CLARKE v COMMISSIONER OF TAXATION & ANOR [2009] HCA 33 The Commonwealth’s attempt to impose a surcharge tax on members of State parliaments, based on their notional entitlements under defined benefit superannuation schemes, was constitutionally invalid, the High Court held today. Ralph Clarke was elected to the South Australian Parliament in 1993. He served as a member of parliament from 11 December 1993 until 8 February 2002. Mr Clarke was a member of three state superannuation schemes: the Parliamentary Superannuation Scheme (PS Scheme); the Southern State Superannuation Scheme (SSS Scheme); and the State Superannuation Benefit Scheme which was rolled into the SSS Scheme under South Australian superannuation scheme merger legislation. Between February 2000 and February 2005 the Commissioner of Taxation issued superannuation contribution surcharge assessments to Mr Clarke, pursuant to the Superannuation Contributions Tax (Members of Constitutionally Protected Superannuation Funds) Imposition Act 1997 (Cth) (the Imposition Act) and the Superannuation Contributions Tax (Members of Constitutionally Protected Superannuation Funds) Assessment and Collection Act 1997 (Cth) (the Assessment Act). Mr Clarke’s objections to the assessments were disallowed by the Commissioner. He applied to the Administrative Appeals Tribunal (AAT) for review of the Commissioner’s decisions and the AAT referred questions of law, including a question about the constitutional validity of the Imposition Act and the Assessment Act, to the Full Court of the Federal Court. The Full Court held that both Acts were valid. The High Court granted special leave to Mr Clarke to appeal the decision of the Full Court. The Imposition Act and the Assessment Act applied to “constitutionally protected funds” which included the schemes of which Mr Clarke was a member. Liability was imposed on a fund member if the member’s adjusted taxable income exceeded a defined threshold amount. (South Australian state parliamentarians fell within this group.) The two Acts obliged fund members to pay amounts calculated on the basis of notional contributions that bore no necessary relation to the pension he or she would actually receive. Potentially the benefits received could be less than the amounts assumed in the calculation of the surcharge. Also, the tax accrued, compounding at market interest rates, until the member actually received his or her superannuation benefits. Potentially the tax due could approximate the whole of the pension due in the first year of receipt. To ameliorate these potential effects the South Australian government passed legislation whose general aim was to ensure that persons with an accumulated surcharge debt with the Australian Taxation Office had at retirement a method of obtaining a lump sum to expunge the debt with the ATO. The High Court concluded that the Constitution recognises the States as independent entities and will not support laws which impair or interfere with the capacities or functions conferred on the States or which inhibit the execution of their constitutional powers. The Court found that the Imposition Act and the Assessment Act were not laws of general application, but rather laws which placed a special disability or burden on the States in relation to the way in which they remunerated members of State parliaments. Remuneration of members of parliament, as one aspect of the capacity to fix terms and conditions of those elected to parliament, is critical to each State’s capacity to function as a government. The Court determined that for these reasons, both the Imposition Act and the Assessment Act were beyond the legislative power of the Commonwealth and made orders reflecting that determination. +HIGH COURT OF AUSTRALIA 15 August 2018 SORWAR HOSSAIN v MINISTER FOR IMMIGRATION AND BORDER PROTECTION & ANOR [2018] HCA 34 Today the High Court unanimously held that an error of law made by the Administrative Appeals Tribunal in relation to one criterion for the grant of a partner visa did not invalidate the Tribunal's decision not to grant the visa because the Tribunal correctly concluded that the requirements of another criterion were not met. The Court held that, in those circumstances, the Tribunal's error of law in relation to the first criterion did not materially affect the Tribunal's decision. Mr Sorwar Hossain, a citizen of Bangladesh, applied for a partner visa in May 2015. A delegate of the Minister for Immigration and Border Protection refused to grant the visa and Mr Hossain applied to the Tribunal for review of the delegate's decision. The Tribunal affirmed the delegate's decision because the Tribunal was not satisfied that two criteria prescribed by the Migration Regulations 1994 (Cth) had been met. The first criterion was that the application for the visa be made within 28 days of the applicant ceasing to hold a previous visa "unless the Minister is satisfied that there are compelling reasons for not applying" that criterion. The second criterion was that the visa applicant "does not have outstanding debts to the Commonwealth unless the Minister is satisfied that appropriate arrangements have been made for payment". Section 65 of the Migration Act 1958 (Cth) provided that "if satisfied" that all the criteria prescribed for the visa had been met, the Minister was to grant the visa; and that, "if not so satisfied", the Minister was to refuse to grant the visa. For the purposes of the review of the delegate's decision, the Tribunal was required to determine whether it was satisfied that the criteria had been met. The Tribunal was not satisfied that the first criterion had been met because Mr Hossain had not applied for the partner visa within 28 days of ceasing to hold a previous visa and the Tribunal was satisfied that there were no compelling reasons, as at the time at which Mr Hossain had applied for the partner visa, for not applying that criterion. The Tribunal also was not satisfied that the second criterion had been met because Mr Hossain had a debt to the Commonwealth which he had made no arrangements to repay, although Mr Hossain told the Tribunal that he intended to repay the debt. The Tribunal accordingly affirmed the delegate's decision not to grant the partner visa. Mr Hossain applied to the Federal Circuit Court of Australia for judicial review of the Tribunal's decision. Before that Court, the Minister conceded that the Tribunal had made an error of law by deciding that there were no compelling reasons for not applying the first criterion as at the time of the visa application, whereas the Tribunal should have decided whether such reasons existed as at the time of the Tribunal's decision. The Federal Circuit Court held that this error was jurisdictional in nature and meant that the Tribunal's decision was invalid, notwithstanding that the Tribunal also had not been satisfied that the second criterion had been met. On appeal, a majority of the Full Court of the Federal Court held that the Tribunal's error was jurisdictional in nature, but that the error had not stripped the Tribunal of authority to affirm the delegate's decision. By grant of special leave, Mr Hossain appealed to the High Court. The Court held that a decision-maker is required to proceed on a correct understanding of the applicable law, but that an error of law will not be jurisdictional in nature if the error does not materially affect the decision. The Tribunal's findings with respect to the second criterion provided an independent basis on which the Tribunal was bound to affirm the delegate's decision. The suggestion that the Tribunal might have allowed Mr Hossain more time to arrange to repay his debt if the Tribunal had not made the error was insufficient to demonstrate that the Tribunal's decision might have been different had it not made the error. The Court therefore dismissed the appeal. +HIGH COURT OF AUSTRALIA 8 June 2011 GARY ERNEST WHITE v THE DIRECTOR OF PUBLIC PROSECUTIONS FOR WESTERN AUSTRALIA [2011] HCA 20 Today the High Court dismissed an appeal against the decision of the Court of Appeal of the Supreme Court of Western Australia to make a crime-used property substitution declaration under s 22 of the Criminal Property Confiscation Act 2000 (WA) ("Act") in respect of property owned by the appellant. In 2003 the appellant was convicted of a wilful murder which was committed immediately outside the boundary fence of fenced and gated premises in Maddington, Western Australia in 2001. At the time of the offence, the appellant was leasing the premises. The appellant shot at the deceased five times inside the premises, the gates to which had been locked at the appellant's direction. At least two shots wounded the deceased. The deceased climbed over one of the locked gates to escape the appellant but was fatally shot while on the ground outside the gate. The appellant was sentenced to strict-security life imprisonment with a non-parole period of 22 years. Section 22 of the Act provides that a "crime-used property substitution declaration" can be made, on the application of the Director of Public Prosecutions ("the DPP"), if two conditions are satisfied: first, the crime-used property is not available for confiscation because it neither belongs to nor is effectively controlled by the offender; and second, it is more likely than not that "criminal use" was made of the "crime-used property". If the conditions in s 22 of the Act are satisfied, the court must make a declaration that property owned by the offender is available for confiscation instead of the crime-used property. Section 147 provides that a person makes criminal use of property if the person, alone or with anyone else, uses the property in a way that brings the property within the definition of "crime-used property". Section 146(1)(a) provides that property is crime-used if the property is used, directly or indirectly, in or in connection with the commission of a confiscation offence, or in or in connection with facilitating the commission of a confiscation offence. Section 146(1)(c) provides that property is "crime-used" if any act or omission was done, omitted to be done or facilitated in or on the property in connection with the commission of a confiscation offence. Relevantly, a "confiscation offence" is an offence that is punishable by imprisonment for two years or more (s 141(1)(a)). The DPP applied for a crime-used property substitution declaration against the appellant. The primary judge dismissed the DPP's application holding that the appellant's conduct only fell within s 146(1)(c), which was not covered by the definition of "criminal use" in s 147. The Court of Appeal of Western Australia allowed the DPP's appeal and made the declaration sought by the DPP against the appellant. The Court of Appeal held that the concept of "criminal use" in s 147 encompassed all activities that brought property within the definition of "crime-used" under s 146. The appellant appealed to the High Court. The High Court held that "makes criminal use of property" within s 147 encompasses conduct within s 146(1)(c) of the Act. It was not disputed that the premises leased by the appellant were "crime-used" within s 146(1)(c). The appellant therefore made "criminal use" of the premises within s 147. The conditions of s 22 of the Act were satisfied and the Court of Appeal was correct to make the crime-used property substitution declaration. +HIGH COURT OF AUSTRALIA 29 September 2010 KOSTAS & ANOR v HIA INSURANCE SERVICES LIMITED T/AS HOME OWNERS WARRANTY & ANOR [2010] HCA 32 In August 1999, the appellants engaged a builder to undertake substantial renovations to their home. By June 2000, disputes had arisen and the appellants purported to terminate the building contract. They made a claim with HIA Insurance Services Limited ("HIA"), the statutory insurer of the builder's work. Upon HIA rejecting the claim, the appellants commenced proceedings in September 2000 against HIA and the builder in the Consumer, Trader and Tenancy Tribunal of New South Wales. In a decision handed down in May 2005, the Tribunal held that the appellants had not validly terminated the contract. Central to this decision was a finding that the builder had served two claims for extensions of time in accordance with the contract and that the appellants had not disputed those claims in the manner required by the contract. The appellants appealed against the Tribunal's decision to the Supreme Court of New South Wales under s 67 of the Consumer, Trader and Tenancy Tribunal Act 2001 (NSW). At the relevant time, that section provided for a right of appeal to the Supreme Court against a decision by the Tribunal of "a question with respect to a matter of law". The appellants contended that there was no evidence properly before the Tribunal that supported its finding that the builder had served time- extension claims in accordance with the contract. They contended that, as a result, the Tribunal had made an erroneous decision of a question with respect to a matter of law. The Supreme Court agreed, finding that the termination of the contract was lawful and effective and quashing the Tribunal's decision. HIA successfully appealed to the Court of Appeal on the basis that there was no "question with respect to a matter of law" to attract the Supreme Court's jurisdiction. The Court of Appeal held that a contention that there was no evidence to support a factual finding of the Tribunal could not form the basis of a statutory appeal under s 67 of the Consumer, Trader and Tenancy Tribunal Act. The appellants were granted special leave to appeal to the High Court against the Court of Appeal's decision. Today, the High Court allowed the appeal and restored the decision of the Supreme Court. The Court held that the question whether there was no evidence to support a factual finding of the Tribunal was a question with respect to a matter of law. The factual finding in this case was that the builder had served the relevant time-extension claims. The Court held that there was no evidence before the Tribunal upon which it could make this finding. The jurisdiction of the Supreme Court under s 67 was properly invoked and the decision of the judge at first instance on this point was correct. +HIGH COURT OF AUSTRALIA 15 November 2005 ANDREW MARK MALLARD v THE QUEEN The High Court of Australia today ordered a retrial of Mr Mallard, who was convicted exactly 10 years ago of the murder of Perth jeweller Pamela Suzanne Lawrence. It unanimously allowed the appeal and held that the Western Australia Court of Criminal Appeal had erred in the way in dealt with Mr Mallard’s petition for clemency. On 23 May 1994, Mrs Lawrence, 45, was found dying in a pool of blood in her shop in Mosman Park after being struck repeatedly on the head with a heavy instrument that was never found. Mr Mallard who was staying nearby was convicted. He appealed unsuccessfully to the Court of Criminal Appeal and was refused special leave to appeal to the High Court. After the discovery of various material in the possession of police but never disclosed to the defence, Mr Mallard petitioned for clemency. Pursuant to section 140(1)(a) of the Sentencing Act, the Attorney-General referred the petition to the Court of Criminal Appeal which dismissed it in 2003. Mr Mallard, now 42, was granted special leave to appeal to the High Court. Section 140(1)(a) requires “the whole case to be heard and determined as if it were an appeal”. The Court held that the Court of Criminal Appeal took an overly narrow approach to this task, as it proceeded as if there were inhibitions upon its jurisdiction to consider, not just the evidence that was adduced at trial, but also its relevance to the further evidence that Mr Mallard sought to introduce and rely upon in his petition for clemency. The High Court held that “the whole case” embraced all the properly admissible evidence, whether new, fresh or previously adduced, for or against an appellant. The words “as if it were an appeal” referred to the making of orders, the following of appropriate procedures, and the requirement that the court should apply the proviso, in section 689(1) of the Criminal Code, that despite an appeal point being decided in favour of an appellant, the appeal should be dismissed if no substantial miscarriage of justice had occurred. Mr Mallard allegedly made various admissions in three police interviews, with only one, of 30 minutes, recorded. One interview was conducted over more than eight hours while he was a patient at Graylands Mental Hospital. Another occurred after Mr Mallard had spent the night at a nightclub, had been beaten up, and had had little sleep. He also frequently denied killing Mrs Lawrence. In the interviews he made highly fanciful and bizarre statements, sometimes speaking in the third person. Discrepancies existed between his statements and the reality, such as what Mrs Lawrence was wearing. He said the assailant would have used a Sidchrome wrench, which he drew for the police. It only emerged at the second appeal that police attempts to replicate Mrs Lawrence’s wounds by striking a pig’s head with a wrench and other objects kept in her workshop were unsuccessful. Witnesses’ statements were retyped by police to exclude material favourable to Mr Mallard, including a description of a man seen in the shop by a girl whose mother worked there. Other witness statements were not disclosed. A scientist’s report had two pages missing. The Court held that Mr Mallard’s confessions were unreliable and the non-presentation of evidence meant this was not a case where the Criminal Code proviso should apply. It ordered that his conviction be quashed and a new trial held. The Court noted however that having regard to what had emerged and to the Criminal Code now providing that admissions in serious cases are inadmissible unless videotaped it would be open to the Crown not to proceed with a retrial. +HIGH COURT OF AUSTRALIA 28 April 2004 BAYSIDE CITY COUNCIL & ORS v TELSTRA CORPORATION LIMITED & ORS MORELAND CITY COUNCIL v OPTUS VISION PTY LTD & ORS WARRINGAH COUNCIL & ORS v OPTUS VISION PTY LTD & ORS HURSTVILLE CITY COUNCIL & ORS v TELSTRA CORPORATION LIMITED & ORS The High Court of Australia today held that New South Wales and Victorian laws allowing councils to collect rates and charges from Telstra and Optus for installing and maintaining cables for pay television and high-speed internet access were contrary to Federal legislation preventing discrimination against telecommunications carriers and were invalid. Telstra and Optus began rolling out their broadband cable networks, using either existing power poles or underground ducts. Some Sydney and Melbourne councils responded by imposing charges for installation of cables or levying rates for space occupied by cables, of between $500 and $1,000 per kilometre, while other services and signage in public places did not attract charges or rates. In the Federal Court of Australia Telstra and Optus challenged the lawfulness of the rates and charges imposed by 14 NSW and five Victorian councils, claiming the impost was discriminatory. The Court dismissed the carriers’ action, but on appeal the Full Court declared invalid NSW and Victorian laws authorising councils to set charges and rates to the extent that these provisions were inconsistent with the Federal Telecommunications Act. Section 44(1)(a) of the Act provides that a State or Territory law has no effect if it discriminates against particular carriers, a particular class of carriers or carriers generally. Under section 109 of the Constitution, if State and federal laws are inconsistent the latter prevails to the extent of the inconsistency. The High Court held that promoting the development of telecommunications fell within a constitutional head of power of the Commonwealth Parliament. Conferring upon carriers immunity from discriminatory burdens imposed by State or Territory laws had a direct and substantial connection with that power. The High Court held that there was a clear general scheme of exemptions for utilities using similar facilities that did not extend to Telstra and Optus. The High Court, by a 6-1 majority, upheld the decision of the Full Court of the Federal Court that Telstra and Optus had made out a case of discrimination within section 44. The High Court dismissed the councils’ appeals with costs. Long titles of the four cases: BAYSIDE CITY COUNCIL, MORELAND CITY COUNCIL, FRANKSTON CITY COUNCIL AND YARRA CITY COUNCIL v TELSTRA CORPORATION LTD, TELSTRA MULTIMEDIA PTY LTD, HURSTVILLE CITY COUNCIL, KOGARAH MUNICIPAL COUNCIL, LEICHHARDT MUNICIPAL COUNCIL, PARRAMATTA CITY COUNCIL, PENRITH CITY COUNCIL, RANDWICK CITY COUNCIL, HORNSBY SHIRE COUNCIL, DRUMMOYNE COUNCIL, BURWOOD COUNCIL, CONCORD COUNCIL AND STRATHFIELD MUNICIPAL COUNCIL MORELAND CITY COUNCIL v OPTUS VISION PTY LTD, OPTUS NETWORKS PTY LTD, WARRINGAH COUNCIL, RANDWICK CITY COUNCIL AND BLACKTOWN CITY COUNCIL WARRINGAH COUNCIL, RANDWICK CITY COUNCIL AND BLACKTOWN CITY COUNCIL v OPTUS VISION PTY LTD, OPTUS NETWORKS PTY LTD AND MORELAND CITY COUNCIL HURSTVILLE CITY COUNCIL, KOGARAH MUNICIPAL COUNCIL, LEICHHARDT MUNICIPAL COUNCIL, PARRAMATTA CITY COUNCIL, PENRITH CITY COUNCIL, RANDWICK CITY COUNCIL, HORNSBY SHIRE COUNCIL, DRUMMOYNE COUNCIL, BURWOOD COUNCIL, CONCORD COUNCIL AND STRATHFIELD MUNICIPAL COUNCIL v TELSTRA CORPORATION LTD, TELSTRA MULTIMEDIA PTY LTD, BAYSIDE CITY COUNCIL, MORELAND CITY COUNCIL, FRANKSTON CITY COUNCIL AND YARRA CITY COUNCIL +HIGH COURT OF AUSTRALIA 8 September 2021 FAIRFAX MEDIA PUBLICATIONS PTY LTD v DYLAN VOLLER NATIONWIDE NEWS PTY LIMITED v DYLAN VOLLER AUSTRALIAN NEWS CHANNEL PTY LTD v DYLAN VOLLER [2021] HCA 27 Today, the High Court dismissed appeals from a judgment of the Court of Appeal of the Supreme Court of New South Wales concerning whether, by posting content relating to news stories about Mr Voller, the respondent, on their respective public Facebook pages, the appellants were liable for the publication of allegedly defamatory "comments" that were posted by third-party Facebook users in response to the content. The appellants are media companies which publish newspapers that circulate in New South Wales or operate television stations, or both. Each appellant maintained a public Facebook page on which they posted content relating to news stories and provided hyperlinks to those stories on their website. After posting content relating to particular news stories referring to Mr Voller, including posts concerning his incarceration in a juvenile justice detention centre in the Northern Territory, a number of third-party Facebook users responded with comments that were alleged to be defamatory of Mr Voller. Mr Voller brought proceedings against the appellants alleging that they were liable for defamation as the publishers of those comments. The primary judge ordered that a question concerning the issue of publication, as agreed by the parties, be decided separately from the balance of the proceedings. The question was whether Mr Voller had "established the publication element of the cause of action of defamation against the defendant[s] in respect of each of the Facebook comments by third-party users". The Court of Appeal concluded that the primary judge did not err in answering that question in the affirmative. The High Court by majority dismissed the appeals and found that the appellants were the publishers of the third-party Facebook user comments. A majority of the Court held that the liability of a person as a publisher depends upon whether that person, by facilitating and encouraging the relevant communication, "participated" in the communication of the defamatory matter to a third person. The majority rejected the appellants' argument that for a person to be a publisher they must know of the relevant defamatory matter and intend to convey it. Each appellant, by the creation of a public Facebook page and the posting of content on that page, facilitated, encouraged and thereby assisted the publication of comments from third-party Facebook users. The appellants were therefore publishers of the third-party comments. +HIGH COURT OF AUSTRALIA Public Information Officer 27 April 2007 GEOFFREY JAMES BENNETT, JOHN EDGAR CHRISTIAN, BRUCE STANLEY WALKER, ANN MITCHELL WALKER, RICHARD AARON KLEINER AND ADMINISTRATION OF NORFOLK ISLAND v COMMONWEALTH OF AUSTRALIA The High Court of Australia today dismissed a challenge to the validity of Commonwealth legislation requiring voters and candidates in Norfolk Island elections to be Australian citizens. Norfolk Island became a territory of the Commonwealth in 1914 when it was placed under the authority of and accepted by the Commonwealth in accordance with section 122 of the Constitution. Norfolk Island has since been governed pursuant to a series of Commonwealth Acts. The Norfolk Island Act 1979 granted self-government and established the Legislative Assembly of Norfolk Island. The Norfolk Island Amendment Act 2004 amended the Norfolk Island Act to make Australian citizenship a necessary qualification for voting for, and standing for election to, the Legislative Assembly. Norfolk Island was occupied as a British penal settlement from 1788 to 1814 and from 1825 to 1855. When the second penal settlement was abandoned, the Imperial authorities set aside Norfolk Island for occupation by the inhabitants of Pitcairn Island, which lies between New Zealand and Chile. Pitcairn Island was settled in 1790 by mutineers from the Bounty and some Polynesian men and women. The 200 or so inhabitants of Pitcairn arrived on Norfolk Island in June 1856. From 1856 to 1914 Norfolk Island was administered by the Governor of New South Wales, although until 1897 the Governor administered Norfolk Island in his capacity as Governor of Norfolk Island. New South Wales took over the financial management of the island from 1897 to 1914. Since 1914, the Commonwealth has been responsible for financing the administration and the development and maintenance of infrastructure on Norfolk Island. The 2001 census recorded that Norfolk Island had a permanent population of 1574, of whom 82.5 per cent were Australian citizens and 14.1 per cent were New Zealand citizens, and that 48 per cent were descended from the Pitcairn Islanders. The Court held unanimously that the Norfolk Island Amendment Act was a valid exercise of the Commonwealth Parliament's power to make laws “for the government of any territory” under section 122 of the Constitution. The Court rejected the plaintiffs' contention that in providing for self-government of the island, Parliament was obliged to provide for a particular type of democratic representation that did not discriminate on the basis of Australian citizenship. Nothing in section 122 or elsewhere in the Constitution prohibited Parliament from granting electoral rights on the basis of Australian citizenship. +HIGH COURT OF AUSTRALIA 12 October 2022 BHP GROUP LIMITED v IMPIOMBATO & ANOR [2022] HCA 33 Today, the High Court dismissed an appeal from a judgment of the Full Court of the Federal Court of Australia. The sole issue in the appeal was a question of statutory construction: does Pt IVA of the Federal Court of Australia Act 1976 (Cth) permit representative proceedings to be brought on behalf of group members who are not resident in Australia? The appellant, BHP Group Limited ("BHP"), is the respondent in a representative proceeding (also known as a class action) commenced in the Federal Court of Australia under Pt IVA, relating to the failure of the Fundão Dam in Brazil in 2015. The group members in the representative proceeding are persons who, during a specified period before the Dam failure, contracted to acquire an interest in fully paid-up ordinary shares in BHP on the Australian Stock Exchange, BHP Billiton Plc ("BHP Plc") on the London Stock Exchange, and/or BHP Plc on the Johannesburg Stock Exchange, and who are alleged to have suffered loss by reason of BHP's conduct in contravention of the continuous disclosure obligations under the ASX Listing Rules and the Corporations Act 2001 (Cth). The group members are also alleged to have suffered loss as a result of BHP engaging in misleading or deceptive conduct contrary to the Australian Securities and Investments Commission Act 2001 (Cth) and the Corporations Act. This appeal was heard at an interlocutory stage of the representative proceeding. Part IVA does not contain any express geographic or territorial restriction on the identity of "persons" who can be group members in a representative proceeding. In this appeal, BHP argued that s 21(1)(b) of the Acts Interpretation Act 1901 (Cth) and the common law presumption against extraterritoriality apply so that Pt IVA must be interpreted as not permitting the inclusion of group members who are not resident in Australia. The High Court unanimously rejected this argument, holding that Pt IVA allows the inclusion of all persons as group members in a representative proceeding, irrespective of whether they are Australian residents, who have "claims" of the kind described in s 33C(1) of the Federal Court of Australia Act that are within the jurisdiction of the Federal Court. Part IVA is concerned with the powers and procedures of the Federal Court relating to the exercise of jurisdiction vested in it by other Commonwealth laws enacted under s 77(i) of the Constitution. The territorial connection of Pt IVA to Australia is direct and specific: it concerns the jurisdiction of the Federal Court of Australia. There is no basis to infer any further territorial limitation into Pt IVA. +HIGH COURT OF AUSTRALIA Manager, Public Information 9 December 2009 ICM AGRICULTURE PTY LTD ABN 32 006 077 765 & ORS THE COMMONWEALTH OF AUSTRALIA & ORS [2009] HCA 51 The replacement of a groundwater bore licence with an aquifer access licence which reduces a licensee’s groundwater entitlement does not constitute an acquisition of property, the High Court held today. In June 2004 the governments of the Commonwealth, New South Wales, Queensland, South Australia, Victoria, the Australian Capital Territory and the Northern Territory made an Intergovernmental Agreement on a National Water Initiative. In accordance with the agreement the Commonwealth established the National Water Commission, one of whose functions was to assist with the implementation of the National Water Initiative. The legislation establishing the Commission also provided that financial assistance could be awarded to particular projects relating to Australia’s water resources. Such financial assistance was awarded to a project proposed by NSW with respect to the Lower Lachlan Groundwater System (LLGS). NSW and the Commonwealth entered into a funding agreement for the project. Under the agreement NSW was required to convert all water licences in the LLGS granted under the Water Act 1912 (NSW) to water licences under the Water Management Act 2000 (NSW). It was required to achieve a reduction of 56 per cent in water entitlements in respect of the LLGS by 1 July 2016. ICM Agriculture Pty Ltd and the other plaintiffs conduct farming enterprises in the LLGS. Until 1 February 2008 the plaintiffs had a number of bore licences issued under the Water Act which enabled them to use bores to extract groundwater with which they irrigated their properties. On 1 February 2008 the plaintiffs’ bore licences were replaced by aquifer access licences issued under the Water Management Act which permitted the plaintiffs to take less water than the old bore licences had permitted. The plaintiffs commenced an action in the High Court, and stated a Special Case concerning the constitutional validity of the funding agreement, and of the various processes by which bore licences were converted to aquifer access licences and water entitlements were reduced, in accordance with the funding agreement. The plaintiffs argued that the replacement of their bore licences issued under the Water Act with aquifer access licences issued under the Water Management Act resulted in an acquisition of property for which just compensation was not paid. This, they argued, contravened s 51(xxxi) of the Constitution, which requires “just terms” to be paid on the acquisition of the property, and was therefore invalid. A component of this argument was that the legislative power of the Commonwealth to grant financial assistance to the States, contained in s 96 of the Constitution, by virtue of which they entered into the funding agreement, was subject to the “just terms” restriction contained in s 51(xxxi). Six of the seven justices of the High Court rejected the plaintiffs’ argument that the replacement of their bore licences was invalid. Chief Justice French and Justices Gummow and Crennan and Justice Heydon held that the legislative power of the Commonwealth conferred by s 96 and s 51(xxxvi) does not extend to the grant of financial assistance to a State on terms and conditions requiring the State to acquire property on other than just terms. The Court, with Justice Heydon dissenting, decided that however the licences may be characterised, there had been no acquisition of property. Acquisition implies that the “acquiring” person obtains some identifiable benefit or advantage relating to the ownership or use of the property. However, water is a natural resource and the State always had power to limit the volume of water to be taken from the resource. In reducing the volume of water to which the plaintiffs had access, the State gained no greater advantage than it already had to extract or permit others to extract water from the system. The State’s capacity to control the water resource was not enlarged by the reduction of the plaintiffs’ water entitlements via the processes provided for in the funding agreement between NSW and the Commonwealth. Having found that replacing the plaintiffs’ bore licences with aquifer access licences did not constitute an acquisition of property, the majority determined that it was unnecessary to answer any of the other questions in the Special Case, other than to state that the plaintiffs were no longer the holders of the bore licences issued to them under the Water Act and that they should pay the costs of the Special Case. +HIGH COURT OF AUSTRALIA Public Information Officer 12 December 2006 STATE OF NEW SOUTH WALES v DOROTHY ISABEL IBBETT The High Court of Australia today upheld awards of aggravated and exemplary damages against the State of New South Wales for trespass and assault by police officers. Mrs Ibbett, 75, brought action in the NSW District Court for damages for trespass and assault after two police officers pursued her son Warren into the garage of her home at Forster on the NSW central coast. Warren Ibbett, 48, lived with his mother after his release from prison in 1997 for armed robbery. About 2am on 23 January 2001, while Mrs Ibbett was asleep, Mr Ibbett arrived home in his van, pursued by an unmarked police car for speeding. He drove into the garage and closed the roller door with a remote control. As the door was closing, Senior Constable Darren Pickavance slid under it and with his service pistol drawn sought to arrest Mr Ibbett. Senior Constable Pickavance had no basis for making such an arrest or entering the property. Neither officer was in uniform but wore casual clothing. Mrs Ibbett was woken by the commotion. When she went into the garage Senior Constable Pickavance swung his gun towards her. He ordered Mrs Ibbett to open the garage door to let in Senior Constable Peter Harman. The officers handcuffed Mr Ibbett and pushed him down on to the driveway. His van was moved on to the driveway and searched. He was taken back into the garage and strip-searched. Criminal proceedings were commenced against Mr Ibbett but the charges were later dropped. Judge Colin Phegan found that the entry on to the property by Senior Constables Pickavance and Harman was without lawful justification and amounted to trespass. He held that Senior Constable Pickavance caused such an apprehension of harm on Mrs Ibbett’s part it amounted to assault. Judge Phegan awarded Mrs Ibbett $50,000 for trespass, made up of $10,000 in general damages, $20,000 in aggravated damages and $20,000 in exemplary damages, and $25,000 for assault, made up of $15,000 in general damages and $10,000 in exemplary damages. The Court of Appeal dismissed the State’s appeal and allowed Mrs Ibbett’s cross-appeal. It increased the total damages to $100,000 by awarding another $15,000 in exemplary damages and $10,000 in aggravated damages for the assault. The State appealed to the High Court. The Court unanimously dismissed the appeal. The State complained that by awarding general, aggravated and exemplary damages for trespass the State was punished twice for the same wrong. The High Court held that the Court of Appeal’s upholding of the award of aggravated damages was consistent with basic principle. It held that the Court of Appeal was mindful of the conceptual distinction between the compensatory nature of aggravated damages and the punitive and deterrent nature of exemplary damages. The Court of Appeal was also aware of the dangers of an excessive overall award where factors supporting one head of damages could also support the other. +HIGH COURT OF AUSTRALIA 18 April 2018 GARRY BURNS v TESS CORBETT & ORS; GARRY BURNS v BERNARD GAYNOR & ORS; ATTORNEY GENERAL FOR NEW SOUTH WALES v GARRY BURNS & ORS; ATTORNEY GENERAL FOR NEW SOUTH WALES v GARRY BURNS & ORS; STATE OF NEW SOUTH WALES v GARRY BURNS & ORS [2018] HCA 15 Today the High Court unanimously dismissed five appeals from the Court of Appeal of the Supreme Court of New South Wales. The High Court held that ss 28(2)(a) and (c), 29(1) and 32 of the Civil and Administrative Tribunal Act 2013 (NSW) were invalid to the extent that they purported to confer jurisdiction upon the Civil and Administrative Tribunal of New South Wales ("NCAT") in relation to matters between residents of different States. Within Ch III of the Constitution, which establishes the federal Judicature, s 75(iv) relevantly provides that the High Court shall have original jurisdiction in all matters between residents of different States. Section 76 of the Constitution empowers the Commonwealth Parliament to confer additional original jurisdiction on the High Court to determine other kinds of matters. Section 77 of the Constitution empowers the Commonwealth Parliament to make laws defining the jurisdiction of any federal court other than the High Court, defining the extent to which the jurisdiction of any federal court shall be exclusive of that which belongs to or is invested in the courts of the States, and investing State courts with federal jurisdiction. In 2013 and 2014, Mr Garry Burns made separate complaints to the Anti-Discrimination Board of New South Wales about statements made by Ms Therese Corbett and Mr Bernard Gaynor, which Mr Burns claimed were contrary to s 49ZT of the Anti-Discrimination Act 1977 (NSW) ("the AD Act"). At all material times, Mr Burns was a resident of New South Wales, Ms Corbett was a resident of Victoria and Mr Gaynor was a resident of Queensland. The complaint against Ms Corbett was referred to the Administrative Decisions Tribunal of New South Wales ("the ADT"), which was the predecessor to NCAT. The ADT found that Ms Corbett had breached the AD Act and ordered her to make a public and private apology. She appealed unsuccessfully to the Appeal Panel of NCAT, whose orders were entered in the Supreme Court of New South Wales. Thereafter, Mr Burns brought proceedings in the Supreme Court charging Ms Corbett with contempt for failing to make either apology. That aspect of Ms Corbett's defence that contended that neither the ADT nor the Appeal Panel of NCAT had jurisdiction because she was a resident of Victoria was removed to the Court of Appeal. Mr Burns' complaint against Mr Gaynor was referred to and dismissed by NCAT. While an appeal by Mr Burns to the Appeal Panel of NCAT was yet to be heard, an interlocutory costs order was made against Mr Gaynor. Mr Gaynor obtained leave to appeal to the Court of Appeal in respect of that order and by summons sought a declaration that NCAT had no jurisdiction to determine matters pertaining to residents in a State other than New South Wales. The Court of Appeal heard the appeals together in order to resolve the common issue of whether NCAT may hear and determine a dispute arising under the AD Act between a resident of New South Wales and a resident of another State. It was common ground before the Court of Appeal that even though, in hearing and determining Mr Burns' complaints, NCAT was exercising the judicial power of the State, NCAT was not a "court of a State" for the purposes of Ch III of the Constitution. The Court of Appeal held that NCAT had no jurisdiction to hear and determine the complaints against Ms Corbett or Mr Gaynor. By grant of special leave, the appellants appealed to the High Court. The Attorney-General of the Commonwealth filed a notice of contention in each matter, pursuant to which he submitted that there is an implied constitutional constraint on State legislative power, such that a State law is invalid to the extent that it purports to confer judicial power in respect of any of the matters identified in ss 75 and 76 of the Constitution on a person or body that is not one of the "courts of the States". The Attorney-General of the Commonwealth's alternative submission was that such a law is inconsistent with s 39(2) of the Judiciary Act 1903 (Cth) and thus invalid by operation of s 109 of the Constitution. The High Court unanimously dismissed the appeals. A majority of the Court did so on the basis that Ch III of the Constitution leaves no room for the possibility that adjudicative authority in respect of the matters in ss 75 and 76 might be exercised by, or conferred by any party to the federal compact upon, an organ of government, federal or State, other than a court referred to in Ch III. The constitutional structure erected by Ch III would be undermined were a State Parliament able to confer adjudicative authority with respect to any of the matters identified in ss 75 and 76 on a State tribunal that is not a State court. +HIGH COURT OF AUSTRALIA 13 June 2018 MILORAD TRKULJA (AKA MICHAEL TRKULJA) v GOOGLE LLC [2018] HCA 25 Today the High Court unanimously allowed an appeal from a decision of the Court of Appeal of the Supreme Court of Victoria that summarily dismissed a defamation proceeding brought by the appellant against the respondent. The appellant claimed that the respondent defamed him by publishing certain search results in response to particular searches made using the Google search engine, for example, "melbourne underworld crime". Those search results included images of the appellant mixed with images of convicted Melbourne criminals, text referring to the appellant, and predictions generated by Google's autocomplete functionality. The search results allegedly conveyed that the appellant was a "hardened and serious criminal in Melbourne", someone in the same league as, or an associate of, other named criminals, and "such a significant figure in the Melbourne criminal underworld that events involving him are recorded on a website that chronicles crime". The respondent brought an application seeking to set aside the proceeding and its service outside jurisdiction on three bases: first, that it had not published the allegedly defamatory search results; second, that the search results in issue were not defamatory of the appellant; and third, that it was entitled to immunity from suit. The primary judge rejected the application. On appeal, the Court of Appeal considered the first ground but ultimately found it unnecessary to decide that ground, and rejected the third ground. The Court of Appeal upheld the second ground on the basis that the search results were not capable of bearing the defamatory imputations pleaded. The Court of Appeal accordingly held that the appellant's proceeding had no real prospect of success and ordered that the proceeding, and its service, be set aside. By grant of special leave, the appellant appealed to the High Court. The Court held that it was evident that at least some of the search results complained of had the capacity to convey to an ordinary reasonable person viewing the search results that the appellant was somehow associated with the Melbourne criminal underworld, and, therefore, that the search results had the capacity to convey one or more of the defamatory imputations alleged. The Court of Appeal had therefore erred in concluding that the proceeding had no real prospect of success. The High Court also observed that the Court of Appeal had erred in making a purportedly determinative finding of mixed fact and law that a search engine proprietor, like the respondent, is a publisher of search results, but that an innocent dissemination defence will almost always be maintainable in a period before notification of an alleged defamation. Given the nature of the proceeding, there should not have been a summary determination of issues relating to publication or possible defences, at least until after discovery, and possibly at all. Accordingly, the appeal was allowed. +HIGH COURT OF AUSTRALIA 7 August 2013 LEO AKIBA ON BEHALF OF THE TORRES STRAIT REGIONAL SEAS CLAIM GROUP v COMMONWEALTH OF AUSTRALIA & ORS [2013] HCA 33 Today the High Court unanimously held that successive Commonwealth and Queensland legislation, which prohibited taking fish and other aquatic life for commercial purposes without a licence, did not extinguish the native title right of certain island communities in the Torres Strait to take resources from defined areas of water. The Court unanimously held that certain reciprocity-based rights between members of the island communities did not constitute native title rights and interests within the meaning of s 223 of the Native Title Act 1993 (Cth). The appellant, on behalf of 13 island communities in the Torres Strait, sought a determination of native title over a large part of the waters of the Torres Strait. In August 2010, the Federal Court of Australia made a native title determination which defined the native title rights held by each of the communities. In defining the native title rights and interests, the Federal Court rejected the appellant's claim that certain reciprocal rights, which arose out of personal relationships in Islander society, were rights in relation to land or waters and were thereby native title rights. The determination did, however, include the native title right to access and take for any purpose resources in the native title areas. One of the ways in which that right could be exercised was by the taking of fish or other aquatic life for commercial purposes. Throughout the litigation, the appellant accepted that such commercial fishing could only be undertaken if any necessary statutory licences had been obtained. The first respondent appealed the Federal Court's decision to the Full Court of the Federal Court of Australia in relation to the native title right to access and take resources for any purpose. The appellant cross-appealed in relation to the conclusion that reciprocal rights were not native title rights and interests. The Full Court allowed the first respondent's appeal, holding that successive Commonwealth and Queensland fisheries legislation had extinguished any native title right to take fish and other aquatic life for commercial purposes. The Full Court rejected the appellant's cross-appeal, holding that the reciprocal rights were not rights in relation to the waters the subject of the native title determination. By special leave, the appellant appealed to the High Court. The High Court held that the successive statutory regimes which prohibited commercial fishing without a licence were not inconsistent with the continued existence of the native title right to access and take for any purpose resources in the native title areas. The Court also held that the claimed reciprocal rights were rights of a personal character dependent upon status. Such rights were not rights in relation to the waters the subject of the native title determination. +HIGH COURT OF AUSTRALIA Public Information Officer 15 June 2006 ISLAND MARITIME LIMITED v BARBARA FILIPOWSKI SACHIN KULKARNI v BARBARA FILIPOWSKI The owner and master of a ship which allegedly discharged oil into Botany Bay were not subjected to double jeopardy when they faced a second trial over the incident, the High Court of Australia held today. The Pacific Onyx, owned by Island Maritime and under master Sachin Kulkarni, allegedly dropped the oil on 14 November 1999. In February 2002, prosecutor Barbara Filipowski, the general counsel with the Sydney Ports Authority, filed two summonses in the New South Wales Land and Environment Court alleging a contravention by both Island Maritime and Mr Kulkarni of section 27 of the NSW Marine Pollution Act covering discharges during transfer operations. A trial proceeded 12 months later but Justice Angus Talbot dismissed the summonses, finding there was no case to answer. In November 2003, two further summonses were filed alleging that section 8, concerning discharge of oil from a ship into State waters, rather than section 27, had been contravened. Ms Filipowski intended to present the same evidence. The penalties were the same. Island Maritime and Mr Kulkarni, the appellants, sought a permanent stay of the second summonses on the grounds that proceedings were barred by the principles of autrefois acquit (formerly acquitted) or that the proceedings were an abuse of process. Justice Neal Bignold dismissed the application for a stay and the Court of Criminal Appeal dismissed an appeal. The appellants appealed to the High Court. The Court unanimously dismissed the appeals. It held that the possibility of a bar based on the principles of autrefois acquit does not exist because the appellants were never actually in jeopardy on the first set of summonses, so there was no double jeopardy in having to answer another charge. There could not have been a valid conviction because those summonses were defective. The appellants could have taken exception to the first summonses but did not. The charges in the second set of summonses were not the same as in the first set. Although the conduct of the case was unsatisfactory it did not amount to an abuse of process. +HIGH COURT OF AUSTRALIA 8 December 2011 DALE CHRISTOPHER HANDLEN v THE QUEEN DENNIS PAUL PADDISON v THE QUEEN [2011] HCA 51 Today the High Court allowed appeals by Dale Christopher Handlen and Dennis Paul Paddison against the decision of the Court of Appeal of the Supreme Court of Queensland, which had upheld each appellant's convictions for several drug-related offences under the Criminal Code (Cth) ("the Code"). The appellants were each charged with multiple drug-related offences under the Code, including two counts of importing a commercial quantity of border controlled drugs into Australia contrary to s 307.1 of the Code. At trial, the appellants were each convicted of the charged offences. The trial was conducted on the mistaken assumption, shared by the parties and the trial judge, that guilt of the importation offences could be established by proof that the appellants were parties to a joint criminal enterprise or "group exercise" to import the drugs into Australia. At the date of the appellants' trial, participation in a joint criminal enterprise was not a basis for the attachment of criminal responsibility respecting a substantive offence under the laws of the Commonwealth. The only basis upon which criminal responsibility could be fixed on the appellants for the importations was under s 11.2 of the Code for aiding, abetting, counselling or procuring the offences of Matthew Reed, who had pleaded guilty and been convicted in relation to the same importations before the appellants' trial. The appellants appealed against their convictions to the Court of Appeal. The Court of Appeal found that the jury had been misdirected as to the basis of the appellants' liability for the importation offences, but that the misdirection had not involved a fundamental departure from a trial according to law. The Court of Appeal was satisfied that the appellants' guilt had been established beyond reasonable doubt, which was a necessary although not sufficient condition for the application of the proviso in s 668E(1A) of the Criminal Code (Q). Section 668E(1A) provides that the Court of Appeal may dismiss an appeal, notwithstanding that it is of the opinion that the point or points raised by the appeal might be decided in the appellant's favour, if it considers that "no substantial miscarriage of justice has actually occurred". The appeals were dismissed under the proviso. The appellants appealed to the High Court upon a single ground challenging the application of the proviso. The appellants submitted that s 80 of the Constitution, which requires a "trial by jury" for Commonwealth offences tried on indictment, is inconsistent with the exercise by an appeal court of the power under the proviso in a case in which there has been a misdirection as to the elements of liability. By majority, the High Court upheld each of the appellants' appeals against conviction, and ordered a new trial. The Court considered that the prosecution of the appellants for the importation offences upon a basis that was not known to law was a fundamental departure from the proper conduct of the trial, which denied application of the proviso. The verdicts on the importation counts reflected the jury's satisfaction that each appellant was a party to the "group exercise", but it did not follow that the jury must have been satisfied of the facts necessary to establish the appellants' guilt on the basis of their having aided, abetted, counselled or procured Mr Reed's offences. Consequently, the Court found it unnecessary to address the appellants' constitutional arguments. +HIGH COURT OF AUSTRALIA 2 February 2011 MINISTER FOR IMMIGRATION & CITIZENSHIP v SZGUR & ANOR [2011] HCA 1 The Refugee Review Tribunal ("RRT") made adverse credibility findings against SZGUR, an applicant for a protection visa. The RRT sent SZGUR a letter inviting him to comment on or respond to inconsistencies and contradictions in information provided by him during the application and review process. In response, SZGUR's migration agent suggested that the inconsistencies and contradictions may have been related to SZGUR's alleged mental health problems, and requested that the RRT "arrange independent assessment of [SZGUR's] mental health, if required". Section 427(1)(d) of the Migration Act 1958 (Cth) relevantly empowered the RRT to require the Secretary to arrange for a medical examination. The RRT did not request the arranging of any such examination, and refused SZGUR's visa application. An application to the Federal Magistrates Court for judicial review was dismissed. On appeal to the Federal Court, Rares J held that the RRT failed to consider the agent's request, and that this failure constituted a constructive failure to exercise jurisdiction. Today the High Court allowed an appeal by the Minister against the decision of the Federal Court. It held that there was insufficient evidence to indicate that the RRT had failed to consider the agent's request. The Court also took the view that the relevant provisions of the Act did not create any general obligation on the RRT to consider whether to exercise the power in s 427(1)(d). Pursuant to an undertaking given to the Court, the Minister was ordered to pay SZGUR's costs. +HIGH COURT OF AUSTRALIA 7 September 2012 PLAINTIFF S10/2011 v MINISTER FOR IMMIGRATION AND CITIZENSHIP & ANOR; KAUR v MINISTER FOR IMMIGRATION AND CITIZENSHIP & ANOR; PLAINTIFF S49/2011 v MINISTER FOR IMMIGRATION AND CITIZENSHIP & ANOR; PLAINTIFF S51/2011 v MINISTER FOR IMMIGRATION AND CITIZENSHIP & ANOR [2012] HCA 31 Today the High Court dismissed four applications for declaratory relief and the issue of certiorari and constitutional writs against the Minister for Immigration and Citizenship ("the Minister") and the Secretary for the Department of Immigration and Citizenship. Sections 48B, 195A, 351 and 417 of the Migration Act 1958 (Cth) ("the Act") confer powers upon the Minister to intervene with respect to the granting of visas under the Act. The powers may only be exercised by the Minister personally and the Minister cannot be compelled to exercise them. Each of the plaintiffs in these proceedings had sought to invoke the exercise by the Minister of those powers after having unsuccessfully sought merits review and, in some cases, judicial review of a decision to refuse to grant a visa. The plaintiffs submitted that in deciding whether or not to consider the exercise of the relevant powers, or deciding whether or not to exercise those powers, the Minister was obliged to afford natural justice or procedural fairness to the plaintiffs. The High Court dismissed the plaintiffs' applications, finding that the distinct nature of the powers conferred on the Minister by the Act meant that the exercise of the powers is not conditioned on the observance of the principles of procedural fairness. +HIGH COURT OF AUSTRALIA 23 May 2007 Public Information Officer LOCKWOOD SECURITY PRODUCTS PTY LTD v DORIC PRODUCTS PTY LTD A deadlock in which a key unlocks the door from the inside as well as the outside so that people are not trapped in their homes involves an inventive step, the High Court of Australia held today. Lockwood filed its patent application in 1997 and the patent was registered in 2000. Lockwood said the new deadlock overcame the problems with existing deadlocks, which have a second key- operated lock inside which must be unlocked separately to exit the house. These were designed to prevent the removal of large items during a break-in. However if a key is left in the outside lock or misplaced without the inner lock being unlocked, occupants could be trapped inside. The patent is for a deadlock where the key would open the outside and inside locks simultaneously. In 2000 Lockwood’s lawyers accused Doric (and others) of infringing the patent. Doric commenced proceedings in the Federal Court of Australia alleging Lockwood had made unjustified threats of legal action. Lockwood cross-claimed for infringement of 19 of the 33 patent claims. Doric in turn cross-claimed seeking revocation of the patent, alleging the patented lock was not novel, that it was obvious and involved no inventive step, that the specification did not fully describe the invention, that certain claims were unclear, and that none of the claims were fairly based on the matters described in the patent specification. Justice Peter Hely rejected all Doric’s claims except lack of fair basis. He consequently found that 32 of Lockwood’s claims were invalid and ordered they be revoked and dismissed Lockwood’s cross-claim alleging infringement but granted a stay of judgment. The Full Court dismissed an appeal, upholding Justice Hely’s conclusion that claims 1 to 32 were invalid. Lockwood appealed to the High Court. In 2004, the Court unanimously allowed the appeal, declared that the patent claims were fairly based on the specification, and remitted the matter to the Full Court of the Federal Court to resolve remaining issues. In 2005, the Full Court dismissed the remainder of Lockwood’s appeal. It revoked many of the patent claims due to a lack of an inventive step with some claims also removed for lack of novelty. Lockwood again appealed to the High Court in respect of the Full Court’s reasons dealing with obviousness or lack of inventive step. The High Court unanimously allowed the second appeal. It held that the addition of a lock-release mechanism to unlock the deadlock inside a house (expressed in claim 1 as adding an extra integer to five existing integers of a deadlock) was an inventive step, as it was not obvious to a person skilled in the relevant art, namely lock design. The combination of integers was not common general knowledge, and it was also relevant that no-one had previously arrived at a solution to the known problem that inspired Lockwood’s design. The Court set aside various orders of the Full Court, including the order that certain claims of the patent be revoked for lack of inventive step. The High Court ordered that Doric be restrained from infringing particular claims of the patent and from supplying a latch assembly with the particular features of Lockwood’s deadlock. Issues still to be finalised were remitted to the Full Court. +HIGH COURT OF AUSTRALIA Manager, Public Information 21 April 2010 HEALTH WORLD LTD v SHIN-SUN AUSTRALIA PTY LTD [2010] HCA 13 to cancel the registration of Shin-Sun's In 2006 Health World Ltd ("Health World") initiated proceedings against Shin-Sun Australia Pty trade mark Ltd ("Shin-Sun") seeking either "HEALTHPLUS" under s 88 of the Trade Marks Act 1995 (Cth) ("the Act") or to remove the mark from the Register under s 92 of the Act. To have standing to do so, Health World had to demonstrate that it was "aggrieved" within the meaning of ss 88(1) and 92(1). Today the High Court held that, for an applicant to be "aggrieved", it is not necessary to show that the applicant intends to use the trade mark or that it will be appreciably disadvantaged in a legal or practical sense by its continuing registration. It is sufficient that the applicant and the proprietor of the mark are rivals in relation to the goods to which the mark applies. Health World is the manufacturer and supplier of a probiotic powder, marketed under the name "Inner Health", and of probiotic capsules, marketed under the name "Inner Health Plus". Shin-Sun is the manufacturer and supplier of health supplements, marketed under the name "HealthPlus". On 7 May 2001 Shin-Sun applied for registration of "HEALTHPLUS" as a trade mark and on 12 September 2001 Health World applied for registration of "INNER HEALTH PLUS". Each opposed the registration of the other's trade mark. Health World's opposition failed, Shin-Sun later withdrew its opposition, and both marks were entered on the Register in February 2005. Health World commenced three separate proceedings against Shin-Sun in the Federal Court of Australia. Only the first two were relevant to today's decision. In the first proceeding, Health World claimed to be an "aggrieved person" under s 88(1) of the Act and sought cancellation of the registration of "HEALTHPLUS" pursuant to that provision. It did so on the ground, among others, that Shin-Sun did not intend to use the mark "HEALTHPLUS" in Australia. In the second proceeding, Health World contended that it was a "person aggrieved" under s 92(1) of the Act and sought removal of the "HEALTHPLUS" mark from the Register pursuant to that provision. It claimed that when Shin-Sun filed its application for registration it lacked an intention in good faith to use the mark in Australia (s 92(4)(a)(i)), and that the mark had not been used in the three years preceding Health World's application for removal (s 92(4)(b)). The primary judge found that Health World was not an "aggrieved person" under s 88(1) of the Act and was not a "person aggrieved" under s 92(1) of the Act. In both proceedings, Health World appealed to the Full Court of the Federal Court, contending that it did have standing under ss 88(1) and 92(1) of the Act. The Full Court dismissed both appeals. On 31 July 2009 Health World was granted special leave to appeal to the High Court in both proceedings. Having considered the subject, scope and purpose of the Act, the Court held that the word "aggrieved" was to be liberally construed. The Full Federal Court had held that, for an applicant to be "aggrieved", there must be a reasonable possibility of the applicant being "appreciably disadvantaged in a legal or practical sense" by the trade mark remaining on the Register. The High Court held that the Full Court had erred in treating this as an exhaustive test. For an applicant to be "aggrieved" it is not necessary to show that the applicant desires or intends to use the mark or could use the mark. It is sufficient to demonstrate that the proprietor of the mark and the applicant are rivals in relation to the goods to which the mark applies. In this case, Health World and Shin-Sun were rivals in selling the health products in question. They were in the same trade, and they each traded in the class of goods in respect of which the challenged mark was registered. The High Court allowed both appeals and remitted the matter to the Full Court for determination of the remaining issues. +HIGH COURT OF AUSTRALIA 16 June 2004 GEORGE PETER OSTROWSKI v JEFFREY RYDER PALMER Ignorance of the law would not enable a Western Australian commercial rock lobster fisherman to escape a conviction for fishing in a prohibited area, the High Court of Australia held today. Mr Palmer was charged by Mr Ostrowski, a Fisheries WA officer, with fishing for rock lobster in a marine life protection zone on the Point Quobba reef near Carnarvon in February 1999. In November 1998, when he was planning to revert to lobster fishing from line fishing, Mr Palmer twice visited the Fremantle office of Fisheries WA to obtain regulations covering Zone B of the WA fishing grounds. On the second visit, when copies of regulations for the 1998-99 lobster season were still unavailable, a staff member photocopied the office copy of the documentation, but these did not include the relevant regulations. Mr Palmer set 54 lobster pots in the exclusion zone near Point Quobba, believing his licence entitled him to do so. Fisheries officers observing him checking and setting pots did not point out his error. A Carnarvon magistrate held that any mistake by Mr Palmer was a mistake of law, not a mistake of fact, so he could not rely on the defence in section 24 of the WA Criminal Code. Section 24 provides that a person who does an act under an honest and reasonable but mistaken belief in the existence of the state of things is not criminally responsible. Section 22 provides that ignorance of the law does not afford an excuse for an act or omission which constitutes an offence. The offence created by the regulation had three elements: holding a commercial fishing licence; fishing for rock lobsters; and doing so in waters around Quobba Point. Mr Palmer satisfied all three elements and held no erroneous belief about any element. What he did not know was that there was a regulation prohibiting his conduct. His conviction resulted in a mandatory penalty of $26,700, a $500 fine and $2,000 costs. Mr Palmer successfully appealed to the Full Court of the WA Supreme Court. Mr Ostrowski appealed to the High Court, undertaking to pay Mr Palmer’s costs. At the end of the appeal hearing Mr Ostrowski also undertook that the penalty, fine and costs would be remitted to Mr Palmer. The Court held that section 24 was not concerned with mistakes at large, particularly whether there was a law against certain conduct. Section 24 applies to mistakes about the elements of the offence, not mistakes about the existence of the law creating the offence. The Court held that the case fell within section 22, not section 24. It allowed the appeal and reinstated Mr Palmer’s conviction. +HIGH COURT OF AUSTRALIA 21 June 2017 JOHN RIZEQ v THE STATE OF WESTERN AUSTRALIA [2017] HCA 23 Today the High Court unanimously dismissed an appeal from the Court of Appeal of the Supreme Court of Western Australia. The appellant had been convicted on two charges in the District Court of Western Australia of offences against s 6(1)(a) of the Misuse of Drugs Act 1981 (WA) ("the MDA"). Although a jury of 12 persons had been unable to reach a unanimous verdict on either charge at trial, the decisions of 11 of the 12 jurors were taken by the District Court to be verdicts of guilty under s 114(2) of the Criminal Procedure Act 2004 (WA) ("the CPA"), and the District Court convicted the appellant of both offences accordingly. The appellant was at all relevant times a resident of New South Wales. His trial involved a matter "between a State and a resident of another State" within the meaning of s 75(iv) of the Constitution and the District Court therefore was exercising federal jurisdiction under s 39(2) of the Judiciary Act 1903 (Cth). The appellant's appeal to the Court of Appeal was dismissed and, by grant of special leave, the appellant appealed to the High Court. The appellant argued that, because the District Court was exercising federal jurisdiction in his trial, Western Australian law was incapable of valid application to the determination of his criminal liability in that trial. The appellant argued that s 6(1)(a) of the MDA could not and did not apply as a law of Western Australia, and instead was picked up and applied as a law of the Commonwealth by operation of s 79 of the Judiciary Act. The result, the appellant argued, was that his trial was a trial on indictment of offences against a law of the Commonwealth for which, by operation of s 80 of the Constitution, the verdicts of the jury were required to be unanimous. Dismissing the appeal, the High Court held that the appellant's trial was of offences against a law of a State, not of offences against a law of the Commonwealth, and that s 80 therefore had no application. Section 79 of the Judiciary Act was not needed, and was not engaged, to pick up and apply s 6(1)(a) of the MDA as a law of the Commonwealth. The plurality held that, as a State Parliament is incapable of commanding a State court as to the manner of its exercise of federal jurisdiction, s 79 is a law enacted by the Commonwealth Parliament to ensure that the exercise of federal jurisdiction is effective. Section 79 fills a gap in the law governing the exercise of federal jurisdiction which exists by reason of the absence of State legislative power, by picking up the text of a State law governing the exercise of State jurisdiction and applying that text as Commonwealth law to govern the manner of exercise of federal jurisdiction. Its operation is limited to making the text of State laws of that nature apply as Commonwealth law to bind a court in the exercise of federal jurisdiction. Section 79 has no broader operation. Section 6(1)(a) of the MDA is a law squarely within State legislative competence and is outside the operation of s 79. The High Court further held that s 79 was needed and was engaged to pick up and apply the text of s 114(2) of the CPA as a law of the Commonwealth. Section 114(2) of the CPA, being a law governing what is to be taken to be the verdict of a jury, can apply to a Western Australian court only when that court is exercising Western Australian jurisdiction. Section 79 operated to apply the text of s 114(2) as a Commonwealth law to a Western Australian court when that court was exercising federal jurisdiction, except as otherwise provided by the Constitution or by some other Commonwealth law. +HIGH COURT OF AUSTRALIA Public Information Officer 3 September, 2003 WHISPRUN PTY LIMITED v SONYA LEA DIXON The High Court of Australia today upheld the decision of a New South Wales Supreme Court trial by Justice Peter Newman who rejected Ms Dixon’s claim that she was suffering from chronic fatigue syndrome as a result of Q fever infection. Ms Dixon contracted Q fever in April 1994 while working at an Inverell, NSW, abattoir where her job was to suck foetal blood into a pipette from unborn calves’ hearts. The disease lasted up to 10 months. Her employer, Whisprun, conceded it had breached a duty of care owed to her but denied she also developed chronic fatigue syndrome, as up to 20 per cent of Q fever sufferers do. Between 1994 and the 2000 trial Ms Dixon saw numerous doctors complaining of headaches, aches and pains, nausea and fatigue. Justice Newman held that Ms Dixon lacked credibility after surveillance videos, photographs and her own testimony showed inconsistencies between what she said she could do and what she did do. There was evidence of brisk walking around Sydney, attending race meetings, riding horses, driving cars and horse floats, riding a jet ski and an inner tube towed by a speedboat, and drinking and dancing at a wedding. Ms Dixon claimed her mood swings caused the break-up of her relationship but there was evidence it had continued. Because the damages which Justice Newman would otherwise have awarded fell short of the statutory threshold that the NSW Workers Compensation Act then provided, he dismissed Ms Dixon’s action with costs. The NSW Court of Appeal unanimously held that Justice Newman had not given weight to medical reports about Ms Dixon’s condition and erred in concluding that it rested on subjective symptoms when there were objective symptoms such as weight loss, abdominal tenderness, pallor and depression, and impaired memory. The Court allowed Ms Dixon’s appeal and ordered a new trial. Whisprun appealed to the High Court. The High Court, by a 3-2 majority, allowed Whisprun’s appeal and restored the judgment of the trial judge. The majority held that, having regard to the way the trial was conducted, Justice Newman had not overlooked relevant evidence and there was no miscarriage of justice. +HIGH COURT OF AUSTRALIA 6 April 2016 MOK v DIRECTOR OF PUBLIC PROSECUTIONS (NSW) [2016] HCA 13 Today the High Court dismissed an appeal from the Court of Appeal of the Supreme Court of New South Wales. The High Court held that by operation of s 89(4) of the Service and Execution of Process Act 1992 (Cth) ("SEPA") the appellant could be found guilty of the offence of attempting to escape lawful custody under s 310D of the Crimes Act 1900 (NSW) ("Crimes Act"). SEPA provides for the execution throughout Australia of warrants authorising the apprehension of persons under State laws. Under SEPA, a person named in a warrant issued in one State may be apprehended in another State and taken before a magistrate in that State. On production of the warrant, the magistrate in the second State must make an order under s 83(8) of SEPA for the person to be remanded on bail to appear in, or be taken in custody to, the State in which the warrant was issued. Section 89(4) of SEPA relevantly provides that the law relating to the liability of a person who escapes from lawful custody which is in force in the place of issue of the warrant will apply to a person taken to the place of issue of the warrant in compliance with an order under s 83(8). In 2003, the appellant was arrested and charged in New South Wales with fraud offences to which he pleaded guilty. He was committed to the District Court of New South Wales for sentence on 13 April 2006 but did not appear. A warrant was issued for his apprehension ("the NSW warrant"). Several years later the appellant was arrested in Victoria and charged with separate offences. After being sentenced for those offences, the appellant was arrested by a Victorian police officer pursuant to the NSW warrant. The following day a Victorian magistrate made an order under s 83(8)(b) of SEPA requiring the appellant be taken in custody to New South Wales. While in custody at Tullamarine Airport, the appellant escaped and was re-arrested a short time later. Once in New South Wales, he was charged under s 310D of the Crimes Act, which makes it an offence for an "inmate" to escape or attempt to escape from lawful custody. At first instance the magistrate dismissed the charge against the appellant, finding that while s 310D was applicable by virtue of s 89(4) of SEPA, the appellant was not an "inmate" within the meaning of that term in s 310D of the Crimes Act. The Supreme Court of New South Wales overturned the magistrate's decision. The appellant appealed to the Court of Appeal. In dismissing the appeal, the Court of Appeal held that s 89(4) of SEPA acted on s 310D of the Crimes Act to create a new federal offence which applied to all persons being taken to New South Wales in compliance with a relevant order under SEPA. Section 89(4) applied s 310D of the Crimes Act in an altered form, meaning it was not relevant whether the appellant was an "inmate" within the scope of s 310D in its ordinary operation as an offence under State law. By grant of special leave, the appellant appealed to the High Court. The High Court dismissed the appeal, holding that s 89(4) of SEPA applied s 310D of the Crimes Act as federal law and, by majority, that it was not a requirement of the federal offence that the appellant answer the description of "inmate". +HIGH COURT OF AUSTRALIA 10 August 2022 FARM TRANSPARENCY INTERNATIONAL LTD & ANOR v STATE OF NEW SOUTH WALES [2022] HCA 23 Today, the High Court answered questions stated in a special case concerning whether ss 11 and 12 of the Surveillance Devices Act 2007 (NSW) ("the SD Act") were invalid on the ground that they impermissibly burdened the freedom of political communication implied by the Constitution ("the implied freedom"). Part 2 of the SD Act regulates the installation, use and maintenance of surveillance devices. Section 8 relevantly prohibits the knowing installation, use and maintenance of optical surveillance devices on or within premises to record visually or to observe the carrying on of an activity if the installation, use or maintenance of the device involves trespass to the premises. Sections 11 and 12 prohibit, respectively, the communication or publication of a record or report, and the possession of a record, of the carrying on of an activity obtained as a direct or indirect result of, relevantly, the use of an optical surveillance device in contravention of s 8. The first plaintiff, a not-for-profit charity, had agitated and advocated for political and legal changes to animal agricultural practices and animal welfare standards with the objective of ending modern farming and slaughtering practices. It had engaged in the publication of photographs, videos and audio-visual recordings of animal agricultural practices in New South Wales. The second plaintiff, a director of the first plaintiff, had obtained recordings of the farming or slaughter of animals through purported acts of trespass contrary to s 8 of the SD Act. The plaintiffs argued that ss 11 and 12 of the SD Act impermissibly burdened their ability to publish information, including video recordings, that showed animal cruelty practices. The special case was presented on the basis that the activities recorded, albeit cruel, were not established to be unlawful. The High Court, by majority, held that ss 11 and 12 of the SD Act did not impermissibly burden the implied freedom in their application to, respectively, the communication or publication by a person of a record or report, or the possession by a person of a record, of the carrying on of a lawful activity, at least where the person was complicit in the record or report being obtained exclusively by breach of s 8 of the SD Act. It was otherwise unnecessary to determine whether ss 11 and 12 burdened the implied freedom in other applications. Assessed by reference to the restraints which the common law, equity and statute law already imposed upon the existing liberty of political communication, ss 11 and 12 imposed an incremental burden on a person's ability to publish records of lawful activities obtained surreptitiously and by conduct which amounted to trespass. The provisions had a legitimate purpose of the protection of privacy. The statutory schemes of other Australian jurisdictions were not obvious and compelling alternatives, as they did not pursue the same purpose and were broader in application. Sections 11 and 12 achieved an adequate balance between the benefit they sought to achieve and the adverse effect on the implied freedom. As ss 11 and 12 were valid in their relevant application, it was unnecessary to answer the questions stated in the special case regarding their severance or partial disapplication in that application. +HIGH COURT OF AUSTRALIA 27 May 2004 APPLICANT S v MINISTER FOR IMMIGRATION AND MULTICULTURAL AFFAIRS The High Court of Australia today allowed an appeal by a young Afghan man who resisted being drafted into the Afghanistan army and who had been refused a protection visa. S, an Afghan of Pashtun ethnicity, arrived by boat in 2000 aged 20, leaving behind his wife, parents and four brothers. The Taliban practised ad hoc, random and forced recruitment of able- bodied young men and thousands of young men left Afghanistan to avoid recruitment. The Taliban tried twice to forcibly recruit S for military service. The first time he paid off the recruiters, and on the second occasion S told them he needed to speak to his parents first, then immediately departed from Afghanistan with the assistance of a people smuggler. The Immigration Department rejected his application for a protection visa, a decision affirmed by the Refugee Review Tribunal. In the Federal Court Justice Christopher Carr held that the RRT had not considered whether S was a member of a “particular social group” within the meaning of the Refugees Convention and whether he was persecuted due to membership of that group. He said the RRT should have considered whether able-bodied young men comprised a “particular social group”. Justice Carr remitted the matter to the RRT for redetermination, but the Minister successfully appealed to the Full Court of the Federal Court. S appealed to the High Court on the ground that the Full Court had erred in requiring evidence that Afghan society perceived young able-bodied men as comprising a “particular social group” before the RRT was obliged to consider whether S was a member of that group. The High Court upheld S’s submission that evidence of Afghan society’s perceptions would be relevant to the question of whether there was a “particular social group”, but absence of such evidence was not determinative. The Court held that a group could be a “particular social group” under the Convention if the group is identifiable by a characteristic common to all members, and if that characteristic distinguishes the group from society at large. The Minister also argued that recruitment by the Taliban did not amount to persecution as a critical element of enmity or malice was absent. The High Court held that persecution did not require this element. The Court also rejected the argument that conscription was a law of general application, as the conscription policy was ad hoc and random. Although the Taliban was in power at the time, the Court held it was not pursuing legitimate national objectives. The High Court, by a 4-1 majority, allowed the appeal with costs and remitted the matter to the RRT for redetermination according to the judgment, although the Court noted that the RRT will also have to take into account developments in Afghanistan since its original decision. +HIGH COURT OF AUSTRALIA 11 September 2019 DANIEL TAYLOR v ATTORNEY-GENERAL OF THE COMMONWEALTH [2019] HCA 30 Today the High Court published reasons for the answers given on 19 June 2019 to questions stated in a special case. A majority of the Court held that it was unnecessary to answer the substantive questions stated in the special case in order to determine the plaintiff's entitlement to relief, on the basis that s 268.121(2) of the Criminal Code (Cth) precludes the private prosecution of an offence against Div 268 of the Criminal Code. Under s 13(a) of the Crimes Act 1914 (Cth), any person may institute proceedings for the commitment for trial of a person in respect of an indictable offence against the law of the Commonwealth, unless the contrary intention appears in the Act creating the offence. Section 268.121(1) of the Criminal Code provides that proceedings for an offence against Div 268 of the Criminal Code must not be commenced without the written consent of the Attorney-General of the Commonwealth. Section 268.121(2) of the Criminal Code provides that an offence against Div 268 "may only be prosecuted in the name of the Attorney-General". On 16 March 2018, the plaintiff lodged a charge-sheet and draft summons at the Melbourne Magistrates' Court alleging that Aung San Suu Kyi, Minister for the Office of the President and Foreign Minister of the Republic of the Union of Myanmar, had committed a crime against humanity in contravention of s 268.11 of the Criminal Code, a Commonwealth indictable offence that appears in Div 268 of the Criminal Code and is unable to be heard and determined summarily. The plaintiff lodged the charge-sheet and draft summons in purported reliance on s 13(a) of the Crimes Act. Also on 16 March 2018, the plaintiff requested the consent of the Attorney-General of the Commonwealth under s 268.121(1) of the Criminal Code to the commencement of the prosecution. The Attorney-General did not consent to the prosecution. On 23 March 2018, the plaintiff commenced a proceeding against the Attorney-General in the original jurisdiction of the High Court, seeking to quash the decision not to consent to the commencement of the prosecution and to compel the Attorney-General to reconsider the request for consent. A majority of the Court held that, by providing that an offence against Div 268 of the Criminal Code "may only be prosecuted in the name of the Attorney-General", s 268.121(2) of the Criminal Code provides a contrary intention for the purpose of s 13(a) of the Crimes Act so as to preclude the private prosecution of an offence against that Division. Accordingly, a majority of the Court held that the decision made by the Attorney-General not to consent to the plaintiff's proposed prosecution of Ms Suu Kyi was the only decision legally open and that the relief sought by the plaintiff could only be refused. +HIGH COURT OF AUSTRALIA 13 May 2015 STATE OF QUEENSLAND v TOM CONGOO, LAYNE MALTHOUSE AND JOHN WATSON ON BEHALF OF THE BAR-BARRUM PEOPLE #4 & ORS [2015] HCA 17 Today the High Court dismissed an appeal from a decision of the Full Court of the Federal Court of Australia, which held that military orders made under reg 54 of the National Security (General) Regulations 1939 (Cth) in relation to certain land did not extinguish the first respondents' native title rights and interests that subsisted on the land. Section 5(1)(b)(i) of the National Security Act 1939 (Cth) provided for the making of regulations for securing the public safety and the defence of the Commonwealth, and in particular for authorising the taking of possession or control, on behalf of the Commonwealth, of any property. The Regulations were enacted pursuant to that section. Regulation 54(1) of the Regulations provided that if it appeared to the Minister of State for the Army to be necessary or expedient to do so in the interests of public safety, the defence of the Commonwealth or the efficient prosecution of the war, or for maintaining supplies and services essential to the life of the Commonwealth, the Minister could, on behalf of the Commonwealth, take possession of any land and give such directions as appeared necessary or expedient in connection with taking possession. Between 1943 and 1945, a delegate of the Minister made military orders under reg 54 with respect to land, which was thereafter used as an artillery range and a live fire manoeuvre range for the training of infantry and armoured units. The military orders directed a particular officer to occupy the land, authorised that officer to do anything in relation to the land that the holder of an estate in fee simple in the land could do by virtue of that interest, and prohibited all other persons from exercising "any right of way over the land or any other right relating thereto". The military orders ceased to take effect no later than six months after the war finished. The first respondents, the Bar-Barrum People, brought an application in the Federal Court for a determination of native title over land, part of which had been subject to the military orders. A Special Case was referred to the Full Court of the Federal Court setting out questions about the effect of the military orders on the native title rights and interests of the Bar-Barrum People. The parties accepted that, subject to the effect of the military orders, the Bar-Barrum People had native title rights and interests over the land. The Full Court of the Federal Court, by majority, held that the military orders did not have the effect of extinguishing the native title rights and interests. By grant of special leave, the appellant appealed to the High Court. Three Justices would have dismissed the appeal, holding that the military orders did not have the effect of extinguishing the native title rights and interests. Three Justices would have allowed the appeal, holding that by taking exclusive possession of the land under the military orders, the Commonwealth asserted rights which were inconsistent with, and thereby extinguished, the native title rights and interests. Where the High Court is equally divided in opinion, s 23(2)(a) of the Judiciary Act 1903 (Cth) provides that the decision appealed from shall be affirmed. Accordingly, the High Court ordered that the appeal be dismissed. This statement is not intended to be a substitute for the reasons of the High Court or to be used in any later consideration of the Court’s reasons. +HIGH COURT OF AUSTRALIA 10 April 2013 CASTLE CONSTRUCTIONS PTY LTD v SAHAB HOLDINGS PTY LTD & ANOR [2013] HCA 11 Today the High Court unanimously allowed an appeal from a decision of the Court of Appeal of the Supreme Court of New South Wales requiring the Registrar-General to restore to the Register an easement which the Registrar-General had deliberately removed. A majority of the Court held that the deliberate removal of an easement from the Register by the Registrar-General was not an "omission" of an easement within the meaning of s 42(1)(a1) of the Real Property Act 1900 (NSW) ("the Act"). That provision provides that, subject to some exceptions, a registered proprietor of land holds title free from all other estates and interests. One exception is in the case of the "omission" of an easement. In 1921, a parcel of land was subdivided and sold. As part of the transaction, an easement of way was created. The easement was in favour of what was to become known as the "Strathallen land", and burdened land that became known as the "Sailors Bay land". In 2001, the appellant, Castle Constructions, became the registered proprietor of the Sailors Bay land and requested that the Registrar-General remove the easement over its land from the Register. The Registrar-General notified the then registered proprietors of the Strathallen land that he intended to remove the easement from the Register. The registered proprietors did not object and, in November 2001, the Registrar-General removed the easement from the Register. In April 2007, the first respondent, Sahab, became the registered proprietor of the Strathallen land and in September 2008, it asked the Registrar-General to restore the easement to the Register. When the Registrar-General in the Supreme Court of New South Wales, seeking to have the easement restored. to do so, Sahab brought proceedings against the Registrar-General refused At first instance, the Supreme Court refused to compel the Registrar-General to restore the easement to the Register. Sahab appealed to the Court of Appeal, which allowed the appeal. The Court of Appeal held that the easement had been omitted from the Register within the meaning of s 42(1)(a1) of the Act because "omission" in that section meant simply that the easement was "not there". The Court of Appeal made orders requiring the Registrar-General to restore the easement to the Register. By special leave, Castle Constructions appealed to the High Court. The High Court allowed the appeal. A majority of the Court held that the easement had not been omitted from the Register because it had been deliberately removed by the Registrar-General. Because this was not a case of the omission of an easement within the meaning of the exception to indefeasibility in s 42(1)(a1) of the Act, Castle Constructions' title to the Sailors Bay land was not subject to the easement. +HIGH COURT OF AUSTRALIA 9 August 2017 COMMISSIONER OF TAXATION v KAMAL JAYASINGHE [2017] HCA 26 Today the High Court unanimously allowed an appeal from the Full Court of the Federal Court of Australia. The High Court held that the taxpayer, Mr Jayasinghe, was not entitled to exemption from taxation in the income years ended 30 June 2010 and 30 June 2011, as he was not a person who held an office in an international organisation within the meaning of s 6(1)(d)(i) of the International Organisations (Privileges and Immunities) Act 1963 (Cth) ("the IOPI Act"), and that the Commissioner of Taxation ("the Commissioner") was not bound to exempt Mr Jayasinghe from taxation by reason of s 357-60(1) of Sched 1 to the Taxation Administration Act 1953 (Cth) and Taxation Determination TD 92/153. Mr Jayasinghe was a qualified civil engineer. During the relevant income years, he was engaged by the United Nations Office for Project Services ("UNOPS"), an operational arm of the United Nations ("the UN"), as a "project manager" to build a 190 kilometre gravel road in Sudan. Mr Jayasinghe was engaged under an "Individual Contractor Agreement" to perform "specialist services" in recognition of his "skills and expertise". Under that agreement, he had the legal status of an independent contractor of UNOPS, had no authority or other right to enter into any legal or financial commitments or incur any obligations on behalf of UNOPS, was responsible for paying any tax levied by the Australian Government on his UNOPS earnings, and did not have the status of an official of the UN for the purposes of the Convention on the Privileges and Immunities of the United Nations [1949] ATS 3 ("the 1946 UN Convention"). From at least 1 May 2010, he was considered an expert on mission for the UN within the terms of s 22 in Art VI of the 1946 UN Convention. The Commissioner disallowed an objection lodged by Mr Jayasinghe to notices of amended assessment issued to him for his earnings from UNOPS. The Administrative Appeals Tribunal set aside the Commissioner's decision, concluding that the substance of the relationship between Mr Jayasinghe and UNOPS, and the obligations created and implemented in carrying out the project, were such that he held an office within the meaning of s 6(1)(d)(i) of the IOPI Act, and that he was an employee and entitled to the benefit of TD 92/153. A majority of the Full Court of the Federal Court dismissed an appeal by the Commissioner. Allowing the Commissioner's appeal, the plurality held that the phrase "a person who holds an office in an international organisation" in s 6(1)(d)(i) directed attention to the structure of the organisation and the person's place within it and was concerned with the incidents of the relationship between a person and an international organisation; which incidents depended on the substance of the terms upon which a person was engaged. The incidents of the relationship between Mr Jayasinghe and the UN were such that he did not "hold an office" within the meaning of s 6(1)(d)(i). Further, as Mr Jayasinghe was engaged by UNOPS as an expert, the Court unanimously held that he fell outside the scope of the phrase "person who holds an office" and that the Commissioner was not bound by TD 92/153 to exempt him from taxation. +HIGH COURT OF AUSTRALIA 12 November 2014 MINISTER FOR IMMIGRATION AND BORDER PROTECTION v SZSCA & ANOR [2014] HCA 45 Today the High Court, by majority, held that the Refugee Review Tribunal failed to properly address whether an applicant for a protection visa had a well-founded fear of persecution. The first respondent ("the respondent"), an Afghan citizen of Hazara ethnicity, arrived in Australia by boat on 21 February 2012. Before coming to Australia, the respondent had lived in Kabul with his family and worked as a self-employed truck driver transporting construction materials between Kabul and Jaghori. Around late January 2011, the respondent was stopped en route to Jaghori by the Taliban, who warned him not to carry construction materials. Thereafter, he took measures to avoid Taliban checkpoints, but continued to carry construction materials. In about November 2011, another truck driver showed the respondent a letter from the Taliban which called on "local council people to perform their Islamic duty ... to get rid of" the respondent. The respondent left Afghanistan 10 days later. The respondent's application for a protection visa was refused by a delegate of the Minister and that decision was affirmed by the Tribunal. The Tribunal accepted that, if the respondent was again intercepted by the Taliban on the roads on which he usually travelled, he would face a real chance of serious harm and even death for a reason specified in the Refugees Convention. However, the Tribunal found that the risk of persecution would only arise on these roads, which could be avoided by the respondent. It therefore concluded that the respondent did not satisfy the criteria for the grant of a protection visa. The Tribunal's decision was quashed by the Federal Circuit Court of Australia and an appeal from that Court was dismissed by a majority of a Full Court of the Federal Court of Australia. Both Courts considered that the Tribunal had committed the error identified by the High Court in Appellant S395/2002 v Minister for Immigration and Multicultural Affairs (2003) 216 CLR 473; [2003] HCA 71. By grant of special leave, the Minister appealed to the High Court. The High Court unanimously held that the Tribunal did not fall into the error identified in S395, but that the Tribunal was required to address whether it would be reasonable to expect the respondent to remain in Kabul and not drive trucks outside it. A majority of the Court held that the Tribunal had failed to address that question and was therefore unable to make a final determination as to whether the respondent had a well-founded fear of persecution. As this constituted an error of law, the Court dismissed the appeal. +HIGH COURT OF AUSTRALIA 7 April 2005 JOHN DAVID RICH v CGU INSURANCE LIMITED MARK ALAN SILBERMANN v CGU INSURANCE LIMITED The High Court of Australia today rescinded the special leave granted to Mr Rich and Mr Silbermann to appeal against a ruling that they were not entitled to insurance indemnity for their legal costs. Mr Rich and Mr Silbermann are former directors of One.Tel Ltd, which went into liquidation in 2001. Following an investigation into One.Tel’s collapse, the Australian Securities and Investments Commission instituted proceedings in the New South Wales Supreme Court relating to possible serious breaches of their duties as directors. Mr Rich and Mr Silbermann are also defending a claim by American Express in relation to credit card use. They have incurred substantial legal costs. CGU has refused to indemnify them for these costs under a Directors and Officers Liability Insurance Policy issued in 2000. CGU claimed an exclusion clause covering dishonesty and fraud by directors applied in relation to the alleged wrongful acts and that the insurance contract had been validly avoided under the Insurance Contracts Act because of both fraudulent non-disclosure and fraudulent misrepresentation. In the Supreme Court, Mr Rich and Mr Silbermann, along with former chairman John Greaves, sought orders that CGU advance the defence costs already incurred and to be incurred. Justice Peter McClellan found against them, answering three questions in favour of CGU. The Court of Appeal dismissed the appeals. Mr Rich and Mr Silbermann were then granted special leave to appeal to the High Court on the ground that the Court of Appeal had erred in upholding the affirmative answer to the first of the three questions. (Mr Greaves did not seek special leave.) Question 1 asked whether CGU could plead the exclusion clause as a defence to the claim for indemnity costs in the absence of a judgment, order or other final adjudication adverse to Mr Rich and Mr Silbermann. The High Court unanimously ordered that special leave be rescinded, with costs borne by Mr Rich and Mr Silbermann. The Court held that it was being asked to consider hypothetical issues, because Mr Rich and Mr Silbermann chose not to appeal against the answers to questions 2 and 3 which were favourable to CGU. If the High Court were to hold in answer to question 1 that CGU could not rely on the exclusion clause before a final judgment establishing fraud or dishonesty, this would still not produce the result sought by Mr Rich and Mr Silbermann, namely funds being advanced to them before the determination of defences raised by CGU. This is because, under question 2, CGU could always seek judgment in its favour in the Supreme Court and thereby refuse liability under the exclusion clause. This indicates that the strategy proposed by Mr Rich and Mr Silbermann was misconceived. They proposed that if the High Court answered question 1 in their favour they could seek summary judgment in the principal proceedings against CGU and a consequent order for defence costs to be advanced, postponing issues of dishonesty and fraud until final adjudication of a cross-claim by CGU based on the answer to question 2. The Court rejected this for two reasons. Issues raised in proceedings are not to be determined summarily except in the clearest of cases, and any order for advance payment, if made independently of an order determining CGU’s cross-claim, would offend the principles respecting avoidance of circularity of action in litigation. Accordingly, special leave to appeal was rescinded. +HIGH COURT OF AUSTRALIA Public Information Officer 21 February 2007 X AND Z AND Y v AUSTRALIAN PRUDENTIAL REGULATORY AUTHORITY AND Notices to two insurance company managers to show cause why they should not be disqualified from operating in Australia, which referred to evidence given by them to the HIH Royal Commission, were not in breach of the Royal Commissions Act, the High Court of Australia held today. Z, an international reinsurance business based in Germany, is authorised by the Australian Prudential Regulatory Authority to carry on business in Australia. X and Y are employed by Z as senior managers outside Australia. Z handed over documents to the Royal Commission into the 2001 collapse of the HIH Insurance Group and X and Y each travelled to Australia in 2002 to make statements and to give oral evidence. In 2005, Mr Godfrey, APRA’s senior manager, wrote to both X and Y seeking submissions on why APRA should not disqualify them under the Insurance Act as not being fit and proper people to act as managers or agents in Australia of a foreign insurer. The show cause notices stated that APRA’s preliminary view that each was not a fit and proper person was based on their involvement in certain arrangements entered into between Z and HIH, as revealed to the Royal Commission. X and Y’s lawyers responded that any decision to disqualify X and Y would be beyond power and would involve the commission of an offence under the Royal Commissions Act. They said such a decision would disadvantage X and Y, including the need to meet obligations to inform regulatory authorities in other countries. X, Y and Z instituted proceedings in the Federal Court of Australia to restrain APRA from taking action against X and Y pursuant to the Insurance Act and for a declaration that APRA did not have the power to disqualify X and Y. Justice Kevin Lindgren and the Full Court on appeal rejected their submission that APRA could only disqualify persons holding senior insurance positions in Australia. Justice Lindgren and the Full Court also rejected a submission that the use by APRA or Mr Godfrey of X and Y’s evidence to the HIH Royal Commission contravened section 6M of the Royal Commissions Act. The High Court granted X, Y and Z leave to appeal in regards to section 6M. This provides that any person who uses, causes or inflicts any violence, punishment, damage, loss or disadvantage to any person for or on account of the person having appeared as a witness before a royal commission or for any evidence given by him or her before a royal commission is guilty of an indictable offence which attracts a penalty of $1,000 or one year’s jail. Justice Lindgren concluded that neither APRA nor Mr Godfrey has caused or inflicted any disadvantage on X, Y or Z on account of X and Y having given evidence to the HIH Royal Commission. The High Court unanimously dismissed the appeal. It held that X, Y and Z could not demonstrate, within the meaning of section 6M, that APRA and Mr Godfrey proceeded “for or on account of” the appearance by X and Y at the HIH Royal Commission or any evidence they gave there. The Court held that Mr Godfrey was proceeding in discharge of the statutory powers of a regulatory nature conferred upon APRA by the Insurance Act. There was not the connection between X and Y’s attendance at the Royal Commission with the past or threatened conduct of Mr Godfrey or APRA that is captured by the expression “for or on account of”. Any disadvantage suffered by X or Y would not be “for or on account of” their attendance at the Royal Commission or the evidence they gave. Neither Mr Godfrey nor APRA victimised or proposed to victimise X, Y or Z in the sense required for the commission of an offence under section 6M. +HIGH COURT OF AUSTRALIA 4 August 2021 WORKPAC PTY LTD v ROSSATO & ORS [2021] HCA 23 Today, the High Court unanimously allowed an appeal from a judgment of the Full Court of the Federal Court of Australia. The appeal concerned the nature of casual employment. The first respondent ("Mr Rossato") was employed as a production worker by the appellant labour-hire company ("WorkPac") under a series of six contracts, or "assignments", to perform work for one of WorkPac's clients. At all relevant times, WorkPac treated Mr Rossato as a casual employee, such that Mr Rossato was not paid the leave and public holiday entitlements owed by employers to non-casual employees pursuant to the Fair Work Act 2009 (Cth) ("the Act") and the enterprise agreement which governed his employment. On 16 August 2018, judgment was delivered in WorkPac Pty Ltd v Skene (2018) 264 FCR 536. In that decision, it was held that Mr Skene, who had been employed by WorkPac in circumstances similar to those of Mr Rossato, was not a casual employee. In reliance on that decision, Mr Rossato claimed that he was likewise not a casual employee, and was therefore entitled to be paid for untaken annual leave, public holidays, and periods of personal leave and compassionate leave taken during his employment. WorkPac denied his claims and filed an originating application in the Federal Court seeking declarations that Mr Rossato had been a casual employee for the purposes of the Act and the relevant enterprise agreement. In the alternative, WorkPac claimed to be entitled to a set off, or to restitution, in respect of payments it had made to Mr Rossato in compensation for, or in lieu of, the entitlements claimed by Mr Rossato. The Full Court concluded that Mr Rossato was not a casual employee for the purposes of the Act and the enterprise agreement, and declared that Mr Rossato was entitled to the payments he claimed. The Full Court also rejected WorkPac's set off and restitution claims. By grant of special leave, WorkPac appealed to the High Court. The High Court held that a "casual employee" is an employee who has no firm advance commitment from the employer as to the duration of the employee's employment or the days (or hours) the employee will work, and provides no reciprocal commitment to the employer. Where parties commit the terms of their employment relationship to a written contract and thereafter adhere to those terms, the requisite firm advance commitment must be found in the binding contractual obligations of the parties; a mere expectation of continuing employment on a regular and systematic basis is not sufficient for the purposes of the Act. Mr Rossato's employment was expressly on an "assignment- by-assignment basis". Mr Rossato was entitled to accept or reject any offer of an assignment, and at the completion of each assignment WorkPac was under no obligation to offer further assignments. That Mr Rossato was to work in accordance with an established shift structure fixed long in advance by rosters did not establish a commitment to an ongoing employment relationship beyond the completion of each assignment. In carrying out each assignment, Mr Rossato worked as a casual employee for the purposes of the Act and the enterprise agreement. On that footing, it was unnecessary to consider WorkPac's set off and restitution claims. +HIGH COURT OF AUSTRALIA 13 October 2009 JOHN HOLLAND PTY LTD v VICTORIAN WORKCOVER AUTHORITY [2009] HCA 45 JOHN HOLLAND PTY LTD v INSPECTOR NATHAN HAMILTON & ANOR [2009] HCA 46 Today the HIGH COURT OF AUSTRALIA unanimously held that a company subject to a Commonwealth occupational health and safety law was liable to be prosecuted under State occupational health and safety laws for offences allegedly committed before the company became subject to the relevant Commonwealth law. Manager, Public Information In October 2006, John Holland Pty Ltd carried on the business of road construction and associated activities. At around this time John Holland was declared by the federal Minister to be eligible to be granted a licence under Part VIII of the Safety, Rehabilitation and Compensation Act 1988 (Cth) (the SRC Act). John Holland was granted a licence under the SRC Act which commenced on 1 January 2007. In March 2007, on the commencement of amendments to the Occupational Health and Safety Act 1991 (Cth) (the OHS Act) and by reason of John Holland’s licence under the SRC Act being in force, John Holland became a “non-Commonwealth licensee” and an “employer” as defined in the OHS Act. This had the effect that John Holland became subject to the occupational health and safety regime created by that Act. In September 2008 the Victorian Workcover Authority (the VWA) authorised Mr Andrew Gildea, an inspector appointed under the Occupational Health and Safety Act 2004 (Vic) (the Victorian Act), to prosecute John Holland for offences allegedly committed under the Victorian Act in October 2006. Mr Gildea arranged for a charge and summons to be issued against John Holland in the Magistrates Court in Victoria. In separate proceedings in the Industrial Court of New South Wales, John Holland was also charged with breaches of the Occupational Health and Safety Act 2000 (NSW) (the NSW Act), arising out of an incident occurring in New South Wales in October 2005. On 19 February 2009 John Holland commenced proceedings in the HIGH COURT OF AUSTRALIA seeking a declaration that the Victorian Act was invalid insofar as it purported to empower the VWA to authorise the prosecution. A case was stated for the consideration of the Full Court asking whether John Holland, once it became a “non-Commonwealth licensee” under the OHS Act, was liable for conviction under the Victorian Act for offences committed before John Holland became a “non-Commonwealth licensee”. John Holland argued that by virtue of section 109 of the Constitution, it could not be charged with offences under the Victorian Act in those circumstances. John Holland later applied to remove part of the proceedings in the Industrial Court of New South Wales to the High Court, insofar as they raised a similar issue to that raised in the Victorian-based litigation. In the matter concerning the Victorian Act the Court held that John Holland, whilst a “non-Commonwealth licensee” for the purposes of the OHS Act, was liable to be prosecuted under the Victorian Act. The Court held that there was no inconsistency between the Victorian Act and the Commonwealth law to the extent that the Victorian Act purported to empower VWA to authorise the bringing of proceedings against John Holland for offences committed before John Holland became a “non-Commonwealth licensee” under the OHS Act. In the second matter (John Holland v Inspector Nathan Hamilton) John Holland’s application for removal was granted but the Court declared that the charges against John Holland were valid and that the Industrial Court of New South Wales had jurisdiction to hear those charges. +HIGH COURT OF AUSTRALIA Manager, Public Information 18 June 2009 DAVID RAYMOND SPRIGGS v THE COMMISSIONER OF TAXATION OF THE COMMONWEALTH OF AUSTRALIA MARK RIDDELL v THE COMMISSIONER OF TAXATION OF THE COMMONWEALTH OF AUSTRALIA [2009] HCA 22 Today the High Court decided that two professional footballers, David Spriggs and Mark Riddell, who paid fees to managers to negotiate player contracts, sponsorships and other income earning activities on their behalf, may claim those fees as deductions on their income tax returns. David Spriggs was selected in the Australian Football League (AFL) National Draft in November 1999. In January 2000 he signed a representation agreement with Connors Sports Management (CSM), which provided for a payment to CSM of 3 per cent of total gross earnings for the successful negotiation of an AFL Standard Playing Contract and 20 per cent of total gross earnings in respect of marketing and media activities. In December 2004 CSM negotiated an AFL Standard Playing Contract for Spriggs with the Sydney Swans which provided for a base payment of $70,000 for the 2005 AFL season. CSM issued Spriggs with a tax invoice for $2,100, which represented 3 per cent of the base payment, in respect of “management and promotional services by CSM for season 2004”. Spriggs claimed the amount as a deduction in his 2005 income tax return. The Commissioner of Taxation refused the claim and disallowed Spriggs’ objection to the Commissioner’s assessment. Mark Riddell, a player in the National Rugby League (NRL) competition, agreed with SFX Sports Group (SFX) that, in exchange for the provision of certain services related to the management of Riddell’s affairs, he would pay to SFX 20 per cent of gross monies paid to him for sponsorship, media contracts, endorsements, advertising and promotional work, and 7 per cent of all monies he earned from his NRL Playing Contract. In June 2004 Riddell entered into an NRL Playing Contract with the Parramatta club which provided for an annual playing fee of $275,000 for each of the 2005–2007 seasons. In November 2004 SFX invoiced Riddell $19,250 for “2005 management fees”, which represented 7 per cent of Riddell’s playing fee for 2005. Riddell claimed the amount as a deduction on his 2005 income tax return, however, as with Spriggs, the Commissioner of Taxation refused the claim and disallowed Riddell’s objection to the Commissioner’s assessment. Spriggs and Riddell each appealed to the Federal Court against the Commissioner’s decision. In each case the primary judge set aside the Commissioner’s decision and allowed the players to claim the deductions. On appeal the cases were heard together and the Full Court of the Federal Court unanimously upheld the Commissioner’s appeals. The High Court granted special leave to both Spriggs and Riddell to appeal the Full Court’s decision. Section 8-1(1) of the Income Tax Assessment Act 1997 provides that a person may deduct from his/her assessable income losses or outgoings which are either incurred in gaining or producing assessable income, or necessarily incurred in carrying on a business for the purpose of gaining or producing assessable income. Pursuant to section 8-1(2) losses or outgoings of capital, or of a capital nature, may not be deducted from assessable income. The critical question the High Court had to determine was whether the management fees Spriggs and Riddell paid to CSM and SFX respectively were incurred in gaining or producing their assessable income. The High Court considered that Spriggs and Riddell were each engaged in the business of commercially exploiting their sporting prowess and associated celebrity. These businesses encompassed contracts for employment with their respective playing clubs, and promotional activities on behalf of the AFL or the NRL, their respective clubs, and on their own behalf. Spriggs’ and Riddell’s promotional activities were inextricably linked to their employment as footballers. Spriggs and Riddell conducted their businesses in a commercial fashion, manifested particularly by retaining managers whose duties went well beyond the negotiation of playing contracts. For these reasons the Court held that there was a sufficient connection between the fees charged by CSM and SFX and the gaining or producing of assessable income for the management fees to be deductible. The High Court set aside the orders made by the Full Court of the Federal Court and substituted orders dismissing the Commissioner’s appeals to that Court. +HIGH COURT OF AUSTRALIA 10 August 2022 O'DEA v THE STATE OF WESTERN AUSTRALIA [2022] HCA 24 Today, the High Court allowed an appeal from a judgment of the Court of Appeal of the Supreme Court of Western Australia. The appeal concerned the meaning of s 7(a) of the Criminal Code (WA) and raised the question of whether an accused person who did not actually do the act constituting an offence, or who the Crown could not prove beyond reasonable doubt actually did the act constituting an offence, could be "deemed ... to be guilty of the offence" under s 7(a). The appellant ("Mr O'Dea") was charged jointly with another ("Mr Webb") with the offence under s 294(1)(a) of the Criminal Code of unlawfully doing grievous bodily harm with intent to maim, disfigure, disable or do some grievous bodily harm. The prosecution's case relied in part upon s 7(a) of the Criminal Code, which, "[w]hen an offence is committed", deems "[e]very person who actually does the act ... which constitutes the offence" to be guilty of the offence. The expert evidence led at trial was inconclusive as to which act or acts of Mr O'Dea or Mr Webb, or combination of their acts, caused the victim's traumatic brain injury amounting to grievous bodily harm. The trial judge's directions to the jury relevantly stated that they could convict Mr O'Dea under s 7(a) if they were satisfied beyond reasonable doubt that "Mr O'Dea and Mr Webb were acting in concert, each of them doing one or more of the acts which caused the traumatic brain injury" and that "[t]he relevant accused's acts were unlawful". While Mr O'Dea was convicted of the offence, the jury could not agree upon a verdict in respect of Mr Webb, who was retried and ultimately convicted of the alternative offence of unlawfully doing grievous bodily harm to another contrary to s 297(1) of the Criminal Code. The Court of Appeal dismissed Mr O'Dea's appeal against conviction, concluding that the jury directions given were not erroneous and that it was open to the jury to convict Mr O'Dea under s 7(a) by amalgamating the acts of each accused without concluding that Mr Webb's acts were unlawful. By majority, the High Court held that the Court of Appeal should have found that the trial judge erred in his directions to the jury, which amounted to a miscarriage of justice. The proper approach to s 7(a) requires that the accused person "actually does the act" which constitutes the offence. That approach does not require the recognition of a fictitious implication in s 7(a) that the acts of another person can be attributed to an accused, and treated as "actually" having been committed by the accused, where the accused acts "in concert" with the other person. In the case of an offence under s 294(1)(a) of the Criminal Code, criminal responsibility will only arise under s 7(a) where the act causing the grievous bodily harm is the actual and not the attributed act of the accused person. That interpretation of s 7(a) is consistent with its plain textual meaning, drafting history and statutory context, and with the unchallenged reasoning of the Court in Pickett v Western Australia (2020) 270 CLR 323. +HIGH COURT OF AUSTRALIA 24 May 2007 Public Information Officer VANDA RUSSELL GOULD v DONALD MAGAREY, DAVID OLIFANT AND PATRICK PONTING being the members constituting the COMPANIES AUDITORS AND LIQUIDATORS DISCIPLINARY BOARD AND AUSTRALIAN SECURITIES AND INVESTMENTS COMMISSION RICHARD ALBARRAN v MEMBERS OF THE COMPANIES AUDITORS AND LIQUIDATORS DISCIPLINARY BOARD AND AUSTRALIAN SECURITIES AND INVESTMENTS COMMISSION MILAN VISNIC v AUSTRALIAN SECURITIES AND INVESTMENTS COMMISSION The suspension of two liquidators and the disqualification of a company director were not invalid as they did not involve the exercise of the judicial power of the Commonwealth by a non-judicial body, the High Court of Australia held today. ASIC applied to the Companies Auditors and Liquidators Disciplinary Board for orders suspending Mr Gould’s registration as a liquidator and cancelling Mr Albarran’s registration as a liquidator. The Board suspended Mr Gould’s registration for three months. He applied to the High Court for prohibition against the Board and ASIC to prevent them carrying out the order. He also sought a declaration that certain laws, particularly section 1292 of the Corporations Act which confers powers and functions on the Board, were invalid on the ground that the power to suspend someone’s registration is punitive, hence exclusively part of the judicial power of the Commonwealth which could not be exercised by the Board. Before ASIC’s application relating to Mr Albarran was heard by the Board, he applied to the High Court for an order preventing ASIC from proceeding with the application. The Court refused a stay of the application. It remitted both liquidators’ proceedings to the Federal Court, where both were dealt with by the Full Court. In May 2006, the Full Court held that the Board’s exercise of power under section 1292 does not involve the exercise of the judicial power of the Commonwealth and dismissed both proceedings. In the meantime, the Board had proceeded to a determination adverse to Mr Albarran and ordered a nine-month suspension of his registration. Both Mr Albarran and Mr Gould appealed to the High Court. Mr Visnic was a director of 14 companies which had been wound up. In January 2006, ASIC, exercising power under section 206F of the Corporations Act, disqualified him from managing corporations for five years. ASIC’s statement of reasons said Mr Visnic was not fit to manage corporations as he had demonstrated a lack of responsibility towards creditors, so for the protection of the public he should be prohibited from managing a corporation. In the Federal Court he sought a declaration that section 206F was invalid but the matter was removed into the High Court and heard concurrently with the two appeals. The High Court unanimously dismissed both appeals and Mr Visnic’s action. It held that the Board’s function is not to find whether an offence has been committed and, if so, to inflict a punishment, but is to assess whether someone should continue to occupy a statutory position involving skill and probity, in circumstances where the Board is satisfied that the person has failed in the performance of their professional duties. The Court held that section 1292 confers upon the Board a power that is administrative in nature, not a power of an essentially judicial character, which may be conferred only upon courts exercising federal jurisdiction. It held that section 206F confers upon ASIC a power to be exercised for the maintaining of professional standards in the public interest. Section 206F empowers ASIC to determine that a person no longer manage corporations and may look to the person’s corporate conduct but there is no determination of guilt with respect to any offence. The Court held that in acting under section 206F ASIC does not exercise the judicial power of the Commonwealth. Both sections 206F and 1292 are constitutionally valid. +HIGH COURT OF AUSTRALIA 30 August 2007 Public Information Officer SZATV v MINISTER FOR IMMIGRATION AND CITIZENSHIP AND REFUGEE REVIEW TRIBUNAL SZFDV v MINISTER FOR IMMIGRATION AND CITIZENSHIP AND REFUGEE REVIEW TRIBUNAL Relocation to another part of a country of nationality by applicants for protection visas may sometimes be reasonable, the High Court of Australia held in two judgments today. SZATV, from Chernovtsky in what is now Ukraine, trained as a civil engineer but worked as a journalist. He wrote stories on regional government corruption but after a campaign of alleged intimidation he came to Australia in 2001. He was refused a protection visa. That decision was affirmed by the Refugee Review Tribunal in 2003. The RRT accepted that SZATV had been subjected to systematic harassment, including physical mistreatment, but held that the persecution was localised and he could reasonably move to another part of Ukraine so his fears of persecution upon returning to Ukraine were not well-founded. Appeals to the Federal Magistrates Court and the Federal Court of Australia were dismissed. SZATV appealed to the High Court. The appeal was heard with the appeal by SZFDV also concerning the internal relocation principle. SZFDV, from the Indian State of Tamil Nadu, claimed his family were Communist sympathisers and that his brother was killed by members of the major Tamil parties, the DMK and AIADMK. At the mill where he worked he was elected a trade union leader and was involved in pay disputes. SZFDV said the mill owners held him responsible for the mill’s closure by government order in 2002, that the owners used their DMK influence to procure the laying of false charges of murder of a DMK leader, and that he was assaulted by DMK members and his family threatened. He moved to the capital Chennai (formerly Madras) where he continued to fear DMK persecution. He left for Australia in May 2004. An application for a protection visa was refused. This decision was upheld by the RRT, the Federal Magistrates Court and the Federal Court. The RRT noted that the adjoining State of Kerala had a large Tamil-speaking community and that the Communist Party had a significant presence. SZFDV appealed to the High Court. The Court unanimously allowed SZATV’s appeal but refused SZFDV’s appeal by a 4-1 majority. It held that whether it is reasonable or practicable to relocate to another part of the country depends upon the particular circumstances of the applicant and the impact upon that person of relocation. Differential treatment in matters like race or religion may be encountered in parts of a country whereas in other parts there is insufficient basis for a well-founded fear of persecution. In other cases, the conduct or attribute of an individual attracting persecution may be unrelated to regional geography. In SZATV’s case, the Court held that the effect of the RRT’s decision was that SZATV was expected to move elsewhere, not work as a journalist, and live discreetly so as not to attract the attention of authorities. It held that the RRT had not properly considered whether his fear of persecution was well-founded and ordered the RRT to reconsider SZATV’s application for review. In SZFDV’s case, the Court held that it may be reasonable to relocate in the country of nationality to a region where objectively there is no appreciable risk of recurrence of the feared persecution. It held that the RRT had properly considered that SZFDV could safely relocate to Kerala and that it would not be unreasonable to expect him to do so. +HIGH COURT OF AUSTRALIA 3 May 2017 RODNEY PETER PICKERING v THE QUEEN [2017] HCA 17 Today the High Court unanimously allowed an appeal from a decision of the Court of Appeal of the Supreme Court of Queensland. The High Court held that a miscarriage of justice occurred in the trial of the appellant because the trial judge did not direct the jury on s 31(1)(c) of the Criminal Code (Q) ("the Code"). The appellant was tried before a judge and jury on a count of murder. The Crown alleged that the appellant had stabbed the deceased. The appellant was acquitted of murder but convicted of manslaughter, which was available as an alternative verdict under the Code. The appellant appealed against his conviction to the Court of Appeal on the sole ground that a miscarriage of justice had occurred because s 31(1)(c) of the Code was not left to the jury. Under s 31(1)(c), a person is not criminally responsible for an act if the act is reasonably necessary in order to resist actual and unlawful violence threatened to the person. But under s 31(2), that protection does not extend, among other things, "to an act … which would constitute the crime of murder, or an offence of which grievous bodily harm to the person of another, or an intention to cause such harm, is an element". Grievous bodily harm or an intention to cause such harm is not an element of the offence of manslaughter in Queensland. It was not in dispute, and the Court of Appeal accepted, that s 31(1)(c) was fairly raised on the evidence at trial unless it was excluded by s 31(2). The Court of Appeal held that s 31(2) applied if, on the evidence, the act for which an accused seeks to avoid criminal responsibility would constitute an offence described in s 31(2), irrespective of whether such an offence had actually been charged. The Court of Appeal held that s 31(1)(c) did not avail the appellant, because his act of stabbing the deceased in the way he did was an act that would constitute the offence of unlawfully doing grievous bodily harm to another, that being an offence described in s 31(2). The Court of Appeal therefore dismissed the appellant's appeal. By grant of special leave, the appellant appealed to the High Court. The High Court held that s 31(2) of the Code applies to an act only if the accused has been charged in relation to that act with an offence described in s 31(2) and seeks to invoke s 31(1) to deny criminal responsibility on that charge. Therefore, s 31(1) is not available to deny criminal responsibility on a charge of any of the offences described in s 31(2), but may be available in relation to any other offence that is charged or that is available as an alternative verdict. In this case, the protection afforded by s 31(1)(c) was available to the appellant in relation to the offence of manslaughter, which was not an offence described in s 31(2). The appellant's conviction was quashed and a new trial ordered. +HIGH COURT OF AUSTRALIA 10 December 2014 ARGOS PTY LTD & ORS v SIMON CORBELL, MINISTER FOR THE ENVIRONMENT AND SUSTAINABLE DEVELOPMENT & ORS [2014] HCA 50 Today the High Court unanimously allowed appeals by two supermarket businesses and held that they are persons aggrieved by the decision of the Minister for the Environment and Sustainable Development to approve an application for a commercial development at a site near their premises. By majority, the Court dismissed an appeal by the landlord of one of the businesses and held that the landlord is not a person aggrieved by the Minister's decision. In 2011, the Minister made a decision under s 162 of the Planning and Development Act 2007 (ACT) to approve a development application made by the second and third respondents for a commercial development, which was to include a supermarket and specialty shops, at the Giralang Local Centre. The second and third appellants each conduct a supermarket business at a Local Centre near Giralang, and the first appellant is the second appellant's landlord. Section 5(1) of the Administrative Decisions (Judicial Review) Act 1989 (ACT) ("the ADJR Act") provided, at the relevant times, that a person aggrieved by a decision was entitled to apply to the Supreme Court of the Australian Capital Territory to have that decision reviewed on one or more stated grounds. Section 3B(1)(a) provided that a reference to a "person aggrieved" by a decision includes a reference to "a person whose interests are adversely affected by the decision". The appellants sought judicial review of the Minister's decision under the ADJR Act. The primary judge accepted that, by increasing competition and reducing the annual turnover of the Local Centres at which they conduct their businesses, the development will have an adverse economic effect on the second and third appellants. The primary judge also accepted that the economic interests of the first appellant may be "indirectly affected" by the development. But the primary judge concluded that none of the appellants were persons aggrieved by the Minister's decision, because the adverse effects were "too remote" and the appellants' interests were not sufficiently "directly affected". The Court of Appeal dismissed the appellants' appeals, stating that "[a]s a general rule mere detriment to the economic interests of a business will not give rise to standing". By special leave, the appellants appealed to the High Court. The High Court rejected the existence of such a general rule and unanimously held that, on the facts established before the primary judge, the second and third appellants had demonstrated that they were each a person whose interests are adversely affected by the Minister's decision and that they are therefore each a person aggrieved by the decision. By majority, the Court held that the first appellant had not established that its interests are adversely affected by the Minister's decision. The Court also held, by majority, that the statutory criterion for standing under s 3B does not alter according to the scope and purpose of the enactment under which a decision is made or purported to be made. +HIGH COURT OF AUSTRALIA 11 February 2015 LAVIN & ANOR v TOPPI & ORS [2015] HCA 4 Today the High Court unanimously dismissed an appeal from a decision of the Court of Appeal of the Supreme Court of New South Wales, which had held that the first and second respondents ("the respondents"), being sureties who paid a creditor a disproportionate amount of a guaranteed debt, were entitled to recover contribution from their co-sureties, the appellants, notwithstanding that the creditor had covenanted not to sue the appellants for payment of the guaranteed debt. In 2008, the National Australia Bank ("the Bank") consolidated various loans into one loan to Luxe Studios Pty Ltd. The consolidated loan was guaranteed by, among others, the appellants and the respondents. In 2010, the Bank made demands upon each of the guarantors for payment of the balance of the loan debt. When those demands were not met, the Bank commenced proceedings against all of the guarantors to enforce the guarantee. The appellants and the Bank entered into a deed of release and settlement whereby the Bank covenanted not to sue the appellants if the first appellant paid, relevantly, a minor portion of the guaranteed debt. The first appellant paid that portion and the Bank's covenant not to sue took effect. Thereafter, the respondents paid the remaining major portion of the guaranteed debt. The respondents commenced proceedings in the Supreme Court of New South Wales claiming contribution from the appellants in respect of the amount of their payment in excess of their proportionate share of the guaranteed debt. The appellants resisted the respondents' claim on the basis that the appellants and respondents were not under "coordinate liabilities" because, by reason of the Bank's covenant not to sue, the respondents' liability under the guarantee was enforceable while the appellants' liability was not. The primary judge rejected that argument and gave judgment for the respondents. The Court of Appeal dismissed the appellants' appeal. The Court of Appeal held that the Bank's covenant not to sue did not alter the appellants' liability under the guarantee and, as a result, the appellants and the respondents continued to share "coordinate liabilities" of the same nature and extent so as to entitle the respondents to recover contribution. By grant of special leave, the appellants appealed to the High Court. The High Court unanimously dismissed the appellants' appeal. The Court held that the Court of Appeal was correct in holding that the Bank's covenant not to sue did not extinguish the appellants' liability under the guarantee. Further, the respondents, subject to proving their readiness and ability to perform their own obligations under the guarantee, were entitled in equity to contribution from the time the appellants and the respondents were called upon to satisfy the guarantee. That entitlement could not be defeated by the Bank giving the appellants a covenant not to sue. +HIGH COURT OF AUSTRALIA Public Information Officer 24 April 2008 SB v THE QUEEN OAE v THE QUEEN Evidence of sexual misconduct against a complainant, other than the subject of the charges brought against the alleged offender, may be given in sexual assault trials for limited purposes and with appropriate directions from trial judges, the High Court of Australia held today. Such evidence was subject to a requirement to exclude it if necessary standards concerning probative value and prejudicial effect were not met. HML was convicted in 2006 of two counts of unlawful sexual intercourse by fellatio and sodomy with his nine-year-old daughter. He and his wife separated when the girl was a baby and he did not see her for several years. The daughter, who lived in South Australia, began access visits to her father, who lived a short distance away in Victoria. The offences allegedly occurred in 1999 when she accompanied HML to Adelaide for eye surgery. HML allegedly said after the anal penetration: “Why isn’t it working? It’s worked before.” When the allegations came to police attention in 2003, HML was questioned by a Victorian detective in Mount Gambier about incidents in Victoria. He has not been charged in Victoria but some of these “uncharged acts” (which were not necessarily criminal) were adduced at his trial in SA. SB was convicted in 2006 of three counts of indecent assault and two counts of incest involving his daughter in 1983 when she was 13 or 14 and in 1986 when she was 17 and visiting. Evidence of other “uncharged acts” was received. OAE was charged with indecently assaulting his sister’s foster daughter in mid-1999 when she was 12 and of digitally penetrating her in 2003. He was convicted of the latter offence only. OAE owned a horse stud and his sister lived on the adjoining property. The foster daughter often helped OAE with his horses. Evidence of “uncharged acts” was allowed at trial. HML and SB appealed to the High Court. OAE’s application for special leave to appeal was heard with the two appeals and was argued as on appeal. The Court unanimously dismissed the appeals by HML and SB. OAE’s application for special leave to appeal was granted unanimously, but the appeal was dismissed by a 4-3 majority. The Court in HML and SB and the majority in OAE held that the “uncharged acts” evidence in each case was admissible and that the judges’ directions to the juries, including warnings against the use of propensity reasoning, were sufficient. +HIGH COURT OF AUSTRALIA Public Information Officer 10 September, 2003 JOHN FAIRFAX PUBLICATIONS PTY LIMITED v RENE RIVKIN The High Court of Australia today ordered a retrial in relation to five imputations stockbroker Rene Rivkin claimed could be drawn from articles run in The Australian Financial Review and The Sydney Morning Herald in February 1998. The two stories and a second Sydney Morning Herald story in March 1998 concerned the mysterious death of model Caroline Byrne, the girlfriend of Mr Rivkin’s employee Gordon Wood and Mr Rivkin’s business and other activities. In a New South Wales Supreme Court defamation trial in 2001, the jury rejected all 17 imputations that Mr Rivkin said could be drawn from the three stories. As the jury found none of the pleaded imputations were conveyed it was unnecessary for the jury to determine whether any of them were defamatory of Mr Rivkin. He appealed to the Court of Appeal which unanimously held that no reasonable jury could find that the first two stories did not contain at least six of the alleged imputations and ordered a general retrial. It also held, by majority, that Justice Carolyn Simpson should have given Mr Rivkin’s counsel a right of reply after Fairfax’s counsel gave the final address to the jury. Two imputations concerned Mr Rivkin’s conduct in relation to the Offset Alpine Press Group, two were that Mr Rivkin was criminally liable in respect of the murder of Ms Byrne, another was that he had engaged in homosexual intercourse with Mr Wood, and the sixth was that Mr Rivkin associated with criminals. The High Court, by a 4-1 majority, held that a retrial was required. Three members of the Court held that the trial should consider five potential imputations, one member held that the trial should consider four. The fifth member of the Court would have allowed Fairfax’s appeal in its entirety. The Court unanimously rejected Mr Rivkin’s claim of a right of reply, holding there was no error in the ruling of the trial judge, Justice Carolyn Simpson, on the point. +HIGH COURT OF AUSTRALIA 12 September 2018 THE QUEEN v DENNIS BAUER (A PSEUDONYM) [2018] HCA 40 Today the High Court unanimously allowed an appeal from a decision of the Court of Appeal of the Supreme Court of Victoria which had quashed the respondent's convictions for sexual offences. The respondent was charged with 18 sexual offences committed against the complainant over a period of about 11 years. The Crown applied to adduce a recording of the complainant's evidence from a previous trial ("the recording"). It also applied to adduce as tendency evidence: evidence of the complainant of the acts comprising Charges 1 and 3 to 18; evidence of a witness, TB, of the act comprising Charge 2 and an uncharged sexual act involving the complainant; and evidence of the complainant of several uncharged sexual acts ("the tendency evidence"). The Crown sought to adduce the tendency evidence in order to establish that the respondent had a sexual interest in the complainant and a willingness to act upon it. The Crown further sought to call evidence of a complaint made by the complainant to another witness, AF, in which the complainant disclosed that she had been sexually assaulted by the respondent ("the complaint evidence"). The respondent objected to the admissibility of the recording, the tendency evidence, and the complaint evidence. He also contended that Charge 2 should be severed from the indictment and tried alone. The trial judge allowed the recording, the tendency evidence, and the complaint evidence, to be admitted and rejected the respondent's application that Charge 2 be severed and tried alone. The respondent was convicted of all charges. The respondent appealed against conviction to the Court of Appeal, arguing that each of the trial judge's rulings in respect of the recording, the tendency evidence, the complaint evidence, and severance was wrong. The Court of Appeal allowed the appeal, holding that the trial judge had been wrong: to admit the recording, because it had not been shown that the claimant was unwilling to give evidence; to admit the tendency evidence, because neither the complainant's evidence nor TB's evidence had any "special feature"; not to order that Charge 2 be severed and tried alone, because TB's evidence of Charge 2 was not cross-admissible in relation to the other charges; and to admit the complaint evidence, because there was no evidence that the relevant facts were fresh in the complainant's memory when she made the complaint, and, in any event, because the probative value of the evidence was so slight as not to outweigh the risk of unfair prejudice. The Court of Appeal found that a substantial miscarriage of justice had been occasioned by the admission of the tendency evidence and the complaint evidence. The Court of Appeal quashed the respondent's convictions and ordered that a new trial be had. By grant of special leave, the Crown appealed to the High Court. In respect of the recording, the Court held that, in view of the complainant's strong preference to avoid giving evidence again if at all possible, and in the absence of competing considerations, no error had been shown in the trial judge's conclusion that it was in the interests of justice that the recording be admitted. The Court further held that the trial judge had been correct to conclude that the complainant's evidence was admissible as tendency evidence. All of the charged and uncharged acts were alleged to have been committed against the one complainant, and none of them was far separated in point of time or far different in nature and gravity from the others. These characteristics meant that there was no need for the evidence to have any "special feature" in order to render the evidence of one charge cross- admissible in proof of the other charges, or to render the evidence of uncharged acts admissible in proof of the charged acts. The complainant's evidence had very high probative value because it showed that the respondent was sexually attracted to the complainant and that he acted upon that attraction by engaging in sexual acts with her, making him more likely to seek to continue to give effect to the attraction by engaging in further sexual acts with the complainant as the opportunity presented. Nor was the complainant's evidence productive of unfair prejudice to the respondent. The complainant's evidence of each charged and uncharged act was cross-admissible in proof of each of the charges. The Court also held that TB's evidence of Charge 2 had significant probative value and was admissible in relation to each other charged act as evidence of the respondent's sexual attraction to the complainant and his tendency to act upon it when the opportunity presented, and that there was no real risk of the jury using that evidence in an unfair way as to justify its exclusion. Therefore, the trial judge had been correct to hold that there was no basis for Charge 2 to be severed and tried alone. Finally, the Court held that the complaint evidence was admissible as there was evidence from which it could be inferred that the facts were fresh in the memory of the complainant when she made the complaint and the probative value of the evidence sufficiently outweighed its prejudicial effect. +HIGH COURT OF AUSTRALIA 27 February 2013 MAN HORAN MONIS v THE QUEEN & ANOR AMIRAH DROUDIS v THE QUEEN & ANOR [2013] HCA 4 Today the High Court dismissed two appeals from a decision of the Court of Criminal Appeal of the Supreme Court of New South Wales, which held that s 471.12 of the Criminal Code (Cth) is valid insofar as it makes it a crime to use a postal or similar service in a way that reasonable persons would regard as offensive. The appellants allegedly sent letters (and in one case a recorded message) to the relatives of Australian soldiers killed in action in Afghanistan and to the mother of an Austrade official killed in Indonesia. The communications criticised Australia's military involvement in Afghanistan. They opened with expressions of sympathy for the grieving relatives but then proceeded to criticise and condemn the deceased person. Section 471.12 makes it a crime for a person to use a postal or similar service "in a way ... that reasonable persons would regard as being, in all the circumstances, menacing, harassing or offensive". One appellant was charged with 12 counts of using a postal service in an offensive way and one count of using a postal service in a harassing way, and the other appellant was charged with eight counts of aiding and abetting in the commission of some of those offences. In the District Court of New South Wales, the appellants unsuccessfully sought orders quashing the indictment on the basis that s 471.12 was invalid because it was inconsistent with the implied constitutional freedom of political communication. The Court of Criminal Appeal dismissed an appeal to that Court. By special leave, the appellants appealed to the High Court. The High Court unanimously held that s 471.12 restricted political communication, but divided in assessing the purpose of s 471.12. Three Justices would have dismissed the appeals, holding that the section protects against the misuse of the postal service to deliver seriously offensive material into a person's home or workplace in a manner which is compatible with the system of representative and responsible government established by the Constitution. Three Justices would have allowed the appeals, holding that the end pursued by the section is neither legitimate nor implemented in a manner that is compatible with the constitutional system of government. Where the High Court is equally divided in opinion, s 23(2)(a) of the Judiciary Act 1903 (Cth) provides that the decision appealed from shall be affirmed. Accordingly, the High Court ordered that the appeals be dismissed. +HIGH COURT OF AUSTRALIA 17 June 2004 NORTH AUSTRALIAN ABORIGINAL LEGAL AID SERVICE INC v HUGH BURTON BRADLEY AND THE NORTHERN TERRITORY OF AUSTRALIA Mr Bradley was validly appointed Chief Magistrate of the Northern Territory in March 1998, the High Court of Australia held today, dismissing a challenge from the NAALAS. When appointed Mr Bradley indicated he wanted to serve only two years. Later he changed his mind and two further determinations were made by Territory Administrator Neil Conn in November 1999, one to revoke the 1998 determination and one to reappoint him until the mandatory retirement age of 65, which Mr Bradley attains in 2009. In the NT Supreme Court in 2000 the NAALAS sought a declaration that his appointment was invalid. The action was transferred to the Federal Court of Australia. Justice Mark Weinberg dismissed the NAALAS application in 2001 and the Full Court of the Federal Court dismissed an appeal. On appeal to the High Court, the NAALAS argued the NT Magistrates Act was invalid insofar as it authorised the appointment of a Chief Magistrate to age 65 but with remuneration fixed for only the first two years. The High Court accepted the NAALAS proposition that under the Constitution a court capable of exercising Commonwealth judicial power (as the NT Magistrates Court was) be, and appear to be, an independent and impartial tribunal, but the Court held that this requirement had not been infringed. Section 6 of the Magistrates Act provides that a magistrate shall be paid such remuneration and allowances and hold office on such terms and conditions as the Administrator from time to time determines. The Court held that this should be construed as impliedly conferring on the Administrator the statutory authority to make determinations from time to time and the power to make determinations necessarily carries a power to revoke them. The phrase “from time to time” was not to be read as permitting the Administrator to fail to exercise power under section 6 where that failure would produce a hiatus where no determination was in operation. There was no such hiatus as the 1999 determinations preceded the end of the two-year period covered by the 1998 determination, and none was contemplated or provided for by section 6. The Court held that NT magistrates were not inappropriately dependent on the legislature or executive in a way that compromised or jeopardised their integrity. Rather, the legislative requirement of continued attention by the NT executive to preserving adequate remuneration of magistrates was apt to defend the interests of judicial independence and impartiality. The High Court unanimously dismissed the appeal. +HIGH COURT OF AUSTRALIA 30 October 2013 TERRENCE JOHN DIEHM & ANOR v DIRECTOR OF PUBLIC PROSECUTIONS (NAURU) [2013] HCA 42 Today the High Court unanimously dismissed an appeal from a decision of the Supreme Court of Nauru, which had found the appellants guilty of rape following a trial by judge alone. The first and second appellants, who were husband and wife respectively, were convicted of the rape of a woman regarded by custom as the wife's niece. The husband was charged as the principal in and the wife as an accessory to the rape. The complainant had consented to intercourse but it was alleged by the prosecution that her consent was given only by virtue of threats or intimidation. The complainant gave evidence that she came to the appellants' house following a quarrel with her boyfriend. Over the course of the following day, the wife repeatedly suggested to her that she have sex with the husband, and she repeatedly rejected that suggestion. On the second evening, she became uncomfortable and went into the children's bedroom to make phone calls to a friend and to the complainant's mother. During that phone call, the wife forced open the door and held a knife to the complainant's face. She was led to the living room where the husband was lying on a mattress. The husband had sexual intercourse with her and the wife performed cunnilingus on her. The complainant's mother notified the police and two police officers attended the scene. One of those police officers testified that she asked both appellants whether the complainant was in the house, and they both said no. The second officer was not called to give evidence, but his police report stated that the question was whether "a lady" was "locked up" in the house. The first appellant testified that he had been asked if he had a girl "locked up" and had truthfully denied that he did. The trial judge referred to the second officer's statement, although it was not in evidence, in order to determine whether the failure to call that witness could give rise to a miscarriage of justice. The husband and wife appealed to the High Court pursuant to the Nauru (High Court) Appeals Act 1976 (Cth). They argued that a reasonable tribunal of fact could not have concluded beyond reasonable doubt that the appellants were guilty of rape without the testimony of the second officer. They submitted that the prosecutor should have called the second officer as a witness, or the trial judge should have called him of his Honour's own motion. They also objected to the trial judge's reference to the statement of the second officer, which was not in evidence. The High Court unanimously dismissed the appeal. It held that the failure to call the second officer did not give rise to a miscarriage of justice, having regard to the other evidence which strengthened the prosecution case. It further held that the statutory obligation of the trial judge to call a witness under s 100(1) of the Criminal Procedure Act 1972 (Nauru) was not enlivened because his evidence was not essential to the just decision of the case, and that the trial judge's reference to the police report was to determine the effect of the failure to call the second officer and occasioned no breach of natural justice. The verdict was not unreasonable. +HIGH COURT OF AUSTRALIA Manager, Public Information 14 April 2010 LEHMAN BROTHERS ASIA HOLDINGS LIMITED (IN LIQUIDATION) v CITY OF SWAN & ORS LEHMAN BROTHERS HOLDINGS INC v CITY OF SWAN & ORS [2010] HCA 11 On 30 March 2010 the High Court pronounced orders dismissing appeals against a decision of the Full Court of the Federal Court which had held that a deed of company arrangement ("DOCA") for Lehman Brothers Australia Limited ("Lehman Australia") was void and of no effect. Today the High Court published its reasons for dismissing the appeals. On 26 September 2008 administrators were appointed to Lehman Australia. Following the recommendation of the administrators, a majority of creditors in both number and value (which included other companies in the Lehman Group) passed a resolution that Lehman Australia execute a DOCA. On 12 June 2009 such a deed was executed by Lehman Australia, its administrators, and Lehman Brothers Asia Holdings Ltd ("Lehman Asia"). In proceedings before the Federal Court of Australia, several creditors of Lehman Australia (the first to third respondents in the High Court) claimed that the DOCA purported to provide a moratorium on and release of their claims against other companies in the Lehman Group. They submitted that such a release was not within the scope of Pt 5.3A of the Corporations Act 2001 (Cth) ("the Act") and that they were therefore not bound by the DOCA. Section 444D(1) of the Act, which is in Pt 5.3A, provides that a DOCA "binds all creditors of the company, so far as concerns claims arising on or before the day specified in the deed". The creditors submitted that the word "claims" in s 444D(1) referred only to claims against the company the subject of the DOCA – here, Lehman Australia. Justice Rares reserved the issue for the consideration of the Full Court of the Federal Court and on 25 September 2009 the Full Court held that the DOCA was void and of no effect. Justice Rares subsequently made a declaration that the DOCA was void and ordered that Lehman Australia be wound up by the Court. Lehman Asia and Lehman Brothers Holdings Inc ("Lehman Holdings") were granted special leave to appeal to the High Court and on 30 March 2010 the Court pronounced orders dismissing both appeals. In its reasons delivered today, the Court held that there was no textual footing for reading the word "claims" in s 444D(1) as including claims against persons other than the company the subject of the DOCA. Lehman Holdings had submitted that the words "so far as concerns claims" required "the existence of a causal connection or association between the claim in question and a claim against the insolvent company" and that, because claims against it and other companies in the Lehman Group arose out of the same transactions as were the subject of the claims against Lehman Australia, the claims fell within the terms of s 444D(1). Lehman Asia focused on the interlocking nature of the claims and submitted that the impugned provisions in the DOCA should be seen as part of the "give and take" of a compromise arrangement of claims. The Court held that even if it were accepted that it would be sensible to recognise that a creditor of one group of companies may have interlocking or dependent claims against one or more companies in the group, Pt 5.3A directs attention only to the particular subject company and does not deal with groups of companies. Section 444D(1) alone makes a DOCA binding on creditors. Since creditors are bound under s 444D(1) only to the limited extent identified in that provision, the fact that some creditors (even a majority in number and value) assented to giving up claims against another does not bind other creditors to do so. In making its decision, the Court said that nothing in the reasons should be understood as endorsing the criticisms made in this matter in the Full Court of the Federal Court of the earlier decision of the Full Federal Court in Fowler v Lindholm (2009) 178 FCR 563. +HIGH COURT OF AUSTRALIA 27 August 2008 Public Information Officer MASTER EDUCATION SERVICES PTY LIMITED v JEAN FLORENCE KETCHELL A breach of the Franchising Code of Conduct by a franchisor did not necessarily bring a franchise contract to an end as other remedies were available, the High Court of Australia held today. Section 51AD, in Part IVB of the Trade Practices Act (TPA), provides that a corporation must not contravene an applicable industry code, such as the Franchising Code of Conduct. Clause 11(1) of the Code provides that a franchisor must not enter into a franchise agreement or receive non- refundable money under an agreement unless a prospective franchisee provided a written statement that they had received, read and had a reasonable opportunity to understand the disclosure document and the Code. Master Education Services (MES), as franchisor, gave Ms Ketchell a disclosure document and a copy of the Code before executing a franchise agreement with her on 11 February 2000, but it failed to obtain the statement required by clause 11(1). In August 2003, MES brought proceedings in Campbelltown Local Court to recover $26,043.59 in monthly fees due under the franchise agreement plus interest. Ms Ketchell claimed that MES failed to comply with clause 11 of the Code so that it was unlawful for MES to receive the money and cross-claimed for unconscionable conduct under section 51AC of the TPA. The Magistrate dismissed the claim of unconscionable conduct and found that Ms Ketchell had received legal advice and had obtained various amendments to the franchise agreement through her solicitor. She conducted the business on a trial basis for 12 months before signing the franchise agreement. The Magistrate found that clause 11(1) had not been complied with but that the contravention did not make the contract illegal although damages might be awarded for any loss and damage suffered. The NSW Supreme Court set that judgment aside and remitted it for full determination with respect to non-compliance with clause 11(1) of the Code. The Magistrate gave judgment for Ms Ketchell on the basis that the Court could not require payment since it would involve MES breaching clause 11(1) by receiving the money. MES appealed to the Supreme Court which held that non- compliance did not render the franchise agreement illegal. Ms Ketchell then successfully appealed to the Court of Appeal, which held that section 51AD, read with clause 11 of the Code, directly prohibited the contract and the recovery of money. MES appealed to the High Court. The Court unanimously allowed the appeal. It held that while failure to comply with clause 11(1) was a contravention of section 51AD of the TPA, it did not result in the contract being void and unenforceable. The prohibition in section 51AD was directed to securing compliance by franchisors with industry codes and the consequence of contravention was the grant of remedies provided in Part VI of the TPA. These included compensation for loss and damage, varying the terms of an agreement entered into in breach of the Code, or termination of an agreement. The Court held that the TPA provided a more flexible approach than the common law. Under the common law the agreement may have been rendered void for contravention of section 51AD. Such a result may have harsh consequences and not necessarily be what a franchisee wanted. The Court ordered Ms Ketchell to pay MES $26,043.59 plus interest dating back to 15 August 2003. +HIGH COURT OF AUSTRALIA 17 November 2004 BRUCE DAVID JENKINS v THE QUEEN A jury did not require a warning about the potential unreliability of the evidence of an accomplice when that evidence was not contested, the High Court of Australia held today. Mr Jenkins, 44, a property developer, and his companies obtained loans from the Order of the Sons of Temperance National Division Friendly Society (OST). The loans were arranged on the basis of inflated valuations on various Queensland properties in 1988 and 1989. All were approved by OST investment director Paul Robinson and arranged by mortgage broker Keith Bulfin with a valuation by valuer Tibor Verebes. Although not revealed at Mr Jenkins’s trial, the other three men had all pleaded guilty to offences relating to the loans, with Mr Bulfin and Mr Verebes jailed. Mr Jenkins contracted to buy the inner-Brisbane Glen Crag building for $2.4 million but, on the basis of Mr Verebes’s valuation of $6.67 million, obtained a loan for $4.2 million. Mr Jenkins arranged to buy a High Street, Southport, property for $1.45 million, which Mr Verebes valued at $6.5 million “as is” or $8.17 million after refurbishment. Mr Jenkins obtained a $3.8 million loan, then a further $2.61 million when Mr Verebes revalued High Street at $8.186 million or $9.5 million once upgraded. Mr Jenkins agreed to purchase for $4 million the Ashmore Commercial Centre, valued at $7 million. OST lent $4.5 million. Mr Jenkins negotiated to buy Dreamworld theme park and a group of properties called the Great Adventures Cairns Portfolio for $150 million. Mr Verebes valued Dreamworld at $186.5 million and Great Adventures at $53.6 million, including the Cairns Ferry Terminal at $12 million, later increased to $36.5 million. These valuations became the basis of a $96 million guarantee facility and a $54 million credit facility. Mr Jenkins was convicted of five counts of obtaining financial advantage by deception and five counts of dishonestly furnishing false information for the purpose of obtaining a loan. He was jailed for a total of seven years with a three-and-a-half-year non-parole period. The Victorian Court of Appeal quashed the furnishing false information convictions but upheld the other five. Mr Jenkins appealed to the High Court, arguing that the remaining convictions should also have been quashed because the trial judge, Justice John Coldrey, failed to warn the jury in relation to the evidence of Mr Verebes, who was an accomplice. The Crown argued no such warning was necessary. Mr Jenkins did not testify at his trial but Mr Verebes gave evidence for the prosecution to explain the valuations and the arrangements with Mr Bulfin and OST. The defence did not attack his credibility. Instead the defence case was to challenge inferences the prosecution sought to draw from his evidence by advancing competing inferences. The defence case was that Mr Verebes was an accomplice of Mr Bulfin and Mr Jenkins was an innocent dupe. The defence did not ask Justice Coldrey to warn the jury about accepting Mr Verebes’s uncorroborated evidence and the potential for accomplices to minimise their roles and to exaggerate others’ roles. The High Court held that due to the way the trial was conducted Justice Coldrey was not obliged to give an accomplice warning. The primary facts elicited from Mr Verebes by both sides were largely undisputed. Rather, each side sought to draw competing inferences. The defence counsel’s failure to seek any warning was consistent with their tactical approach to Mr Verebes’s evidence. The Court unanimously dismissed the appeal. +HIGH COURT OF AUSTRALIA 14 September 2012 [2012] HCA 35 On 20 June 2012, the High Court allowed an appeal by the appellant against her conviction for the manslaughter of David Hay, quashing her conviction for manslaughter and directing that a verdict of acquittal be entered. Today the High Court published its reasons for allowing the appeal. The appellant and her husband were registered participants in a methadone programme conducted by a Sydney clinic. They were in the business of selling some of their methadone to friends and acquaintances. On 9 February 2007, the deceased attended the Burns' unit to purchase methadone. His body was discovered in the toilet block at the rear of the block of units the next day. The deceased died as a result of the combined effect of methadone and a prescription drug. The appellant was convicted of manslaughter in the New South Wales District Court. At trial, the prosecution case was left to the jury on the basis that either the appellant was a party to a joint criminal enterprise with her husband to supply methadone to the deceased and that the supply of methadone was an unlawful and dangerous act which caused the deceased's death, or the appellant's failure to seek medical attention for the deceased was a grossly negligent cause of his death. The appellant appealed against her conviction to the New South Wales Court of Criminal Appeal, but that appeal was dismissed. By special leave, the appellant then appealed to the High Court. The High Court allowed the appeal because, as the Crown conceded, the supply of methadone to the deceased without more was not a dangerous act that was capable of supporting the appellant's conviction for unlawful and dangerous act manslaughter. That was one of the two possibilities left to the jury to consider, and it could not be known whether the jury convicted the appellant on this impermissible basis. The Court held that the appellant was not under a legal duty to take steps to preserve the deceased's life and a majority held that the evidence given at the trial was not capable of establishing the appellant's complicity in injecting, or assisting to inject, the deceased with the drug. Accordingly, a majority of the Court declined to order a new trial and the Court made orders quashing the appellant's conviction and entering a verdict of acquittal. +HIGH COURT OF AUSTRALIA 27 July 2016 PACIOCCO & ANOR v AUSTRALIA AND NEW ZEALAND BANKING GROUP LIMITED; PACIOCCO & ANOR v AUSTRALIA AND NEW ZEALAND BANKING GROUP LIMITED [2016] HCA 28 Today the High Court, by majority, dismissed two appeals from the Full Court of the Federal Court of Australia. The majority of the High Court held in the first appeal that late payment fees charged by the respondent ("the Bank") on consumer credit card accounts were not unenforceable as penalties, and in the second appeal that the imposition of late payment fees did not contravene statutory prohibitions against unconscionable conduct, unjust transactions and unfair contract terms. The first appellant ("Mr Paciocco") held two consumer credit card accounts ("the accounts") with the Bank. The terms and conditions of the accounts required Mr Paciocco, following receipt of a monthly statement of account, to pay a minimum monthly repayment. If the minimum monthly repayment plus any amount due immediately was not paid within a specified time, a late payment fee was charged. The late payment fee was $35 before December 2009, and $20 thereafter. 26 late payment fees were charged to Mr Paciocco's accounts. Mr Paciocco and the second appellant, Speedy Development Group Pty Ltd, a company controlled by Mr Paciocco, were applicants in representative proceedings commenced against the Bank in the Federal Court of Australia, in which they alleged that the late payment fees, and various other fees charged by the Bank, were unenforceable as penalties. They also claimed that the Bank engaged in unconscionable conduct under the Australian Securities and Investments Commission Act 2001 (Cth) ("ASIC Act") and the Fair Trading Act 1999 (Vic) ("FTA"), that the contracts for the accounts were made by unjust transactions under the National Credit Code (a schedule to the National Consumer Credit Protection Act 2009 (Cth)), and that the late payment fees were void as unfair terms under the ASIC Act and the FTA. The appellants and the Bank each adduced expert evidence as to the losses suffered by the Bank upon the failure by Mr Paciocco to pay the amounts owing on the accounts by the due date. The expert retained by the appellants provided evidence of the amounts needed to restore the Bank to the position it would have been in had Mr Paciocco paid the amounts owing on time. The expert retained by the Bank provided evidence of the maximum costs that the Bank could conceivably have incurred as a result of Mr Paciocco's late payment, which included loss provision costs, regulatory capital costs, and collection costs. The primary judge held that the approach of the appellants' expert ought to be adopted and that the late payment fees were penalties because, amongst other things, they were extravagant and unconscionable in comparison with the greatest loss that could reasonably be proved. On appeal, the Full Court preferred the approach of the Bank's expert and held that the late payment fees were not penalties because, amongst other things, the legitimate interests of the Bank were affected by each of the categories of costs identified by its expert. The Full Court also rejected the statutory claims raised by the appellants, which were not considered by the primary judge. By grant of special leave, the appellants appealed to the High Court. The majority of the High Court dismissed the first appeal, holding that the Full Court was correct to characterise the loss provision costs, regulatory capital costs and collection costs as affecting the legitimate interests of the Bank. The fact that those categories of costs could not be recovered in an action for damages did not alter that conclusion. Further, neither the fact that the late payment fees were not genuine pre-estimates of damage nor the fact that the amounts charged were disproportionate to the actual loss suffered by itself rendered the late payment fees penalties. The High Court also dismissed the second appeal, the majority rejecting the statutory claims on their merits. \ No newline at end of file